DUNPHY PEDS. EXAM -11
b. once weekly
"Good control" of asthma is measured by the number of times weekly a pt uses a rescue inhaler. What choice below indicates "good control"? a. six times monthly at nighttime b. once weekly c. not more than three times weekly d. not more than once daily and once nightly
Pulse rating
0 = absent 1+ diminished 2+ normal 3+ bounding pg 487
LDL = near optimal
100-129 mg/dl pg 438
LDL = Borderline high
130-159 mg/dl pg 438
Triglycerides = borderline high
150-199 mg/dl pg 438
all other person (pg 372)
15mm is positive in
LDL = high
160-189 mg/dl pg 438
*recent arrivals (< 5 years) *foreign-born persons from high risk countries in Africa, Asia, Latin America *medically underserved low income populations and high risk racial or ethnic minority population *IV drug users *resident/employees of high risk congregate settings, e.g. prison/jails, nursing homes, and other residential settings for elderly, AIDS patients, homeless shelters *mycobacteriology lab personnel *persons with medication conditions known to increase the risk for TB, e.g. diabetes, renal failure, silicosis, immunosuppressive therapy, hematological d/o, gastrectomy, 10% or more < ideal body weight (pg 372)
>10mm is positive in
LDL - very high
>190 mg/dl pg 438
total cholesterol = high
>240 mg/dl pg 438
Triglycerides = very high
>500 mg/dl pg 438
*person with HIV or risk factors and unknown HIV status * recent exposure to clinically active TB, person with organ transplants * persons with chest films indicating healed TB (pg 372)
>5mm is positive in
HDL = High - cardiac protective
>60 mg/dl pg 438
b. Scabies.
A patient is complaining of severe pruritus that is worse at night. Several family members have the same symptoms. Upon examination, areas of excoriated papules are noted on some of the interdigital webs of both hands and the axillae. This is most consistent with : a. Larva migrans. b. Scabies. c. Contact dermatitis. d. Impetigo.
c. that a significant reduction in the overall PSA would occur if the level is associated with true benign prostatic hypertrophy
A. has been on finasteride (proscar) for 6 months for benign prostatic hypertrophy. a decrease in his prostate-specific antigen (PSA) from the original value of 5.4 has not occurred. your initial expectation is: a. that his PSA would remain stble, neither increasing nor decreasing b. A.'s dosage should be reduced only after he has been on the med for approximately 12 months c. that a significant reduction in the overall PSA would occur if the level is associated with true benign prostatic hypertrophy d. that an elevation of the antigen would occur because of the effect of the alpha-adrenergic antagonist
a. to palpate for tenderness, swelling, and crepitus just distal to the lateral epicondyle
A., a 67 y.o female who sustained a fall on an outstretched hand, presents holding her arm against her chest with her elbow flexed. based on the specific location of her pain, you suspect a radial head fx. your best initial assessment strategy to assess for radial head fx would be? a. to palpate for tenderness, swelling, and crepitus just distal to the lateral epicondyle b. to palpate for tenderness, swelling, and crepitus along the radial wrist c. to palpate for tenderness in the "anatomical snuffbox" d. to order an x-ray of the wrist
b. I should rub the area with snow
A., age 36, is planning to go skiing with her fiance. He has warned her about frostbite, and she is wondering what to do if frostbite should occur. You know she's misunderstood the directions when she tells you which of the following? a. I should remove wet footwear if my feet are frostbitten b. I should rub the area with snow c. I should apply firm pressure with a warm hand to the area d. I should place my hands in my axillae if my hands are frostbitten
b. hyperuricemia
A., age 46 presents with a tender, red, swollen knee. you r/o septic arthritis and dx gout by confirming: a. an elevated WBC b. hyperuricemia c. a significant response to a dose of ceftriaxone (Rocephin) d. a positive antinuclear antibody test
b. he should stay away from children and pregnant women who have not had chickenpox
A., age, 57, has just been given a dx of herpes zoster. He asks you about exposure to others. You tell him that: a. once he has been on the medication for a full 24 hrs, he is no longer contagious b. he should stay away from children and pregnant women who have not had chickenpox c. he should wait until the rash is completely gone before going out in crowds d. he should be isolated from all persons except his wife
b. Persistent proteinuria. quiz 624
ACE inhibitors are given to clients with diabetes who have a. Insulin sensitivity. b. Persistent proteinuria. c. An elevated serum creatinine level. d. An elevated glycohemoglobin level.
decreasing interglomerular pressure (pg 586)
ACEi have been found to reduce proteinuria by
c. Sjogren's syndrome
AM sttes that she has a maternal hx of rheumatoid disease but that she has never been affected. today she presents with c/o dryness of the eyes and mouth. what do you suspect? a. rheumatoid arthritis b. systemic lupus erythematosus c. Sjogren's syndrome d. rosacea
prerenal intrarenal postrenal
ARF is classified into three major grps based ont he anatomic nature of the lesions, these major grps are:
a.Inhaled steroids (621 quiz)
According to the NAEPP-3 Guidelines for the Treatment and Management of Asthma, what is the Gold Standard for daily treatment of mild-persistent asthma? a.Inhaled steroids b.Combination therapies c.Long-acting bronchodliators d.Short-acting bronchodilators
Middle ear effusion indicated by: Bulging of TM Decreased or no mobility of TM OR Air fluid level Otorrhea Inflammation indicated by: Distinct erythema OR Distinct otalgia that interrupts normal activities
Acute Otitis Media (AOM (ppt week 2)
a diffuse hyper-pigmentation, especially on skin creases because of increased levels of adrenocorticotropic hormone pg 148
Addison's disease can cause
b. clostridium perfringens pg 191
Alan has necrotizing fasciitis of his right lower extremity. Pressure on his skin reveals crepitus due to gas production by which anaerobic bacteria? a. staphylococcal aureus b. clostridium perfringens c. streptococcus d. s. pyrogenes
Serum calcium (pg 421 table 10.2)
Alice has palpitations that are occurring with muscle twitching, paresthesia, and fatigue. What might a specific diagnostic test be to determine the cause?
a. Watchful waiting with an annual follow-up. quiz 624
Alice, age 48, has a benign thyroid nodule. The most common treatment involves: a. Watchful waiting with an annual follow-up. b. Surgery. c. Administration of levothyroxine therapy. d. Radioactive iodine therapy.
c. Antihistamines (621 quiz)
All of the following are useful in treating patients with COPD except: a. Short-acting beta2-agonists b. Oral steroids c. Antihistamines d. Anticholinergics
no txment is indicated
American Urological Association scale score of 7 or less =
medical txment or surgery can be presented to the clt as an option
American Urological Association scale score of 8-19 or 20-35, then,
The antibiotic of choice for beta-lactamase coverage of otitis media is:
Amoxicillin and potassium clavulanate
a. acute lymphocytic leukemia (ALL) pg 931/935
Amy is 13 y/o and was just dx with leukemia, she is c/o bone and joint pain. Which type of leukemia does she probably have? a. acute lymphocytic leukemia (ALL) b. acute myelogenous leukemia (AML) c. chronic myelogenous leukemia (CML) d. chronic lymphocytic leukemia (CLL)
b. Bouchard's nodes
C., age 62, has swollen, bony proximal interphalangeal joints. you describe these as: a. Heberden's nodes b. Bouchard's nodes c. Osler's nodes d. Murphy's nodes
a. visualizes the cervical, vaginal, or vulvar epithelium under magnification to identify abnormal areas that may require a biopsy
C., says that her healthcare provider wants to do a colposcopy. She ask you what this is. You tell her that a colposcopy: a. visualizes the cervical, vaginal, or vulvar epithelium under magnification to identify abnormal areas that may require a biopsy b. involves removal of one or more areas of the endometrium by means of a curette or small aspiration device without cervical dilation c. allows visual exam of the uterine cavity with a small fiber-optic endoscope passed through the cervix d. allows visualization of the abdominal and pelvic cavity through a small fiber-optic endoscope passed through a subumbilical incision
Confusion - new onset BUN ->19mg per dL Respiratory rate >30breaths/min Blood pressure systolic - 90; diastolic -60 65 or older each get a point 0-1 point tx at home - low risk 2 point - short inpt in hospital or closely watched outpt 3 or > hospitalize and consider ICU (pg 364)
CURB-65 is
d. hepatotoxicity (get a baseline liver function profile and CBC prior and 4 weeks after tx) pg 172
Cali is on systemic antifungals for a severe tinea infection. You are aware that they may cause: a. renal failure b. skin discoloration c. breathing difficulties d. hepatotoxicity
a. smoking
Candy, an older adult, is homeless and has iron-deficiency anemia. She smokes and drinks when she can and has an ulcer. Which of the following is not one of the risk factors of iron-deficiency anemia? a. smoking b. poverty c. ulcer disease d. age older than 60
b. Smoking. 624 quiz
Caroline, an older adult, is homeless and has iron-deficiency anemia. She smokes and drinks when she can and has an ulcer. Which of the following is not one of the risk factors of iron-deficiency anemia? a. Poverty. b. Smoking. c. Age older than 60. d. Ulcer disease.
b. > than 50% pg 116
Carotid endarterectomy should only be considered for symptomatic patients with greater than what percent stenosis? a. > than 25% b. > than 50% c. > than 75% d. only for 100% occlusion
b. as you age, you have decreased pigment in your skin, which puts you at more risk
Cathy, age 64, is a sun worshipper. She tells you that because she did not get skin cancer in her youth, she certainly will not get it now. How do you respond? a. you are probably right; if you haven't had it by now, you are probably safe b. as you age, you have decreased pigment in your skin, which puts you at more risk c. your skin elasticity is decreased, so you have more of a chance of contracting skin cancer as you age d. skin cancer is not dependent on age; anyone can get it
A hordeolum that does not resolve and eventually forms granulation tissue Slow-developing, painless, hard mass Pea-sized nodule within the eyelid, typically top eyelid
Chalazion (ppt week 2)
A palpable preauricular node is usually present Treatment - Doxycycline 100mg BID x 7-10 days or Azithromycin 1 gram po in combination with topicals
Chlamydia Conjunctivitis (ppt week 2)
COPD (pg 349)
Chronic bronchitis and emphysema are grouped together as
b. heart failure
Cilostazol (Pletal) should be used with great caustion in the presence of which of the following dx? a. diabetes mellitus b. heart failure c. hypertension d. dyslipidemia
vasodilation and platelet inhibitor and is used to prevent atherosclerotic ischemic events in pts with PAD - it lengthens the pain-free walking distance for pts with claudication (pg 488)
Cilostazol is a
Heart Failure (pg 488)
Cilostazol should NOT be prescribed in patients with any degree of
a change in glomerular filtration
Cimetidine (Tagamet) can cause an elevation in serum creatinine levels without
b. Mild persistent (pg 342)
Cindy presents with a history of asthma. Her sxs are more than 2x per week, but less than 1x per day, may be several times at night/month, her exacerbations affect her activity and sleep, nighttime asthma sxs are more than twice a month and her PEFR or FEV1:>80% predicted; PFT variability 20-30% How would you classify her asthma severity? a. Mild intermittent b. Mild persistent c. Moderate persistent d. Severe persistent
c. Moderate persistent (pg 342)
Cindy presents with a history of asthma. She has not been treated for a while. She complains of daily symptoms but not continual, greater than 1 week and at nighttime. She has been using her rescue inhaler. Her FEV1 is 60% to 80% predicted. How would you classify her asthma severity? a. Mild intermittent b. Mild persistent c. Moderate persistent d. Severe persistent
d. Severe persistent (pg 342)
Cindy presents with a hx of asthma. She is having continuous daily sxs, frequent nighttime sxs, frequent exacerbations. Her physical activities are limited by asthma and her PEFR or FEV1: <=60% predicted; PFT variability >30% How would you classify her asthma severity? a. Mild intermittent b. Mild persistent c. Moderate persistent d. Severe persistent
b. Cognitive, motor, and behavioral changes. quiz 624
Clients with AIDS typically experience the neurological symptomatic triad consisting of: a. Seizures, depression, and paresthesias. b. Cognitive, motor, and behavioral changes. c. Seizures, paresthesias, and dysesthesias. d. Kaposi's sarcoma, cryptococcal meningitis, and depression.
140 - age X lean body weight in kilogram / 72 stable serum creatinine in mg/dl (this value is multiplies by 0.85 for women **** creatinine clearance is a far more informative diagnostic tool as a measure of renal function (pg 620)
Cockcroft-Gault formula for creatinine clearance =
1.streptococcus pneumoniae (70%) 2.pneumococcal pneumonia (25-35%) 3.Staphyloccus aureus 4. Klebsiella pneumoniae 5. Moraxella catarrhalis (less common) Atypical Pneumonias: 1.mycoplasma pneumoniae (2nd most common cause of CAP) 2.legionella pneumoniae 3.chlamydia pneumoniae 4.fungi 5.oral anaerobes 6. viruses (pg 358)
Common causes of community acquired pneumonia are:
Otitis Media (AOM, OME) Otitis Externa Paroxysmal Positional Vertigo Meniere's disease Presbycusis Cerumen impaction
Common ear conditions (ppt week 2)
outside the hospital, or is dx within 2 days of hospitalization in a pt who has not resided in a long term care facility for 2 weeks or more prior to the onset of sxs (pg 358)
Community acquired pneumonia (CAP) occurs
neurologic disorders pg 77
Confusion is a key sign of
S/S: eye pain hesitant or unable to open eye Foreign body sensation Important to identify situation leading to abrasion - scratching the eye, trauma, or contact lens irritation
Corneal abrasion (ppt week 2)
c. 7-10 days prednisone spurt, Albuterol/Atrovent inhaler 2 puffs QID (621 quiz)
An adult patient with a history of chronic obstructive lung disease (COPD) presents to your office with 1 day of worsening shortness of breath, wheezing, and non-productive cough. You determine he has a COPD exacerbation. What is your treatment plan? a. Chest x-ray b. Z-pak, 7-10 days prednisone spurt, Atrovent inhaler 2 puffs QID c. 7-10 days prednisone spurt, Albuterol/Atrovent inhaler 2 puffs QID d. Levaquin 500mg QD for 7 days, prednisone spurt, Albuterol/Atrovent inhaler 2 puffs QID
smoking obesity inactivity hypertension high cholesterol diabetes (pg 484)
Arterial risk factors for Peripheral Arterial Disease are:
mumps virus, echo virus herpes virus MORE BENIGN, self limited type pg 130
Aseptic meningitis is caused by:
1.intermittent sxs < than 1 time a week; brief exacerbation lasting a few hrs to a few days 2.nighttime asthma sxs < than 2x a month 3.asymptomatic and normal PEF between exacerbations 4.PEFR or FEV1:>80% predicted; PFT variability >20% (pg 342)
Asthma - Mild intermittent classification =
1. Sxs more than 2x per week, but less than 1x per day, may be several times at night/month 2. exacerbations may affect activity and sleep 3. nighttime asthma sxs more than twice a month 4. PEFR or FEV1:>80% predicted; PFT variability 20-30% (pg 342)
Asthma - Mild persistent classification =
1. Sxs daily but not continual, >1 week, nighttimes 2. exacerbations affect activity and sleep 3. daily use of inhaled short-acting beta-2 agonist 4. PEFR or FEV1: 60-80% predicted; PFT >30% (pg 342)
Asthma -Moderate persistent classification =
1. continuous daily sxs, frequent nighttime sxs 2. frequent exacerbations 3. physical activities limited by asthma 4. PEFR or FEV1: <=60% predicted; PFT variability >30% (pg 342)
Asthma -Severe persistent classification =
c. r/o a neurological disorder
Gary, age 5, has a dx of encopresis. After the dx, what would be your next action? a. order extensive lab work b. send Gary to a psychologist c. r/o a neurological disorder d. bring Gary's parents in for counseling
8-10 hrs and is performed before the pt arises, about 50ml of gastric contents is required (pg 372)
Gastric aspiration is done following a period of fasting for
d. rewashing undergarments without fabric softener and applying corticosteroids to the affected area
Gean, a 33 y/o female pt, c/o external vaginal irritation after adding new fabric softener to her laundry. You have dx her with reactive vaginitis. The tx of choice for this condition includes: a. metronidazole (flagyl) 500mg BID x 7 days b. conjugated vaginal estrogen cream externally every day for 1 wk c. rewashing undergarments without fabric softener and applying petroleum jelly to the affected area d. rewashing undergarments without fabric softener and applying corticosteroids to the affected area
a. Stage 1 mild COPD (pg 352)
George has chronic obstructive pulmonary disease (COPD) and an 80% forced expiratory volume in 1 second. How would you classify the severity of his COPD? a. Stage 1 mild COPD b. Stage 2 moderate COPD c. Stage 3 severe COPD d. Stage 5 very severe COPD
d. an NSAID
George, age 52, has a recurring headache every Monday morning. What do you plan to order? a. an EEG b. a CT scan c. an MRI d. an NSAID
Presbycusis - bilateral high-frequency sensorineural hearing loss most common pattern of hearing loss in older adults Differential Diagnosis of Other Causes Cerumen or foreign body impaction AOM, OME, perforation of TM, OE Meniere's disease Ototoxic Medications Tumors, ie. acoustic neuroma
Geriatric ConsiderationsHearing Loss (ppt week 2)
twice weekly during daytime OR twice monthly at nighttime
Good asthma management is characterized by using short-acting bronchodilators (rescue inhaler) no more than
glomerulonephritis and pulmonary hemorrhage resulting from immune complex damage to the glomerular and alveolar basement membranes
Goodpasture's syndrome is characterized by
b. sundowning
Grace, age 82, has Alzheimer's disease. Her daughter states that she is agitated, has time disorientation, and wanders during the afternoon and evening hrs. How do you describe this behavior? a. Alzheimer's dementia b. sundowning c. deficits of the Alzheimer's type d. senile dementia
Major Pathogens in Acute Bacterial Conjunctivitis In Adults
Gram-Positive Organisms Staphylococcus aureus Streptococcus species Gram-Negative Organisms Escherichia coli Pseudomonas species Moraxella species (ppt week 2)
Major Pathogens in Acute Bacterial Conjunctivitis in Children
Gram-Positive Organisms Streptococcus pneumoniae Staphylococcus aureus Gram-Negative Organisms Haemophilius Influenzae Moraxella species (ppt week 2)
immunocompetent
Group A strept can cause a fulminant pneumonia even in pts who are
gram negative bacterium and is the 2nd most common cause of CAP and may occur in healthy as well as debilitated patients and chronic alcohol abuse (pg 360)
H. Influenzae is cause by a
d. spinal fx
H. age 73, presents with a c/o low back pain. red flags inher hx of a minor fall, having osteopenia, and prolonged steroid use for systemic lupus erythematosus suggest the possibility of which of the following serious underlying conditions as the cause of her low back pain? a. cancer b. cauda equina syndrome c. neurological compromise d. spinal fx
d. postvoid residual (PVR) urine measurement
H. has BPH and complains of some incontinence. your first step in dxing overflow incontinence would be to order a: a. urinalysis b. cystometrogram c. cystoscopy d. postvoid residual (PVR) urine measurement
d. Peyronie's disease
H. is c/o crooked, painful erections. he has palpable, nontender, hard plaques just beneath the skin of his penis. what do you suspect? a. carcinoma of the penis b. genital herpes c. syphilitic chancre d. Peyronie's disease
c. bulbocavernous reflex
H. is c/o stress urinary incontinence. to assess the autonomic arch innervating the bladder, you test the a. inguinal reflex b. neuronal reflex c. bulbocavernous reflex d. meatal resistance
c. candida
H. who has diabetes is c/o a rash on his penis. before examining him, you suspect that he may have: a. tinea cruris b. genital herpes c. candida d. intraepithelial neoplasia
c. let pain be your guide and continue activities
H., age 29 is a nurse who has an acute episode of back pain. you have determined that it is a simple "mechanical" backache and order: a. bedrest for 2 days b. muscle relaxants c. let pain be your guide and continue activities d. back-strengthening exercises
a. osteosarcoma
H., age 59 has Paget's disease of the bone. it was dx as a result of routine blood work during his annual physical, which showed an increased serum alkaline phosphate level. you know that the most serious complication of Paget's disease is: a. osteosarcoma b. nerve compression c. fx d. bone pain
c. it may exacerbate your concurrent condition of tinea corporis
H., uses a high potency corticosteroid cream for his dermatoses. You tell him the following: a. you must use this for an extended period of time for it to be effective b. it will work better if you occlude the lesion c. it may exacerbate your concurrent condition of tinea corporis b. be sure to use it daily
plantar warts pg 195
HPV 1 typically causes
anogenital warts pg 195
HPV 6 & 11 causes
hypothyroidism in the U.S., more common in women and the average age at onset from 30-60 yrs pg 846-847
Hashimoto's thyroiditis is the most common etiology of
four
Headaches are classified into how many types?
c. the condition is self-limiting, and most likely complete recovery will occur
Herbert, age 58, has just been dx with Bell's palsy. He is understandably upset and has questions about the prognosis. Your response should be: a. although most of the sxs will disappear, some will remain but can usually be camouflaged by altering your hairstyle or growing a beard or mustache b. unfortunately, there is no cure, but you have a mild case. c. the condition is self-limiting, and most likely complete recovery will occur d. with suppressive drug therapy, you can minimize the sxs
b. temporal lobe location
Herpes simplex encephalitis is characterized by which of the following? a. parietal lobe location b. temporal lobe location c. absence of CSF lymphocytes d. decreased CSF protein levels
Dry eye is more prevalent in
Hispanic and Asian populations pg 245
Focus on symptom of concern Otalgia - deep ear pain or superficial pain Also ask about associated symptoms Examine external ear Otoscopic exam & pneumatic otoscopy Other tests: Weber, Rinne, Romberg, Audiometry
History, Exam, and Testing (ppt week 2)
pain with dorsiflexion of the foot
Homan's sign is
An acutely presenting, erythematous, tender lump within the eyelid is called:
Hordeolum
Acutely presenting, erythematous, tender lump within the eyelid Involves infection or inflammation of hair follicles of the eyelashes along the eyelid margin Caused by blocked meibomian gland or infection in sebaceous glands of the eyelash
Hordeolum (ppt week 2)
Pimple or abscess in either upper or lower lid External. Next to eyelash Internal. Under eyelid Painful swelling Develop suddenly
Hordeolum presents as (ppt week 2)
Treatment Warm compresses 15mins/ 4x per day Do NOT squeeze or pop stye Antibiotic drops if persistent/ edema of eyelid
Hordeolum treatment consists of (ppt week2)
as a cough suppresion (pg 333)
Horehound is suggested
maturational delay
Eric, age 5 has enuresis, the probable cause is
Manipulation of pinna and tragus cause pain Ear canal may be swollen and inflammed with white or green discharge Most common pathogens: Pseudomonas, S. aureus, S. epidermidis, various fungi Laboratory tests are rarely needed - culture of discharge if diagnosis uncertain or patient does not respond to treatment
Examination findings in OE (ppt week 2)
Reduction of visual acuity Ciliary flush Photophobia Severe foreign body sensation that prevents the pt from opening eye Fixed pupil Severe headache/nausea Corneal opacity
Eye - Red flags (ppt week2)
Visual Acuity Penlight & fundoscopic exam Check for pupillary response Foreign body Direct trauma Fluorescein stain and use cobalt blue filter
Eye assessment (ppt week 2)
b. bowel sounds may be heard over a hernia
G. is c/o a scrotal mass; however, the scrotum is so edematous that it is difficult to assess. how do you determine if it is a hernia or a hydrocele? a. you can always return a hernia's contents to the abdominal cavity b. bowel sounds may be heard over a hernia c. you can transilluminate a hernia d. with a hydrocele, a bulge appears on straining
c. Proscar (finasteride)
G. states tht he heard that if he takes a certain pill, it is less likely he will need surgery for his BPH, as well as less likely he will develop acute urinary retention. which of the following meds has these positive outcomes because it is actually disease modifying? a. hytrin (terazosin) b. viagra (s9ldenafil citrate) c. Proscar (finasteride) d. flomax (tamsulosin)
b. hyponatremia
G., age 26, runs marathons and frequently c/o painful contractions of his calf muscles after running. you attribute this to: a. hypercalcemia b. hyponatremia c. heat exhaustion d. dehydration
b. gelling
G., age 48, has rheumatoid arthritis and gets achy and stiff after sitting through a long movie. this is referred to as: a. longevity stiffness b. gelling c. intermittent arthritis d. molding
Inflammation of the lining of the auditory canal A bacterial, viral, or fungal infection of the ear canal is one of the most common causes of otalgia in adults OE occurs when the "acid mantle" and cerumen of the ear fail to protect. Other symptoms: pruritis, edema, and malodorous discharge Risk factors: exposure to water or excessive use of headphones and earplugs
Otitis Externa or swimmer's ear (ppt week 2)
b. MRI scan
P. has a malignant fibrosarcoma of the femur. he recently had surgery and is now on radiation tx. you want to order a test to determine the extend of the tumor invasion of the surrounding tissues and the response of the bone tumor to the radiation. which of the following tests should you order? a. x-ray b. MRI scan c. CT scan d. a needle biopsy
AIDS defining opportunistic infection (pg 361)
PCP is an
peak expiratory flow rate (pg 344) and is used to gauge the patients response to tx. If the PEFR is <80% of the pt's personal best - then readjustment of medication/lifestyle may be needed (pg 344)
PEFR stands for
May be asymptomatic Symptomatic Otalgia without fever - usually does not interfere with normal activities Fullness, tinnitus, vertigo, hearing loss Popping or cracking sounds when yawning or blowing nose Otoscopic exam: effusion, no inflammation
Presentation of OME (ppt week 2)
a. Celtic background (Irish, Scottish, English) pg 234
Prevalence of actinic keratosis is highest in which group? a. Celtic background (Irish, Scottish, English) b. African Americans c. Asians d. Native Americans
a. Scandinavians pg 222
Prevalence of psoriasis is highest in which group? a. Scandinavians b. African Americans c. Asians d. Native Americans
Prevention Good handwashing Proper contact lense care Take off eye makeup each night
Prevention of Hordeolum consists of (ppt week 2)
primary Raynaud disease
Primary Raynaud phenomenon is also known as
d. third-degree uterine prolapse
Procidentia is a: a. cystocele b. rectocele c. vaginal fistula d. third-degree uterine prolapse
b. non-selective
Propranolol is: a. cardio-selective beta blocker b. non-selective beta blocker
Age 3 months to 18 yrs: neisseria meningitidis haemophilus influenzae type B streptococcus pneumoniae Age 18-50 yrs: streptococcus pneumoniae neisseria meningitidis Age 50 or older: streptococcus pneumoniae neisseria meningitidis l. monocytogens gram-negative bacilli HAS a rapid onset: hrs or days after exposure pg 130
Purulent forms of acute meningitis are usually caused by:
rare autosomal recessive disorder that causes chronic hemolytic anemia, usually with the onset in childhood
Pyruvate kinase deficiency is a
b. withhold one or more days of anticoagulant therapy pg 907
If the international normalized ratio (INR) result is above the therapeutic range in a patient with atrial fibrillation on warfarin, what might the clinician do? a. stop the warfarin for 1 week, and then repeat the INR. b. withhold one or more days of anticoagulant therapy. c. restart therapy at a lower dose immediately. d. the prothrombin time (PT) and INR should be reevaluated within 1 month of the dosage adjustments.
rifabutin because rifampin interacts with protease inhibitors and non-nucleoside retroviral inhibitors (pg 374)
In HIV patients which TB medication should always be used?
stage II moderate COPD (pg 343)
Inhaled anticholinergic bronchodilators (tiotropium bromide - atrovert) are the recommended maintenance therapy for
c. refer J. to a specialist
J. comes in for an eval after a testicular self-exam. he states tha tit is probably nothing to worry about because his testicle is not tender, but he does have a tiny, hard nodule on the testicle. you confirm that there is a hard fixed nodule on his testicle. your next course of action would be: a. order a urinalysis b. schedule J. for a recheck next month c. refer J. to a specialist d. tell J. that it it is a cyst and if it does not resolve by itself, he will have it excised
c. you must use protection for at least 6 wks after the procedure
J. has a no-scalpel vasectomy and asks if he can proceed immediately with sexual relations with his wife without worrying about getting her pregnant. you tell him a. yes, you are now sterile b. you must use protection for at least 2 wks after the procedure c. you must use protection for at least 6 wks after the procedure d. in 6 months, we will do a sperm count to see if you con dc other precautions
b. continue breastfeeding the baby to avoid milk stasis
J. has been breastfeeding and has developed puerperal mastitis. You tell her: a. using cool compresses to the affected breast before pumping will increase milk expression b. continue breastfeeding the baby to avoid milk stasis c. continue doing your normal activities during the acute phase to keep things flowing d. do not massage the breasts
c. she may have been excessively anxious and worried on most days for more than 6 months
J. has been dx with a generalized anxiety disorder (GAD). You know that she may experience which of the following? a she may be worried or anxious about having a panic attack b. she may be worried about being separated or about being away from home or close relatives c. she may have been excessively anxious and worried on most days for more than 6 months d. she may have multiple physical complaints or believe she has a physical illness
b. danazol (danocrine)
J. has severe pain monthly with her fibrocystic breast disease. Which medication do you suggest she try? a. micronized estradiol (estrace) b. danazol (danocrine) c. paroxetine (paxil) d. venlafaxine (effexor)
a. production of human chorionic gonadotropin
J. is 7 wks pregnant and experiencing morning sickness. the most common cause for this condition is: a. production of human chorionic gonadotropin b. suppression of estrogen c. suppression of linea alba d. suppression of progesterone
d. caffeine
J. is nursing her 8 wk old baby and states that he is very irritable and sleeps poorly. What medication or substance do you ask her if she is taking or using? a. cimetidine b. ergotamine c. nicotine d. caffeine
b. a high caffeine intake has a diuretic effect that may cause calcium to be excreted more rapidly
J. says that she has heard that caffeine can cause osteoporosis and ask you why? how do you respond? a. caffeine has no effect on osteoporosis b. a high caffeine intake has a diuretic effect that may cause calcium to be excreted more rapidly c. caffeine affects bone metabolism by altering intestinal absorption of calcium and assimilation of calcium into the bone matrix d. caffeine increases bone resorption
b. musculoskeletal strain
J., a 59 y/o cashier, present with back pain with no precipitating event. the pain is located over her lower back and muscles without sciatica, and it is aggravated by sitting, standing, and certain movements. it is alleviated with rest. palpation localizes the pain, and muscle spasms are felt. there was an insidious onset with progressive improvement. what is your initial dx? a. ankylosing spondylitis b. musculoskeletal strain c. spondylolisthesis d. herniated disk
c. stop the amoxicillin
J., age 12, just started taking amoxicillin for otitis media. His mother said that he woke up this am with a rash on his trunk. What is your first action? a. prescribe systemic antihistamines b. prescribe a short course of systemic steroids c. stop the amoxicillin d. cont the drug, this rx on the first day is normal
b. assess for an adequate airway
J., age 16 was involved in a motor vehicle accident. he walks into the office with an obvious facial fx, then collapses. what should your first action be? a. call his parents for permission to tx b. assess for an adequate airway c. obtain head and maxillofacial Ct d. assess for a septal hematoma
a. de-bride and irrigate the wound thoroughly
J., age 16, comes into the office with a human bit on his fist. what is the first course of action? a. de-bride and irrigate the wound thoroughly b. initiate broad-spectrum antibiotics c. leave the wound open for drainage d. administer a tetanus injection
a. relative rest; he could benefit from hamstring stretching, heel cord stretching, and quadriceps stretching exercises
J., age 17 has been c/o a painful knob below his rt knee that has prevented him from actively participating in sports. he has recently been given a dx of Osgood-Schlatter disease and asks you about his tx options. you tell him that the initial tx is: a. relative rest; he could benefit from hamstring stretching, heel cord stretching, and quadriceps stretching exercises b. immobilization; a long-leg knee immobilizer is recommended c. surgical intervention; removal of the bony fragments is necessary d. bedrest for one week
a. bend your knees and face the object straight on
J., age 17 works as a stock boy at the local supermarket. he is in the office for a routine visit. you notice that he had an episode of low back pain 6 months ago from improperly lifting heavy boxes. in discussing proper body mechanics with him to prevent future injuries, you tell him: a. bend your knees and face the object straight on b. hold boxes away from your body at arm's length c. bend and twist simultaneously as you lift d. keep your feet firmly together
c. acute prostatitis
J., age 20, presents to the college health clinic with c/o difficulty passing his urine and a discharge from his penis. upon further investigation you note that the discharge is urethral in origin. the most common cause of these sxs in the young adult male population is: a. chronic prostatitis b. prostatic abscess c. acute prostatitis d. prostatic hypertrophy
d. yes, shedding of the herpes simplex virus from mucocutaneous surfaces int he absence of visible lesions is a primary mode of transmission
J., age 26, who has genital herpes, ask if her partner has to use a condom during sexual intercourse even if she does not have a visible lesion. How do you respond? a. yes, we are not sure if its still transmitted when the lesions are not visible, so it is better to be on the safe side b. no, you are not contagious when the lesions are not visible c. no, use of spermicidal agent is all that is required d. yes, shedding of the herpes simplex virus from mucocutaneous surfaces int he absence of visible lesions is a primary mode of transmission
d. antihistamines
J., age 29 has numerous transient lesions that come and go. and she is dx with urticaria. What do you order? a. aspirin b. NSAIDs c. opioids d. antihistamines
c. trichloracetic acid
J., age 32 is pregnant and has genital warts (condylomata) and would like to have them txed. What should you order? a. benzoyl peroxide b. podophyllin c. trichloracetic acid d. corticosteroids
b. limiting consumption of purine-rich foods
J., age 52 has gout. what do you suggest? a. using salicylates for an acute attack b. limiting consumption of purine-rich foods c. testing his uric acid level q 6 months d. decreasing fluid intake
a. uremia from chronic renal disease
J., age 52, presents with pruritus with no rash present. He has HTN, diabetes, and end-stage renal disease. One of the differential dx would be: a. uremia from chronic renal disease b. contact dermatitis c. lichen planus d. psoriasis
a. sheep and goats
J., age 58 is a farmer. He presents with a painful finger ulcer and a palpable olecranal lymph node. Suspecting an oft skin ulcer., you ask him if he works with: a. sheep and goats b. horses c. metals d. tile
a. have an excisional biopsy
J., age 59 has a nevus on his shoulder that has recently changed from brown to bluish black. You advise him to: a. have an excisional biopsy b. monitor the nevus for a change at the end of 1 month c. apply benzoyl peroxide solution d. apply hydrocortisone 1% cream
c. psoriasis
J., age 59, presents with recurrent, sharply circumscribed red papules and plaques with a powdery white scale on the extensor aspect of his elbows and knees. What do you suspect? a. actinic keratosis b. eczema c. psoriasis d. seborrheic dermatitis
a. if the dog was a domestic pet that had been vaccinated, the wound should be cleaned and irrigated
J., age 6 was bitten by a dog. Her mother asks you if the child needs antirabies tx. You tell her: a. if the dog was a domestic pet that had been vaccinated, the wound should be cleaned and irrigated b. antirabies tx must be started immediately c. rabies can be contracted only through the bites of wild animals d. wait until you have observed the animal for 2 wks to determine if it is rabid
a. no txment at this time
J., age 62 has a low International Prostate Symptom Score for lower urinary tract sxs associated with his BPH. you recommend: a. no txment at this time b. immediate referral to urology c. balloon dilation d. starting him on an alpha blocker
d. adequate to perform only few or none of the duties of usual occupation or self-care
J., age 69, has a class III case of rheumatoid arthritis. according to the American Rheumatism Association, her function would be: a. adequate for normal activities despite a handicap of discomfort or limited motion of one or more joints b. largely or wholly incapacitated, bedridden, or confined to a wheelchair permitting little or no self-care c. completely able to carry on all usual duties without handicaps d. adequate to perform only few or none of the duties of usual occupation or self-care
b. change the position of J.'s arm every hr
J., age 87, broke her wrist after falling ff a curb.she just had a plaster cast applied to her wrist. in instructing her family on allowing the cast to dry properly, tell them to: a. continuously elevate J.'s arm on a pillow b. change the position of J.'s arm every hr c. position a fan near J.'s during the night to ensure even drying of the cast d. put a blanket over the cast to absorb the dampness
b. cellulitis
J., an obese 42 y/o, cut his rt leg 3 days ago while climbing a ladder. Today his rt lower leg is warm, reddened, and painful without a sharply demarcated border. What do you suspect? a. diabetic neuropathy b. cellulitis c. peripheral vascular disease d. a beginning stasis ulcer
c. phenobarbital (luminal)
J., has a seizure disorder and wants to get pregnant. What is the drug of choice for her during pregnancy a. valproate (depakene) b. trimethadione (tridione) c. phenobarbital (luminal) d. phenytoin (dilantin)
c. this is a mongolian spot. It is common in Asians and black, and no tx is necessary because it will fade with age.
J., is the 26 y.o Asian mother of Alysia, age 2 months. She is concerned about the large blue spot covering her infant's entire rt lower leg. J. tells you that Alysia was born with the spot. You tell her: a. when the infant reaches her adult height, the macule can be surgically removed b. she should take the infant immediately to a plastic surgeon because this is a rare cancerous lesion c. this is a mongolian spot. It is common in Asians and black, and no tx is necessary because it will fade with age. d. she should always keep the spot covered because sunlight will aggravate it
a. although measles, mumps, rubella, and influenza vaccines contain a minute amt of egg, most egg-allergic individuals can tolerate these vaccines without any problems
J., states that her son is allergic to eggs, and she heard that he should not receive the flu vaccine. How do you respond? a. although measles, mumps, rubella, and influenza vaccines contain a minute amt of egg, most egg-allergic individuals can tolerate these vaccines without any problems b. most of the allergic reactions are caused by the actual vaccinations, therefore, a skin test should be done fist c. you are right. we should not give this vaccination to your son d. he should not have a skin test done if he has this allergy because a serious cellulitis may occur at the testing site
c. Chronic bronchitis
James, age 78, presents to the clinic with respiratory symptoms. His pulmonary function tests (PFTs) are as follows: a normal total lung capacity, a decreased PaO2, and an increased PaCO2. On assessment, you auscultate coarse crackles and forced expiratory wheezes. What is your diagnosis? a. Asthma b. Emphysema c. Chronic bronchitis d. Influenza
b. decreases vitamin C absorption
Jane asks how smoking increases the risk for folic acid deficiency. You respond that smoking: a. causes small vessel disease and constricts all vessels that transport essential nutrients b. decreases vitamin C absorption c. affects the liver's ability to store folic acid d. causes nausea, thereby inhibiting the appetite and ingestion of food rich in folic acid
a. Inhaled corticosteroid
Jane is taking a long-acting beta agonist for her asthma. What additional medication should she be taking? a. Inhaled corticosteroid b. Leukotriene receptor antagonist c. Systemic corticosteroid d. Methyl xanthenes
d. Lea's shield
Jane wants to use some form of birth control, but because she is getting married next year,s he wants to be able to stop the birth control method after the wedding and have her fertility restored almost immediately. Which method do you recommend for her? a. birth control pills b. vaginal ring c. depot-medroxyprogesterone acetate (DMPA) d. Lea's shield
b. PID
Jane, age 27, is c/o of lower abdominal pain. After doing some lab studies, you find leukocytosis, an elevated erythrocyte sed rate and an elevated C-reactive protein level. Which is the most appropriate dx? a. ovarian cyst b. PID c. tubal pregnancy d. diverticulitis
d. in addition to its efficacy in the tx of osteoporosis in postmenopausal women, it is also useful for the prevention of osteoporosis
Jane, age 60 asks, you about taking alendronate (Fosamax). What do you tell her about using the medication? a. if you decide to take it, stick with a lower dose of 5mg because the side effects are much worse with a 10mg dose b. fosamax works better in younger women, so you should start this now rather than wait until you're 70 c. you should take a daily dose because the wkly dose is not as effective d. in addition to its efficacy in the tx of osteoporosis in postmenopausal women, it is also useful for the prevention of osteoporosis
ACE inhibitor (pg 432-433 table 10.1 indication)
Janelle has hypertension and previously had a stroke. Which hypertensive drug would you order for her?
stopping smoking
Janet has stress urinary incontinence and has been doing Kegel exercises with some success. What other simple action might help in relieving some of the problem?
c. you must observe these precautions forever
Janice had a modified mastectomy with radiation tx 10 years ago. She asks when she can have her blood pressure or needle sticks taken in the affected arm. How do you respond? a. if its been 10 yrs and you've had no problems,you can discontinue those precautions b. because you didn't have a radical mastectomy, you can do those things now c. you must observe these precautions forever d. as long as you do limb exercises and have established collateral drainage, you can discontinue these precautions
b. acute myelogenous leukemia (AML) pg 931/935
Janice is 42 y/o and was just dx with leukemia, she is c/o bone and joint pain. Which type of leukemia does she probably have? a. acute lymphocytic leukemia (ALL) b. acute myelogenous leukemia (AML) c. chronic myelogenous leukemia (CML) d. chronic lymphocytic leukemia (CLL)
c. immunotherapy
Janice is having biological therapy for her pancreatic cancer. What kind of tx is this? a. surgery b. radiation therapy c. immunotherapy d. chemotherapy
b. a compulsive eater does not induce vomiting
Janice, age 14, is markedly obese and has a poor self-image. How do you differentiate between compulsive eating and bulimia? a. bulimia results in irregular menstruation b. a compulsive eater does not induce vomiting c. a compulsive eater has tooth and gum erosion d. a compulsive eater does compulsive exercising
c. cellulitis pg 190
Janice, age 62, presents with a wide, diffuse area of erythematous skin on her left lower leg that is warm and tender to palpation. There is some edema involved. You suspect: a. necrotizing fasciitis b. Kaposi's sarcoma c. cellulitis d. a diabetic ulcer
c. C6
Jeff, age 12, injured his spinal cord by diving into a shallow lake. He is in a wheelchair but can self-transfer. He can use his shoulder and extend his wrist but has no finger control. At what level of the spinal cord was the damage? a. C4 b. C5 c. C6 d. C7
a. there are round hypopigmented macules on his back pg 169
Jeff, aged 22, has tinea versicolor. Which description is the most likely for this condition? a. there are round hypopigmented macules on his back. b. there are red papules on his face. c. there are crusted plaques in his groin area. d. there are white streaks on his neck.
b. do you have an associated vaginal discharge or irritation?
Jenn, a 25 y/o female pt, c/o dysuria. In taking a thorough hx to formulate a dx, it is most important to ask: a. do you have painful intercourse? b. do you have an associated vaginal discharge or irritation? c. do you also have a problem with defecation? d. do you have stress incontinence?
b. if you have surgery early, rather than continue with conservative therapy, your rate of pain relief and recovery will be faster
Jessica, a retired NURSE, is contemplating surgery for severe sciatica and asks you for your opinion. How do you respond? a. if you continue with conservative tx for a while, your pain will be relieved faster b. if you have surgery early, rather than continue with conservative therapy, your rate of pain relief and recovery will be faster c. the 1-yr outcomes related to pain relief and recovery are the same for both early surgery and conservative therapy d. what has your doctor told you?
"let's try it. It corrects iron deficiency faster, more safely, and better than oral iron"
Jessica, age 79, has been on dialysis for 6 years due to CKD. She has been on an oral iron preparation and now asks you if she should think about switching to the IV iron preparation. How do you respond?
b. classic migraine
Jessie, age 29, sees flashing lights 20 minutes before experiencing severe headaches. How would you describe her headache? a. migraine without aura b. classic migraine c. tension headache d. cluster headache
b. 6 months
Jill has just been given a dx of HIV infection and has a normal initial Pap test. When do the CDC guidelines state that she should have a repeat Pap smear? a. 3 months b. 6 months c. one year d. she should have a colposcopy every yr rather than an Pap smear
b. 30ml
Jill, who has had urinary incontinence for several months, wants to have an evaluation to determine if any therapy will be beneficial. After having an initial pelvic examination, she has a post-void residual catheterization. A residual volume of more than how many milliliters is abnormal? a. 10ml b. 30ml c. 50ml d. 100ml
b. panic attack
Jim, 45, is a postal worker, presents for the first time with a sudden onset of intense apprehension, fear, dyspnea, palpitations and a choking sensation. What is your initial dx? a. anxiety b. panic attack c. depression d. agoraphobia
d. polycythemia vera
Jim, a 66 y/o white male who is on diuretic tx, presents with an elevated hematocrit. He also has splenomegaly on exam, as well as subjective c/o of blurred vision, fatigue, headache, and tinnitus. You suspect: a. multiple myeloma b. Waldenstrom's macroglobulinemia c. dehydration related to the diuretics d. polycythemia vera
b. have you thought about committing suicide?
Jim, age, 45, has 2 small children. He states that his wife made him come to this appt because she thinks he has been impossible to live with lately. He admits to being stressed and depressed because he is working 2 jobs, and he says he sometimes takes his stress out on his family. About twice a week he c/o palpitations along with nervous energy. What is the most important question to ask him at this time? a. how is your wife handling stress? b. have you thought about committing suicide? c. do you and your wife spend time alone together? d. tell me more about what you think is causing this
His age - 10 points
John has cardiovascular disease (CVD), is 64 years old, has a total cholesterol of 280 mg/dL, has a systolic blood pressure (BP) of 158, and is being treated for hypertension. You are doing a Framingham Risk Assessment on him. Which assessment factor would give him the highest number of points on the scale?
c. immunize John with diphtheria, tetanus, and pertussis; Haemophilus influenzae type b (HIB); hepatitis B (HBV); and poliomyelitis vaccines
John is a 6 month old with newly dx sickle cell disease. His mother brings him to the clinic for a well-baby visit. Which of the following should you do on this visit? a. tell the parents that John will not be immunized because of his dx b. tell the parents that John should not go to day care c. immunize John with diphtheria, tetanus, and pertussis; Haemophilus influenzae type b (HIB); hepatitis B (HBV); and poliomyelitis vaccines d. immunize John with measles, mumps, and rubella, HIB; and HBV vaccines only
c. syncope
Jonas, age 62, experienced a temporary loss of consciousness that was associated with an increased rate of respiration, tachycardia, pallor, perspiration, and coolness of the skin. How would you describe this? a. lethargy b. delirium c. syncope d. a fugue state
tylenol 650 tid daily
Joyce is being tx conservatively for her low back pain with NSAIDs, muscle relaxants, and physical therapy. She has recently been dx with chronic kidney disease. What could she replace the NSAIDs with?
c.The cancer is less than 2 cm in size and has not spread to the lymph nodes or other parts of the body (pg 385)
Julie has breast cancer that has been staged as T1, N0, M0. What might this mean? a.The tumor size cannot be evaluated, the cancer has not spread to the lymph nodes, and the distant spread cannot be evaluated. b.The cancer is in situ, it is spreading into the lymph nodes, but the spread otherwise cannot be evaluated. c.The cancer is less than 2 cm in size and has not spread to the lymph nodes or other parts of the body. d.The cancer is about 5 cm in size, nearby lymph nodes cannot be evaluated, and there is no evidence of distant spreading.
c. of thrombocytopenia, in which the life expectancy may be only 2 yrs
Julie's brother has chronic lymphatic leukemia. She overheard that he was in state IV and asks what this means. According to the Rai classification system, stage IV is a stage: a. at which the lymphocytes are greater than 10,000 mm3 b. with an absolute lymphocytosis, in which the clt may live 7-10 yrs or more c. of thrombocytopenia, in which the life expectancy may be only 2 yrs d. of anemia
d. confront Julie with the fact that you suspect an eating disorder
Julie, age 15, is 5 ft tall and weights 85 lbs. You suspect anorexia and know that the best initial approach is to: a. discuss proper nutrition b. tell Julie what she should weight for her ht and suggest a balanced diet c. speak to her parents before going any further d. confront Julie with the fact that you suspect an eating disorder
b. clopidogrel (plavax)
Julie, age 58, has had several TIAs. After a diagnostic evaluation, what medication would you start her on? a. ticlopidine (ticlid) b. clopidogrel (plavax) c. warfarin (coumadin) d. nitroglycerin (nitro-dur)
c. cholesterol-lowering agents
June, age 79, comes to your office with a recent onset of depression. She is taking several medications. Which medication is safe for her to take because depression is not one of the side effects? a. antiparkinsonian agents b. hormones c. cholesterol-lowering agents d. antihypertensive agents
c. vitamin D increases intestinal absorption of the dietary calcium and mobilizes calcium from the bone
K. who is postmenopausal, is taking 1,500mg of calcium but does not understand why she also needs to take vitamin D. you tell her that: a. a deficiency of vitamin D results in an inadequate mineralization of bone matrix b. all vitamins need to be supplemented c. vitamin D increases intestinal absorption of the dietary calcium and mobilizes calcium from the bone d. vitamin D binds with calcium to allow active transport into the cells
c. I will exercise in the evening to tire myself out before bed
Kare, age 52, has 4 children and a very stressful job. (might be a nurse):). After you perform her physical which was normal, she tells you she has insomnia. You make several suggestions for lifestyle changes that might assist in promoting helpful sleep. You know she misunderstands when she states which of the following: a. I'll wind down before bedtime by taking a warm bath or by reading for 10 minutes b. I will try some valerian extract from the health food store c. I will exercise in the evening to tire myself out before bed d. I won't read or watch tv while in bed
c. diamox pg 81
Karen has peripheral vestibular disease causing dizziness and vertigo. Which of the following medications will help to decrease the edema in the labyrinth of the ear? a. meclizine b. diphenhydramine c. diamox d. promethazine
stress incontinence
Kegel exercises are used for a dx of
a. develops when loss of bone matrix (resorption) occurs more rapidly than new bone growth (deposition)
L. age 70 was told that she has osteoporosis. when she asks you what this is, you respond that osteoporosis: a. develops when loss of bone matrix (resorption) occurs more rapidly than new bone growth (deposition) b. is a degenerative joint disease characterized by degeneration and loss of articular cartilage in synovial joints c. is a chronic, systemic inflammatory disorder characterized by persistent synovitis of multiple joints d. is a metabolic bone disorder characterized by inadequate mineralization of bone matrix
a. let's schedule an exam now because women no longer need a rectal exam with routine pelvic exams
L., age 38 states that she has not had a pelvic exam in 5 yrs because she does not like having the digital rectal exam. How do you respond? a. let's schedule an exam now because women no longer need a rectal exam with routine pelvic exams b. ok, we'll do a pelvic, and I'll just put refused on the chart to cover my liability c. we really need to don one because the rectal exam has been shown to pick up many abnormalities such as rectal polyps d. I'll try to be quick with the rectal exam and get it over with
c. I'll go ahead and order the Yuzpe regimen
L., age 43 comes to your office in tears, stating that last night she had unprotected sex and forgot to take her birth control pill. she wants to know about the morning -after pill. You tell her: a. if your period does not start at the schedule time, come back to see me b. I'll go ahead and order the estrogen-only postcoital contraception pill c. I'll go ahead and order the Yuzpe regimen d. I'll refer you to a gynecologist
b. determine the need for 0.5ml 1:1,000 epinephrine SQ
L., age 50 is c/o an itchy rash that occurred about a half hr after putting on his leather jacket. He recalls a slightly similar rash last yr when he wore his jacket. The annular lesions are on his neck and both arms. They are erythematous, sharply circumscribed, and both flat and elevated. His voice seems a little raspy, although he states that his breathing is normal. What is your first action? a. order a short course of systemic corticosteroids b. determine the need for 0.5ml 1:1,000 epinephrine SQ c. start daily antihistamines d. tell L. to get rid of his leather jacket
begin a course of glucocorticoids
L., age 52, who has just been given a dx of sarcoidoxis, has joint sxs including arthralgias and arthritis. your next plan of action would be to: a. order a bone scan b. obtain a tissue biopsy c. begin a course of glucocorticoids d. obtain an electrocardiogram
moderate to severe, use before sleep to prevent nocturnal attacks (pg 344)
LABA medications are appropriate for use in what level of asthma?
without use of inhaled corticosteroids (pg 344)
LABA medications should never be used
12 hrs
LMWH will exert their effect for at least how long?
b. maintains an acid pH range
Lactobacilli in the vagina: a. decreases glycogen metabolism b. maintains an acid pH range c. increases the development of WBCs d. maintains an alkaline pH range
b. begin a planned exercise program to strengthen back muscles
M. a golf pro, has had chronic back pain for many yrs. his work-up reveals that is is not the result of a degenerative disk problem. his back goes out about twice per yr, and he is out of work for about a week each time. which of the following should you advise him to do? a. consider changing careers to a less physical job b. begin a planned exercise program to strengthen back muscles c. make an appt with a neurosurgeon for a surgical consultation d. start on a daily low dose narcotic to take away the pain
a. varicocele
M. age 16 states that he feels like he has a bag of worms on his left scrotum. even before examining him, what do you suspect? a. varicocele b. hydrocele c. cystic nodules d. spermatocele
a. musculoskeletal pain
M. age 34 presents with sxs resembling both fibromyalgia and chronic fatigue syndrome, which have many similarities. which of the following is more characteristic of fibromyalgia than of chronic fatigue syndrome? a. musculoskeletal pain b. difficulty sleeping c. depression d. fatigue
b. L4/L5 (L5 root involvement)
M. age 52 is a postal worker who drive a truck every day. he presents with lower back pain and has decreased sensation to a pinprick int he lateral leg and web of the great toe. this indicates discogenic disease int he dermatomal pattern of which area? a. L3/L4 (L4 root involvement) b. L4/L5 (L5 root involvement) c. L5/S1 (S1 root involvement) d. none of the above
d. pack the fingers in a saline-soaked dressing and seal in a plastic bag
M. age 52, is a chef who just severed two of his fingers with a meat cutter. you would recommend that he: a. wrap the severed fingers tightly in a dry towel for transport to the ED with him b. leave the severed fingers at the scene because fingers cannot be reattached c. immediately freeze the severed fingers for reattachment in the near future d. pack the fingers in a saline-soaked dressing and seal in a plastic bag
d. reactive arthritis
M. age 58 presents with urethritis, conjunctivitis, and asymmetric joint stiffness, primarily in the knees, ankles, and feet. which condition do you suspect? a. syphilis b. gonorrhea c. HIV d. reactive arthritis
c. no more than 3-4 times a yr int he same joint
M. age 72, has severe osteoarthritis of her rt knee. she obtains much relief from corticosteroid injection. when she asks you how often she can have them, how do you respond? a. only once a yr in the same joint b. no more than twice a yr in the same joint c. no more than 3-4 times a yr int he same joint d. no more than 5-6 times a yr in the same joint
c. homosexuality
M. c/o a UTI. which of the following is a risk factor for a UTI in men? a hx of circumcision b. hx of testicular torsion c. homosexuality d. presence of a left inguinal hernia
c. if it hasn't descended by now, it probably won't. he needs to have surgery by age 6
M. had an undescended testicle at birth, and at age 2 it remains in the inguinal region. his mother is afraid of surgery and asks for your advice. how do you respond? a. in many children,t he testicle descends close to the 6th birthday b. even with only one normal testicle, he will have normal development c. if it hasn't descended by now, it probably won't. he needs to have surgery by age 6 d. don't worry; it can remain in that position forever with no problems
c. there is no evidence to support combination txment
M. has been taking finasteride (Proscar) and states that he has had dramatic relief. he previously took terazosin (HYtrin), which also helped, and he asks you about taking that again. you tell him: a. yes, let's try the combination txment because two are better than one b. no, they are absolutely contraindicated together c. there is no evidence to support combination txment d. when sxs get so bad that you need two different meds, it is time for surgery
c. diabetes mellitus
M. is c/o ED. he also has a condition that has reduced arterial blood flow to his penis. the most common cause of this condition is: a. epilepsy b. multiple sclerosis c. diabetes mellitus d. parkinson's disease
a. nocturnal penile tumescence and rigidity test
M. is in a new relationship and is not sure whether his ED is caused by stress about his performance or is organic. what simple test could you suggest to determine if he has the ability to have an erection? a. nocturnal penile tumescence and rigidity test b. penile duplex ultrasonography c. intraspongiosum injection d. serum PSA
b. doxorubicin (adriamycin)
M. is on a chemotherapeutic antibiotic for a musculoskeletal neoplasm. which drug do you think he is taking? a. cyclophosphamide (cytoxan) b. doxorubicin (adriamycin) c. methotrexate (rheumatrex) d. cisplatin (platinol)
c. cause atypical pneumonia
M. pneumoniae and C. pneumoniae are respiratory pathogens that: a. are spread via direct contact b. associated with immunosuppression c. cause atypical pneumonia d. transmitted via direct contact
aerobic and are attracted to the apical segments of the upper lung lobes where 02 is high (pg 369)
M. tubercolosis organisms are
person to person by droplet nuclei that are aerosolized by coughing, sneezing, or speaking (pg 368)
M. tuberculosis organsms are typically transmitted via
b. scaphoid fx
M., a 55 y.o female, presents to you for evaluation of left hand and wrist pain a d swelling after a slip and fall on the ice yesterday. on exam, you note tenderness at her anatomical snuffbox. you know this probably indicates: a. ulnar styloid fx b. scaphoid fx c. hamate fx d. radial head fx
b. urethral stricture
M., a 72 y/o unmarried, sexually active white man, presents to your clinic with c/o hesitancy, urgency, and occasional uncontrolled dribbling. although you suspect benign prostatic hypertrophy, what else should your diff dx include: a. antihistamine use b. urethral stricture c. detrusor hyperreflexia d. renal calculi
c. systemic lupus erythematosus
M., age 32 comes into the clinic. She has painful joints and a distinctive rash in a butterfly distribution on her face. The rash has red papules and plaques with a fine scale. What do you suspect? a. lymphocytoma cutis b. relapsing polychondritis c. systemic lupus erythematosus d. an allergic rx
c. to continue
M., age 40 has been given a dx of rheumatoid arthritis. She asks you whether she should continue taking her birth control pills. You tell her: a. to check with her rheumatologist b. to stop c. to continue d. the dose will have to be altered
a. medroxyprogestone
M., age 42, is having dysfunctional uterine bleeding (DUB) and cannot take oral contraceptives due to a hx of a blood clot (emboli). Management includes which medication? a. medroxyprogestone b. ethinyl estradiol c. conjugated estrogen d. piroxicam
c.its's an autogenous procedure that uses skin fromt he rectus abdominis muscle to fashion a breast
M., age 52, states that she is going to have a TRAM procedure after her breast surgery. However, she was in shock from the dx when the surgeon explained the procedure. She ask you to explain it. How do you respond? a. it's when a breast implant is inserted under the pectoris muscle b. its' an autogenous procedure that uses skin from the latissimus dorsi muscle to fashion a breast c.its's an autogenous procedure that uses skin fromt he rectus abdominis muscle to fashion a breast d. it's a breastimplant that is done after the mastectomy scar heals
c. your mother does not need a bath every day
M., asks what she can do for S., her 90 y/o mother, who has extremely dry skin. You respond: a. after bathing every day, use a generous supply of moisturizers b. use a special moisturizing soap every day c. your mother does not need a bath every day d. increase your mother's intake of fluids
b. beefy red
M., just came from visiting her husband, Sam, age 82, who recently had an ileostomy resulting in a stoma. She did not think that Sam's stoma looked right. You tell her that the color of the stoma should be: a. pale pink b. beefy red c. dark red or purple d. flesh colored
multi-drug resist TB - TB resistant to at least INH and rifampin (pg 376)
MDR-TB is
c. A deficiency of folic acid.
Macrocytic normochromic anemias are caused by: a. A nutritional deficiency of iron. b. An infection or tumor. c. A deficiency of folic acid. d. Acute blood loss.
No benefit to antihistamines or decongestants demonstrated unless allergy If eustachian tube dysfunction caused by allergy symptoms: Nasal steroids, antihistamines, decongestants If effusion persists >3 months, single course of amoxicillin/clavulanate Refer if symptoms persist Avoid activities that increase pressure to inner ear; avoid exposure to tobacco
Management of OME (ppt week 2)
c. we will stop at the store and buy plastic eating utensils
Mandy's 16 y/o daughter has hepatitis A. Which of the following statements made by Mandy indicates that she understand the teaching you have just completed? a. I guess she needs to be hospitalized until she is recovered b. we will keep her at home with strict isolation precautions c. we will stop at the store and buy plastic eating utensils d. we will stop at the drugstore and pick up prescription medications immediately
c. abdominal pain, pallor, and tachycardia
Mandy, age 6, recently was dc fromt he hospital after a sickle cell crisis. You are teaching her parents to be alert to the manifestations of splenic sequestration and tell them to be alert to: a. vomiting and diarrhea b. decreased mental acuity c. abdominal pain, pallor, and tachycardia d. abdominal pain and vomiting
d. hallucinations
Morrison exhibits extrapyramidal side effects of antipsychotic medications. Which of the following sxs would lead you to look for another dx? a. akathisia b. dystonia c. Parkinsonism d. hallucinations
c. Add a dosage of long-acting insulin at bedtime to the regimen. quiz 624
Morton has Type 2 diabetes. His treatment, which includes diet, exercise, and oral antidiabetic agents, is insufficient to achieve acceptable glycemic control. Your next course of action is to: a. Discontinue the oral antidiabetic agents and start insulin therapy. b. Suggest treatment using an insulin pump. c. Add a dosage of long-acting insulin at bedtime to the regimen. d. Increase the dosage of the oral antidiabetic agents.
Gram negative bacteria
Most NOSOCOMIAL pneumonia are caused by
b. reduces the incidence of endometrial hyperplasia and cancer
Mrs. H. inquires about why she needs progestereone in addition to her estrogen for hormone replacement. Women who have an intact uterus need to add progesterone to their prescribed estrogen because progestin: a. assists in relieving the typical hot flashes ofmenopause b. reduces the incidence of endometrial hyperplasia and cancer c. decreases the risk of osteoporosis d. controls mood swings
b. rheumatoid arthritis
Mrs. Hubbard, age 45, is complaining of generalized morning stiffness especially in both her wrists and hands. It is much worse in the morning and lasts for a few hours. She also complains of fatigue and generalized body aches that have been present for the past few months. Which of the following is most likely? a. Osteoarthritis b. Rheumatoid arthritis c. Gout d. Osteoporosis
b. Rheumatoid arthritis 624 quiz
Mrs. Hubbard, age 45, is complaining of generalized morning stiffness especially in both her wrists and hands. It is much worse in the morning and lasts for a few hours. She also complains of fatigue and generalized body aches that have been present for the past few months. Which of the following is most likely? a. Osteoarthritis b. Rheumatoid arthritis c. Gout d. Osteoporosis.
c. cytomegalovirus infection
Mrs. James c/o unilateral blurry vision and partial blindness in the left eye. On PE, you find decreased peripheral vision on her left side. Funduscopic exam reveals cottom-wool spots. Your most likely dx is: a. cryptococcosis b. toxoplasmosis c. cytomegalovirus infection d. herpes simplex virus infection
a. allow a fingertip between it and the pubic arch
Mrs. L. would like to be fitted for a diaphragm. She has been on numerous hormones int he past and does not like the side effects. It is important to remember that when properly fitting a pt for a diaphragm, it should: a. allow a fingertip between it and the pubic arch b. be small enough to allow for vaginal expansion c. lie snugly over the pubic arch and under the cervix d. provide firm tension against the vaginal walls
d. lysosomal degradation
Mrs. M has rheumatoid arthritis. on reviewing an x-ray of her hip, you notice that there is a marked absence of articular cartilage. what mechanism is responsible for this? a. antigen-antibody formation b. lymphocyte response c. immune complex formation d. lysosomal degradation
d. I would be happy to see your daughter and complete her gyn exam; however, she does not need a Pap smear until the age of 21
Mrs. W. would like to schedule an appt to bring her 18 y/o daughter in to see you for her first gyn exam and Pap smear. the most appropriate reply would include: a. I would be happy to see your daughter and complete her gyn exam and Pap smear b. your daughter does not need a gyn exam and Pap at this time c. Your daughter only needs a gyn exam and Pap smear when she becomes sexually active d. I would be happy to see your daughter and complete her gyn exam; however, she does not need a Pap smear until the age of 21
d. a disease with extrapulmonary manifestations
Mycoplasma pneumonia is: a. a dx of exclusion b. a typical respiratory pathogen c. only identifiable on chest x-ray d. a disease with extrapulmonary manifestations
a. tinea cruris
Mycoses commonly affect the skin in the groin. which fungus commonly affects the groin? a. tinea cruris b. candida albicans c. trichomonas d. trichophyton
b. candida albicans
Mycoses commonly affect the skin in the groin. which fungus commonly affects the scrotum? a. tinea cruris b. candida albicans c. trichomonas d. trichophyton
ranging from N0 to N3
N indicates the nodal involvement
myasthenia gravis
Neostigmine is usually used for
Grade N No Recommendation for or against ("There is insufficient evidence or evidence is unclear or conflicting.")
Net benefit is unclear. Balance of benefits and harms cannot be determined because of no evidence, insufficient evidence, unclear evidence, or conflicting evidence, and the committee thought no recommendation should be made. Further research is recommended in this area
Grade E Expert Opinion ("There is insufficient evidence or evidence is unclear or conflicting, but this is what the committee recommends.")
Net benefit is unclear. Balance of benefits and harms cannot be determined because of no evidence, insufficient evidence, unclear evidence, or conflicting evidence, but the committee thought it was important to provide clinical guidance and make a recommendation. Further research is recommended in this area.
B-type natriuretic peptide (BNP) (pg 464, >500 pg/ml)
What is abundant in the heart and rapidly rises in the bloodstream in the presence of heart failure which makes it a good diagnostic test?
b. urge incontinence
R. a health 68 y/o man, comes to see you with a c/o sudden episodes of an urgent need to void. he has had several episodes of moderate amts of unintentional urine loss during these times. other than these episodes, he is voiding in amts normal for him, with no leakage when he coughs or sneezes, your initial dx is which type of incontinence? a. stress incontinence b. urge incontinence c. overflow incontinence d. mixed incontinence
glomerular bleeding that is strongly suggestive of glomerulonephritis
RBC casts indicate
c. basketball
RM has a 15 y/o son who wants to play sports; however, she is very leery because she has heard of so many accidents. which one of the following sports does the American Academy of Pediatrics list as a limited contact/impact sport? a. field hockey b. soccer c. basketball d. lacross
c. cryptosporidiosis
Raina has HIV infection and is having a problem with massive diarrhea. You suspect the cause is: a. cryptococcosis b. toxoplasmosis c. cryptosporidiosis d. cytomegalovirus
the cause is unknown, but suspect it is due to an immunologic mechanism pg 146
Ralph presents with alopecia areata with well-circumscribed patches of hair loss on the crown of his head. How do you respond when he asks you the cause?
b. His collar size is 17 inches.
Ralph, aged 63, has obstructive sleep apnea. What do you think is one of his contributing factors? a. He is a recovering alcoholic of 6 years. b. His collar size is 17 inches. c. He is the only person in his family who has this. d. He is extremely thin.
b. hydrocortisone base 2.5% pg 228
Ralphie has atopic dermatitis. You are prescribing a low-dose topical corticosteroid for him. Which would be a good choice? a. betamethasone dipropionate 0.05% b. hydrocortisone base 2.5% c. halcinonide 0.1% d. desonide 0.05%
d. gonorrhea
Reiter's syndrome is a complication of: a. bacterial vaginosis b. syphilis c. chlamydia d. gonorrhea
c. "You may have difficulty salvaging iron from old red blood cells for reuse." quiz 624
Rena, age 27, had her spleen removed after a MVA. You are seeing her in the office for the first time since her discharge from the hospital. She asks you how her surgery will affect her in the future. How do you respond? a. "Your lymphatic system may have difficulty transporting lymph fluid to the blood vessels." b. "Your red blood cell production will be slowed." c. "You may have difficulty salvaging iron from old red blood cells for reuse." d. "You'll have difficulty storing the nutritional agents needed to make red blood cells."
The clinician is assessing a patient complaining of hearing loss. The clinician places a tuning fork over the patient's mastoid process, and when the sound fades away, the fork is placed without restriking it over the external auditory meatus. The patient is asked to let the clinician know when the sound fades away. This is an example of which type of test?
Rinne test
watermelon
Rita, age 16, has a 3 day hx of red urine. You want her to go home for a few days and then return for some lab work. In teaching her what to avoid, you know she has misunderstood the directions when she tells you she should ingest any
d.Continue to ask him at every visit if he is ready yet, tell him you ask because you care about his health and document this on a regular basis as a new vital sign (pg 409).
Robert, is 32-year old , has a 15-pack year hx of smoking and continues to smoke heavily. He gets irate during every visit when you try to talk to him about quitting. What should you do? a.Just hand him literature about smoking cessation at every visit. b.Wait until he is ready to talk to you about quitting. c.Just document in the record that he is not ready yet. d.Continue to ask him at every visit if he is ready yet.
c. try neutral position wrist splinting and order an oral NSAID
S. a computer programmer, has just been given a new dx of carpal tunnel syndrome. your next step is to: a. refer her to a hand surgeon b. take a more complete hx c. try neutral position wrist splinting and order an oral NSAID d. order a nerve conduction study such as an electromyography (EMG)
c. hematuria
S. age 32, fell off a roof while shingling it. he is c/o pain in his left hip and leg area. other than an x-ray, whould would make you suspect a fx pelvis? a. a clicking sensation when moving the hips b. a positive pelvic tilt test c. hematuria d. absence of distal reflexes
a. fever, tachycardia, rapid respirations, and neurological manifestation
S. age 34, fx his femur when his horse tripped over a jump. with this type of injury you know that S. is at risk for fat emboli. early assessment findings for this complication include: a. fever, tachycardia, rapid respirations, and neurological manifestation b. neurological manifestations, temperature elevation, bradycardia and pallor c. hostility; combativeness; substernal pain; and weak thready pulse d. lethargy, hypothermia, paresthesia, and absent peripheral pulses
b. dietary counseling
S. age 48 has asymptomatic hyperuricemia. what is your inital tx? a. NSAIDs b. dietary counseling c. colchicine d. allopurinol
d. refer to a neurosurgeon
S. age 49 presents with loss of anal spincter tone, impaired micturition, incontinence, and progressive loss of strength in the legs. you suspect cauda equina syndrome. what is your next action? a. order physical therapy b. order a lumbar/sacral x-ray c. order extensive lab work d. refer to a neurosurgeon
c. bisphosphonates
S. age 50, presents with Paget's disease that has been stable for several yrs. recently, his serum alkaline phosphatase level has been steadily rising. you determine that it is time to start him on: a. NSAIDs b. corticosteroids c. bisphosphonates d. calcitonin
b. chronic bacterial prostatitis
S. age 72 presents to the clinic with c/o a weak urine stream, hesitancy, and painful ejaculation. on DRE, you note that his prostate is boggy. the most common cause of his sxs is: a. acute bacterial prostatitis b. chronic bacterial prostatitis c. chronic nonbacterial prostatitis with chronic pelvic pain syndrome d. noninflammatory prostatitis
c. chocolate
S. has been dx with fibrocystic breast disease. Which of the following may exacerbate the condition? a. daily dose of aspirin b. spicy foods c. chocolate d. wearing tight bras
young children, older adults and person with chronic, debilitating pathology (pg 359)
S. pneumoniae commonly affects
pneumococcal pneumonia (pg 359)
S. pneumoniae is also commonly known as
c. staging will determine the extent of the spread of the cancer and txment options
S. states that he was recently given a dx of prostate cancer and that he has to return to the urologist for staging. he doesn't understand why because he says, cancer is still cancer., i just want to get rid of it. you tell him: a. staging determines the type of tests required b. you have time to decide on txment until the cancer gets to the last stage c. staging will determine the extent of the spread of the cancer and txment options d. you already know you have prostate cancer; you don't need another test unless you want to know how long you've had it
d. infarction and necrosis of the affected tissue can happen with repeated frostbite
S. states that her fiance has been frostbitten on the nose while skiing and is fearful that is will happen again. What do you tell her? a. don't worry, as long as he gets medical help in the first few hrs after being frostbitten again, he will recover b. once frostbitten, he should not go out skiing again c. if it should happen again, massage the nose with a dry hand d. infarction and necrosis of the affected tissue can happen with repeated frostbite
b. reserpine
S. who takes many different meds, is c/o ED. you know that several meds could be the cause of his problem. which of his meds is most likely the culprit? a. lasix b. reserpine c. prilosec d. isosorbide
a. craniotabes
S., a new mother, states that when she pushes her index finger on one of the baby's skull bones, it presses in and then returns to normal when she removes her finger. She is concerned about this. You tell her that is is common and is called? a. craniotabes b. molding c. caput succedaneum d. cephalhematoma
c. as long as you can protect the remaining testicle, go for it
S., age 15, has only one testicle. when he asks you if he can play on the soccer team at school, how do you respond? a. no, you'd be taking too much of a chance of injuring your remaining testicle b. you can play any noncontact sport; however, soccer is too strenuous c. as long as you can protect the remaining testicle, go for it d. it should have no bearing on any activity
b. tinea versicolor
S., age 18, presents with a pruritic rash on his upper trunk and shoulders. You observe flat to slightly elevated brown papules and plaques that scale when they are rubbed. You also note areas of hypopigmentation. What is you initial dx? a. lentigo syndrome b. tinea versicolor c. localized brown macules d. ochronosis
a. compartment syndrome
S., age 26 has a cast on his rt arm because of an in-line skating accident. twelve hrs after the cast was applied, he c/o severe pain ever though he recently took his pain meds. his fingers are pink, yet he states that they are tingling and feel slightly numb. what do you suspect? a. compartment syndrome b. phlebitis c. osteomyelitis d. muscle contraction
c. staphylococcus aureus
S., age 29 has a carbuncle on his neck. After an I&D, an antibiotic is ordered. What is the most common organism involved? a. streptococcus b. Moraxella catarrhalis c. staphylococcus aureus d. Klebsiella
b. increase her intake of complex carbohydrates
S., age 29, c/o premenstrual syndrome. She state she was told that changing her diet might help in managing some of the sxs. What changes in her diet do you recommend? a. increase her intake of protein b. increase her intake of complex carbohydrates c. increase her intake of sale and salty foods d. decrease her intake of fatty foods
b. skin tag
S., age 52 has an acrochordon on her neck. She refers to it as a: a. nevus b. skin tag c. lipoma d. wart
b. keep the area as dry as possible
S., age 6 months, has a candida infection in the diaper area. What do you suggest to the mother? a. use rubber or plastic pants to contain the infection and prevent it from getting to the thighs b. keep the area as dry as possible c. use baby power with cornstarch d. keep S. away from other babies until the infection is cleared up
b. use tepid water and a mild cleansing cream
S., age 69 is c/o dry skin. What do you advise her to do? a. bathe every day b. use tepid water and a mild cleansing cream c. use a dehumidifier d. decrease the oral intake of fluids
a. systemic corticosteroids
S., age 72, has just been dx with temporal arteritis. What do you prescribe? a. systemic corticosteroids b. topical corticosteroids c. antibiotics d. antifungal preparations
a. a fungal infection, usually Candida albicans
S., age 82, comes to your office with a fairly new colostomy. Around the stoma he has a papular rash with satellite lesions. What does this indicate? a. a fungal infection, usually Candida albicans b. an allergic rx to the appliance c. a normal rx to fecal drainage d. a fluid volume deficit
a. open comedones
S., ask you about the blackheads on her face. You tell her these are referred to as: a. open comedones b. closed comedones c. papules d. pustules
c. stage III
S., brings in her husband, N., age 72 who is in a wheelchair. On his sacral area he has a deep crater with full-thickness skin loss involving necrosis of subcutaneous tissue that extends down to the underlying fascia. Which pressure ulcer stage is this? a. stage I b. stage II c. stage III d. stage IV
d. don't let her share hats, combs, or brushes with anyone
S., has a 7 y/o daughter who has had 2 recent infestations of lice. She asks you what she can do to prevent this. You respond: a. after 2 days of no head lice, her bedding is lice free b. boys are more susceptible, so watch out for her brother also c. after several infestation, she is now immune and is no longer susceptible d. don't let her share hats, combs, or brushes with anyone
a. bulla
S., has a blister filled with clear fluid on her arm. It is the result of contact with a hot iron. How do you document this? a. bulla b. wheal c. cyst d. pustule
d. douching may increase the risk of ectopic pregnancy
S., states that she has heard that douching effectively washes out the sperm after intercourse and that she has been using this as a method of birth control. Which of the following statements about douching is true? a. douching prevents sperm from entering the uterus b. douching should be used at least once a month after menses if not used after intercourse c. douching is a reliable contraceptive d. douching may increase the risk of ectopic pregnancy
heart failure
S3 is commonly noted in
poorly controlled hypertension
S4 is commonly noted in
acute asthma attacks may be used prophylactically before exercising increased need indicates need to change regime (pg 344)
SABA's are indicated for
first line of tx for Stage I mild COPD (pg 354)
SABAs are the
MI -STEMI is a result of a
ST-segment elevation - caused by an occlusive thrombus that leads to complete transmural MI - full thickness pg 451
a. latex condoms
Sally has HIV infection and ask which method of birth control, other than abstinence, would be best for her. You suggest a. latex condoms b. the spermicide nonoxynol-9 c. an IUD d. an oral contraceptive
a. give azithromycin (zithromax) 1g PO now
Sally has a dx of a chlamydia vaginal infection. You believe that is is questionable whether she will fill the prescription that you write or take it for 7 days as order. What would you do? a. give azithromycin (zithromax) 1g PO now b. emphasize the importance of the drug and tell her the consequences of not taking it. c. send out the public health nurse to follow up on whether she takes the drug for 7 days d. assume that Sally is an adult and will follow your instructions
a. this cord blood sample is identical to the genetic profile of the infant
Sally is 9 months pregnant and asks you about using umbilical cord blood after birth for newborn DNA identification. What do you tell you? a. this cord blood sample is identical to the genetic profile of the infant b. fingerprinting and footprinting that have been done in the past work perfectly c. a better method of obtaining newborn DNA is with a buccal swab d. the traditional method of a newborn heel stick for a DNA sample is tried and true
d. drink 4-6 glasses of water daily - should drink 8-10 glasses of water daily
Sally, age 24, has frequent UTIs. You are teaching her about measures to prevent them. You know she has misunderstood your directions when she tells you that she will: a. void after intercourse b. avoid bubble baths c. wear underwear with pantyhose d. drink 4-6 glasses of water daily
b. Guillain-Barre syndrome
Sally, age 52, presents with a rapidly progressive weakness of her legs that is moving up the trunk. She also has absent reflexes and no sensory change. What do you suspect? a. peripheral neuropathy b. Guillain-Barre syndrome c. myasthenia gravis d. radiculopathy
a. increase the estrogen content
Sally, age, 21 is taking an oral contraceptive. She c/o acne. How should you adjust the estrogen in the oral contraceptive? a. increase the estrogen content b. decrease the estrogen content c. delete the estrogen content d. no adjustment should be made to the estrogen content
d. both b and c pg 9315/935
Sam is 65 y/o and was just dx with leukemia, he is c/o fatigue, night sweats, and low grade fever. He also has nausea and occasional vomiting. Which type of leukemia does he probably have? a. acute lymphocytic leukemia (ALL) b. chronic myelogenous leukemia (CML) c. chronic lymphocytic leukemia (CLL) d. both b and c
b. endoscopic retrograde cholangiopancreatography
Sam is being worked up for pancreatic cancer. He states that the doctor wants to put a "scope" in and inject dye into his ducts. He wants to know more about this. What procedure is he referring to? a. percutaneous transhepatic cholangiography b. endoscopic retrograde cholangiopancreatography c. angiography d. upper GI series
b. anosmia
Sam, age 29, has lost his sense of smell. You would document this as: a. hyposmia b. anosmia c. ageusia d. agnosia
d. bone marrow smear
Sam, age 4, is brought into the office by his mother. His sxs are pallor, fatigue, bleeding, fever, bone pain, adenopathy, arthralgias, and hepatosplenomegaly. You refer him to a specialist. Which of the following tests do you expect the specialist to perform to confirm a dx? a. an enzyme-linked immunosorbent assay b. monospot test c. a prothrombin time, partial thromboplastin time, bleeding time, complete blood count, and peripheral smear d. bone marrow smear
Have Sam collect one urine specimen on first arising and then another in two hours
Sam, age 42, has had persistent proteinuria on the previous two office visits. Which action is warranted next?
b. brain tumor
Sam, age 5, is receiving radiation tx for his acute lymphocytic leukemia. He is at increased risk of developing which type of cancer as a secondary malignancy when he becomes an adult? a. chronic lymphocytic leukemia b. brain tumor c. liver cancer d. esophageal cancer
because your kidneys are not functioning well, your insulin is not being metabolized and excreted as it should, so you need less of it"
Sam, age 67 is a diabetic with worsening renal function. He has frequent hypoglycemic episodes, which he believes means that his diabetes is getting "better". How do you respond?
c. foraminotomy
Sandra has a ruptured intervertebral disk and is not responding to conservative management. She is requesting surgery for relief of her pain. She is going to have an enlargement of the opening between the disk and the facet joint to remove the bony overgrowth compressing the nerve. This describes which surgical procedure? a. laminectomy b. diskectomy c. foraminotomy d. chemonucleolysis
c. 1 hr before eating or between meals
Sandra, age 19 is pregnant. She is c/o breathlessness, tiredness, weakness, and is pale. After dxing anemia, you order medication and tell her to take it: a. only with meals because it can be irritation to the stomach b. in the morning if she experiences morning sickness c. 1 hr before eating or between meals d. at bedtime
b. 5,000 to 8,000 mm3
Sandy is being given platelets because of acture leukemia. One pack of platelets should raise her count by how much? a. 2, 000 to 4,000 mm3 b. 5,000 to 8,000 mm3 c. 9,000 to 12,000 mm3 d. about 15, 000 mm3
a. aspirin or aspirin compounds
Sara comes today with numerous petechiae on her arms. You know that she is not taking warfarin (coumadin). What other drugs do you ask her about? a. aspirin or aspirin compounds b. antihypertensive agents c. oral contraceptives d. anticonvulsants
respiratory acidosis pH is normal PaCO2 is acidic (respiratory) HCO3 is normal (metabolic)
Sara, age 82, was unable to breath for several minutes after choking on a piece of meat. A successful Heimlich maneuver was done and her ABCs now are pH 7.39 PaCO2 48 Pa02 92 HCO3 24 what do you suspect?
a. Kernig's sign
Sarah comes into the office with a severe headache, fever, delirium, nausea, vomiting, and a stiff neck. Which physical maneuver will you perform to help determine a dx? a. Kernig's sign b. Hoover's sign c. Hoffman's sign d. Babinski's reflex
atherosclerosis, collagen vascular disease, and select autoimmune disease such as scleroderma
Secondary Raynaud phenomenon is seen in the presence of an underlying condition, such as
c. A rise in the ESR is a normal part of aging. quiz 624
Select a statement that is true about the erythrocyte sedimentation rate (ESR)? a. It is useful in detecting pancreatic cancer. b. It is a very specific indicator of inflammation. c. A rise in the ESR is a normal part of aging. d. It is diagnostic for rheumatoid arthritis.
Eye Conditions requiring immediate referral:
Severe or sudden vision loss Eye pain Corneal ulceration Trauma Foreign body Herpetic infection of the eye (ppt in week 2)
Strategy B
Start one drug and then add a second drug before achieving maximum dose of the initial drug Start with one drug then add a second drug before achieving the maximum recommended dose of the initial drug, then titrate both drugs up to the maximum recommended doses of both to achieve goal BP If goal BP is not achieved with 2 drugs, select a third drug from the list (thiazide-type diuretic, CCB, ACEI, or ARB), avoiding the combined use of ACEI and ARB. Titrate the third drug up to the maximum recommended dose to achieve goal BP
Strategy A
Start one drug, titrate to maximum dose, and then add a second drug If goal BP is not achieved with the initial drug, titrate the dose of the initial drug up to the maximum recommended dose to achieve goal BP If goal BP is not achieved with the use of one drug despite titration to the maximum recommended dose, add a second drug from the list (thiazide-type diuretic, CCB, ACEI, or ARB) and titrate up to the maximum recommended dose of the second drug to achieve goal BP If goal BP is not achieved with 2 drugs, select a third drug from the list (thiazide-type diuretic, CCB, ACEI, or ARB), avoiding the combined use of ACEI and ARB. Titrate the third drug up to the maximum recommended dose to achieve goal BP
Controller: no daily medications needed Reliever: SABA prn <than twice per week (pg 343)
Step 1: intermittent asthma:
Controller: Preferred daily medications = low-dose ICS, Alternative daily medications = LTRA, cromolyn, nedocromil OR theophylline Reliever: SABA prn, not to exceed 3-4/per day (pg 343)
Step 2: persistent asthma, mild persistent:
Controller: Preferred daily medications = low-dose ICS or LABA or medium dose ICS Alternative daily medications = low-dose ICS plus either LTRA, theophylline or zileuton Reliever: SABA prn, not to exceed 3-4/per day (pg 343)
Step 3: Moderate persistant
Controller: Preferred daily medications = medium-dose ICS plus LABA Alternative daily medications = medium-dose ICS plus either LTRA, theophylline or zileuton Reliever: SABA prn, not to exceed 3-4/per day consider short course of oral systemic corticosteroids (pg 343)
Step 4: Severe Persistent
Controller: Preferred daily medications = high-dose ICS plus LABA AND consider omalizumab for pts who have allergies Reliever: SABA inhaled txs at 20min intervals x3 if needed consider short course of oral systemic corticosteroids (pg 343)
Step 5:
Controller: Preferred daily medications = high-dose ICS plus LABA plus oral corticosteroid AND consider omalizumab for pts who have allergies Long term therapy may include systemic corticosteroids Reliever: SABA inhaled txs at 20min intervals x3 if needed (pg 343)
Step 6:
bacterial conjunctivitis is usually caused by
Streptococcus pneumoniae, Haemophilus influenzae, group A Streptococcus S. aureua pseudomonads pg 259
b. perform a fecal occult blood test
Stuart, age 49, has sltly reduced hemoglobin and hematocrit readings. What is your next action after you ask him about his diet? a. repeat the lab tests b. perform a fecal occult blood test c. start him on an iron preparation d. start him on folic acid
a. 1
Susan has a slipped lumbar disk. when assessing her Archilles tendon, what would you expect her reflex score to be? a. 1 b. 2 c. 3 d. 4
a. estrogen plus progestin increased the risk of a cardiac event in apparently healthy women
Susan says that her previous doctor never discussed why he took her off HRT. You share with her some of the results of the women's Health Initiative (WHI). which statement is true regarding the study? a. estrogen plus progestin increased the risk of a cardiac event in apparently healthy women b. persons on HRT are at a higher risk of colorectal cancer c. postmenopausal hormones do not actually prevent fxs of the hip d. estrogen alone is associated with a greater risk of breast cancer than a combination of estrogen plus progestin
ranging from T0 to T4
T stage denotes the extent of the primary tumor
b. 3 yrs
T, age 12, asks you how long it will take him to make the complete change from preadolescence to adulthood once he starts puberty. you tell him it will take approximately: a. 2 yrs b. 3 yrs c. 4 yrs d. all of his teen yrs
a. testicular torsion
T. age 15, comes to the clinic in acute distress wit belly pain. when obtaining his hx, you find that he fell off his bike this am and has vomited. upon closer exam, you determine the belly pain to be left-sided groin pain, or pain in his left testicle. he is afebrile and reports no dysuria. you suspect: a. testicular torsion b. epididymitis c. hydrocele d. varicocele
c. you may resume sexual activity 4-6 wks after surgery
T. asks you about returning to his normal sex life after surgery for BPH. you tell him, a. you probably won't be able to have an erection after surgery; we need to discuss alternative ways of lovemaking b. you need to wait several months after surgery to make sure the site has healed c. you may resume sexual activity 4-6 wks after surgery d. you will have to ask the surgeon
a. toe walking is consider normal until age 3
T. the mother of a 2 y/o is concerned because her daughter walks on her toes all the time. what do you tell her? a. toe walking is consider normal until age 3 b. don't worry, she will outgrow it c. toe walking is normal until she starts kindergarten d. we should do further testing now
c. acyclovir (zovirax)
T., a 24 y/o African American mother of 4 young children, presents in the clinic today with varicella. She states that 3 of her children also have it and that her eruption started < than 24 hrs ago. Which action may shorten the course of the disease in T.? a. calamine lotion b. cool baths c. acyclovir (zovirax) d. corticosteroids
a. Duchenne muscular dystrophy
T., age 4, has an apparent hypertrophy of the calf muscles, which seem doughy on palpation. his mother is concerned because T. is unable to raise himself from the floor without bracing his knees with his hands. what do you suspect? a. Duchenne muscular dystrophy b. cerebral palsy c. Legg-Calve-Perthes disease d. multiple sclerosis
kidneys, brain, and bones (pg 369)
TB can spread to the
host transmission for its survival (pg 368)
TB depends on host transmission
single infectious disease (pg 368)
TB is the leading cause of death worldwide by an
HIV associated opportunistic infection in many developing countries (pg 368)
TB is the most common
>15mm are considered positive for TB (pg 372)
TB testing = all other persons who test
consumption (pg 368), during the 18 & 19 century was known as the "white plague" and is the leading cause of death worldwide from any single infection agent (pg 368)
TB used to be known as
a. asymmetry pg 231
The "A" in the ABCDEs of assessing skin cancer represents: a. asymmetry b. alpha c. Arnold's rash d. atypical
d. border irregularity pg 231
The "B" in the ABCDEs of assessing skin cancer represents: a. biopsy b. best practice c. boundary d. border irregularity
d. tinea circinata pg 169
Tinea corporis is also known as: a. onychomycosis b. tinea versicolor c. tinea manuum d. tinea circinata
d. jock itch - ringworm of the groin, is seen more frequently in men pg 169
Tinea cruris is also known as: a. onychomycosis b. tinea versicolor c. tinea manuum d. jock itch - ringworm of the groin
c. tinea of the hands pg 169
Tinea manuum is also known as: a. onychomycosis b. tinea versicolor c. tinea of the hands d. tinea corporis
a. onychomycosis and is seen more frequently in adults and the elderly pg 169
Tinea unguium is also known as: a. onychomycosis b. tinea versicolor c. tinea manuum d. tinea corporis
b. pityriasis versicolor pg 169
Tinea versicolor is also known as: a. onychomycosis b. pityriasis versicolor c. tinea manuum d. tinea corporis
GOLD - Global Initiative for Chronic Obstructive Lung Disease, stage II along with albuterol as a rescue medication (pg 354)
Tiotropium is the recommended therapy in the
a 9 month regime of INH and rifampin; ethambutol should also be included until the results of the susceptibility studies are available (pg 374)
Tx of TB for persons who cannot take Pyrazinamide (PZA or Z) like PREGNANT women consists of
Cataracts are classified as
Type I - visual acuity better than 20/40 in the affected eye(s) Type II - visual acuity of 20/40 or worse in the affected eye(s) pg 265
b. pyelonephritis
UTI in the male clt are divided into upper and lower-tract infections. a classic example of upper-tract infection includes: a. cystitis b. pyelonephritis c. prostatitis d. epidymitis
cigarette smoke, both active and passive inhalation
What accounts for 1/2 of the bladder tumors among men and 1/3 in women?
Stage D (pg 463)
There are four stages of heart failure classified as A to D that describe the evolution and progression of disease. Patients in which stage have refractory symptoms of heart failure at rest despite medical therapy, are hospitalized, or require specialized interventions or hospice care?
Grade D Recommendation against
There is at least moderate certainty based on evidence that it has no net benefit or that risks/harms outweigh benefits.
Grade C Weak Recommendation
There is at least moderate certainty based on evidence that there is a small net benefit.
Grade A Strong Recommendation
There is high certainty based on evidence that the net benefit is substantial.
Grade B Moderate Recommendation
There is moderate certainty based on evidence that the net benefit is moderate to substantial or there is high certainty that the net benefit is moderate.
b. a vesicular skin lesion on one side of the body pg 137
What is usually the first sign or symptom that a patient would present with that would make you suspect herpes zoster? a. a stabbing type of pain on one small area of the body b. a vesicular skin lesion on one side of the body c. a pain that is worse upon awakening d. a lesion on the exterior ear canal
there is no tx for ALS pg 93
What is the tx for ALS (amyotropic lateral sclerosis)?
a. nystatis oral suspension for 2 wks, 2-3ml in each side of the mouth, held for as long as possible
What is the tx for thrush? a. nystatis oral suspension for 2 wks, 2-3ml in each side of the mouth, held for as long as possible b. clotrimazole oral troches (10mg) 2x per day for 7 days c. fluconazole (100mg) twice a day for one wk d. antiseptic mouth rinses after each meal
Complaints of dysuria, urgency, frequency, nocturia, and suprapubic heaviness. (quiz 621)
What is the usual clinical presentation of an adult patient with cystitis?
b.WIQ (walking impairment questionnaire) (pg 487)
Which assessment tool is used for peripheral artery disease? a.FAQ b.WIQ (walking impairment questionnaire) c.MMSE d.MOCA
a. Asthma
Which obstructive lung disease is classified as reversible? a. Asthma b. Chronic bronchitis c. Emphysema d. COPD
MMSE and MRI scan pg 77
What two tests have the greatest usefulness in the diagnosis of dementia?
Variant (Prinzmetal's angina) (pg 449, table 10.7)
What type of angina do you suspect in Harvey who complains of chest pain that occurs during sleep and most often in the early morning hours?
c. use gel foam to control bleeding pg 233
Which of the following statements is accurate when you are removing a seborrheic keratosis lesion using liquid nitrogen? a. do not use lidocaine as it may potentiate bleeding b. pinch the skin taut together c. use gel foam to control bleeding d. this should only be performed by a dermatologist
b.The size of the TST reaction has nothing to do with erythema but is based solely on induration
Which of the following statements is true regarding tuberculin skin testing (TST)? a.Tests should be read 48 hours after the injection. b.The size of the TST reaction has nothing to do with erythema but is based solely on induration. c.It is a type V T-cell mediated immune response. d.The diameter of the induration is measured in centimeters.
d. a 45-year-old male awakened at night pg 125
Which patient is more likely to have a cluster headache? a. a female in her reproductive years b. a 40-year-old African American male c. a 55-year-old female who drinks 10 cups of coffee daily d. a 45-year-old male awakened at night
b.It is the most prevalent carcinoma of the lung in both sexes and in nonsmokers, representing 35% to 40% of all tumors (pg 381)
Which statement about adenocarcinoma of the lung is accurate? a.It is the least common type of lung cancer, representing approximately 5% to 10% of cases. b.It is the most prevalent carcinoma of the lung in both sexes and in nonsmokers, representing 35% to 40% of all tumors. c.It is more common in men than in women and occurs almost entirely in cigarette smokers. d.It is aggressive, with rapid growth and early local and distant metastases via the lymphatic and blood vessels.
d. polypharmacy is a major contributor in older adults pg 77
Which statement about confusion is true? a. confusion is a disease process. b. confusion is always temporary. c. age is a reliable predictor of confusion. d. polypharmacy is a major contributor in older adults.
Mainly caused by factors outside the urinary tract, especially immobility, that prohibit proper toileting habits. (quiz 621)
Which statement characterizes functional incontinence?
b. stroke and coronary heart disease (quiz 621)
While performing a routine physical exam on a 60-year-old hypertensive male, you notice a bruit over the carotid area on the left side of the neck. There is no induration of the skin. This patient is at higher risk for: a.abdominal aneurysm and congestive heart gailure b. stroke and coronary heart disease c. temporal arteritis and brain aneurysms d. dizziness and headaches
c. She has been on long-term corticosteroid therapy.
Why do you suspect that your patient may have a decreased response to the tuberculin skin test (TST)? a. She is on a high-protein diet. b. She is an adolescent. c. She has been on long-term corticosteroid therapy. d. She just got over a cold.
a. fine needle aspiration
Which of the following is essential for diagnosing thyroid cancer? a. fine needle aspiration b. thyroid ultrasound c. computed tomography (CT) scan d. magnetic resonance imaging (MRI)
a. fine needle aspiration pg 842
Which of the following is essential for diagnosing thyroid malignancy? a. fine needle aspiration b. thyroid ultrasound c. computed tomography (CT) scan d. magnetic resonance imaging (MRI)
a. A cough that is productive with large amounts of purulent sputum (621 quiz)
Which of the following is less likely to be found in a patient with emphysema dominant COPD? a. A cough that is productive with large amounts of purulent sputum b. Prolonged expiration c. Weight loss d. Exertional dyspnea
c. Antinuclear antibody (ANA)
Which of the following is positive in a large percentage of patients with systemic lupus erythematosus? a. Immunoglobulin b. Antiparietal antibody c. Antinuclear antibody (ANA)
c. Antinuclear antibody (ANA) 624 quiz
Which of the following is positive in a large percentage of patients with systemic lupus erythematosus? a. Immunoglobulin b. Antiparietal antibody c. Antinuclear antibody (ANA)
c. slipped capital femoral epiphysis (SCFE)
a 13 y/o obese (BMI > than 90%) boy reports low-grade lft knee pain for the past 2 months. he denies antecedent trauma but admits to frequent horseplay with his friends. the pain has progressively worsened, and he is now unable to bear wt at all on his left leg. his current c/o include left groin, thigh, and medial knee pain and tenderness. his exam demonstrates negative drawer, Lachman, and McMurray tests; left hip with decreased internal rotation and abduction; and knee flexion causing external hip rotation. based on the above scenario, you suspect? a. left meniscal tear b. left anterior cruciate ligament (ACL tear) c. slipped capital femoral epiphysis (SCFE) d. Osgood-Schlatter disease
b. advise her to use another method for now and return with her next menses
a 17 y/o female pat requests to start Depo-provera injections as her method of birth control. She discloses that she has had four sexual partners in the past year. Her LMP was 12 days ago and she had unprotected intercourse 3 days ago. The appropriate management for this pt would be to: a. administer the injection today b. advise her to use another method for now and return with her next menses c. give the injection after a negative pregnancy test and tell her to use condoms for the next 7 days d. give the injection and tell her to use a barrier method for 7 days
c. lower abdominal tenderness and adnexal tendernes
a 17 y/o female presents to your office with the c/o lower abdominal pain since her period ended 2 days ago. She has a new sexual partner in the past 3 months and does not use condms. On PE, you find that she has cervical motion tenderness. You are concerned that she may have PID. To meet the CDC minimum criteria for empiric tx of PID, she must also have: a. an oral temp greater than 101F and mucopurulent cervicitis b. a positive test for cervical infection and an adnexal mass c. lower abdominal tenderness and adnexal tenderness d. mucopurulent cervicitis and an elevated WBC
b. she should receive the HPV vaccination
a 21 y/o woman comes to your ofice and reports a hx of genital warts. In reference to the HPV vaccination (Gardasil or Cervarix), she should be educated that: a. she is not in the correct age grp and is not a candidate for the vaccination b. she should receive the HPV vaccination c. she already has been exposed to HPV; therefore, she is not a candidate for the vaccine d. there is a vaccine coming out shortly specifically for those who have been exposed. She should wait
a nephrotic problem and the patient should be referred to a nephrologist
a 24 hr urine specimen with more than 3.0 - 3.5 grams is indicative of
hypertension secondary to pheochromocytoma
a 24 hr urine test for vanillylmandelic acid and catecholamines is done to dx
abnormal (pg 585)
a 24 hr urine with more than 165 mg of protein is considered
c. clarithromycin
a 24 y/o college student who does not smoke is dx with pneumonia. He is otherwise healthy and does not need hospitalization at this time. What antibiotic represents the best choice for tx for him? a. erythromycin b. levofloxacin c. clarithromycin d. amoxicillin
c. pertussis
a 24 y/o presents with fever, rhinorrhea, and paroxysmal, high-pitched whooping cough. This is: a. bronchiolitis b. croup c. pertussis d. epiglottitis
c. stop azithromycin and initiate a penicillin this pt should have improved within 24-48 hrs
a 26 y/o being tx for CAP has been taking azithromycin in therapeutic doses for 72 hrs. His temperature has gone from 102F to 101F. What should be done? a. continue the same dose and monitor his status b. increase the dose to high dose azithromycin c. stop azithromycin and initiate a penicillin d. this is probably viral, stop the antibiotic
c. mirena IUD
a 26 y/o female comes to your office to discuss birth control options. Her hx includes migraine headaches with aura while on combination oral contraceptives in the past. She does not want to become pregnant. Which of the following birth control options would be the best choice for her? a. combined hormonal contraceptive pills b. ortho evra patch c. mirena IUD d. vaginal Nuvaring
blood in urine and frequency
a 26 y/o female presents with a cc of burning and pain on urination. There is no previous hx of a UTI. What are some other additional symptoms consistent with a dx of a lower UTI?
a. immediately
a 27 y/o female presents to your office for a Mirena (levonorgestrel intrauterine system) insertion. She reports that her menses started 3 days ago and is normal. How soon after insertion will she be able to safely rely on it for contraception? a. immediately b. after 48 hrs c. in one wk d. in one month
a. lower urinary tract infection (pg 583)
a 32 y/o presents with pain on urination. The urine microscopy of unspun urine showed >than 10 leukocytes/mL and a dipstick positive for nitrites. What is the probable cause? a. lower urinary tract infection b. candidiasis c. chlamydia infection d. pyelonephritis
c. azithromycin
a 44 y/o non-smoker is dx with pneumonia. He is otherwise healthy and does not need hospitalization at this time. Which antibiotic can be used for empirical tx according to the 2007 Infectious Diseases Society of America/american Thoracic Society? a. erythromycin b. levofloxacin c. azithromycin d. amoxicillin
b. he should receive both today he belongs to the grp of pts with chronic illnesses between ages 19 and 64
a 60 y/o pt newly dx with COPD presents to your office. He would like to get the influenza immunization. He has no evidence of having had the pneumococcal immunization. What statement is correct? a. he should receive pneumonia immunization after age 65 years b. he should receive both today c. he should receive the flu immunization annually until age 65 d. he should not receive both on the same day
c. COPD
a 60 y/o pt reports chronic cough and sputum production. He has a long hx of exposure to second hand cigarette smoke from his wife. What dx is most likely? a. lung cancer b. emphysema c. COPD d. allergic cough
a. prescribe an alpha adrenergic blocker
a 63 y/o presents to you with hematuria, hesitancy, and dribbling. DRE reveals a moderately enlarged prostate that is smooth. the clt's PSA is 1.2. what is the most appropriate management strategy for you to follow at this time? a. prescribe an alpha adrenergic blocker b. recommend saw palmetto c. prescribe an antibiotic d. refer the clt to urology
a. he should be revaccinated today the recommendation are pneumococcal immunization once at age 65 years, regardless of other disease present. However, if the initial immunization was given prior to age 65 and 5 years has elapsed since the initial one, he should be revaccinated
a 67 y/o pt presents an immunization record that reflects having received the pneumococcal immunization when he was 60 y/o. Which statement below reflects the current standard of practice recommended by ACIP and CDC for this pt? a. he should be revaccinated today b. he should receive the immunization every 5 years c. he can elect to receive it today if she has COPD d. he does not need the immunization
a. ophthalmic zoster
a 70 y/o clt with herpes zoster has a vesicle on the tip of the nose. This may indicate: a. ophthalmic zoster b. herpes simplex c. Kaposi's sarcoma d. orf and milker's nodules
d. she does not need the immunization
a 72 y/o pt presents an immunization record that reflects having received the pneumococcal immunization when she was 65 y/o. Which statement below reflects the current standard of practice recommended by ACIP and CDC for this pt? a. she should be revaccinated today b. she should receive the immunization every 5 years c. she can elect to receive it today if she has COPD d. she does not need the immunization
a. cigarette smoking
a 74 y/o pt has peripheral artery disease. Which item listed below is the most important risk factor for PAD? a. cigarette smoking b. hyperlipidemia c. diabetes d. alcohol consumption
d. all of the above (pg 612)
a 83 y/o woman with renal failure is seen. The provider should question the patient about the intake of which medications that can cause renal toxicity? a. captopril b. losartan c. ibuprofen d. all of the above
c. the buildup of synovial fluid behind the knee
a Baker's cyst is: a. an inflammation of the bursa b. a form of tendinitis c. the buildup of synovial fluid behind the knee d. the result of a swollen ligament
Lyme disease pg 141
a Bell's palsy-like facial paralysis is the most common neurologic problem caused by:
obstructive sleep apnea, the device is used for the entire sleep period every night, typically 5-20cm H2O is the pressure needed to stop apneas, snoring, and oxyhemoglobin desat in all positions and during REM sleep (pg 403)
a CPAP administered through a nasal mask has become the most common tx for
d. anthrax
a Gram stain of which lesion reveals large, square-ended, gram + rods that grow easily on blood agar? a. dermatophyte infection b. tuberculosis (scrofuloderma_ c. sarcoidosis d. anthrax
kidney to remove a stone (pg 606)
a Lithonephrotomy is an incision into the
TB (pg 332)
a Mantoux (PPD) test should be done for suspicion of
pulmonary thrombosis is suspected (pg 332)
a Pulmonary angiography is indicated when
asthma (pg 340)
a Spirometry is recommended for the dx of
in 48 to 72 hrs (pg 371) and in the older population should be read at 72 hrs
a TST should be read
bacterial infection (pg 332)
a WBC and differential is helpful in determining
a. tinea versicolor
a Wood's light is especially useful in dxing which of the following: a. tinea versicolor b. herpes zoster c. a decubitus ulcer d. a melanoma
diet, medications, acidosis, ketoacidosis, COPD pg 583)
a acidic pH is a result of
diet, sodium bicarbonate, vomiting, metabolic alkalosis, and UTI pg 583)
a alkaline pH is a result of
a. an epithelial tumor that originates from either the basal layer of the epidermis or cells in the surrounding dermal structures
a basal cell carcinoma is: a. an epithelial tumor that originates from either the basal layer of the epidermis or cells in the surrounding dermal structures b. a malignant tumor of the squamous epithelium of the skin or mucous membranes c. an overgrowth and thickening of the cornified epithelium d. lined with epithelium and contains fluid or a semisolid material
c. lipid-laden cells
a biopsy of a small, yellow-orange papulonodule on the eyelid will probably show: a. fragmented, calcified elastic tissue b. mature sebaceous glands c. lipid-laden cells d. endothelial swelling and an infiltrate rich in plasma cells
vesicle larger than 1cm in diameter pg 154
a bulla is a
a lung tumor (pg 381)
a change in one's cough or sputum production or unresponsiveness to previous effective tx should raise the suspicion of
widely accepted as the primary cross-sectional modality for evaluation of the thorax and is recommened to stage NSCLC (pg 386)
a chest CT scan with infusion of contrast material has become
hyperinflation, but may include thickening of the bronchial wall and diminished peripheral lung vascular shadows (pg 341)
a chest x-ray may only show
when the uti is unresolved after the usual tx is given (pg 594)
a chronic lower uti is characterized by
d. peripheral neuropathy
a clinical manifestation of symmetric neurogenic pain may indicate: a. radiculopathy b. reflex sympathetic dystrophy c. entrapment neuropathy d. peripheral neuropathy
c. start therapy now because the clt's CD4 count is less than 500 and the HIV RNA level is > than 10,000
a clt with HIV infection has a CD4 count of 305 and an HIV RNA level of 13,549. The clt is asymptomatic. What is your course of action? a. negotiate with your clt a time to start therapy b. recheck the lab results in 1 month. If the counts remain like this, start tx. c. start therapy now because the clt's CD4 count is less than 500 and the HIV RNA level is > than 10,000 d. wait to start therapy until the clt becomes asymptomatic
a. drug fever
a clt with HIV infection has a fever of unknown origin (FUO). Which of the following is a possible cause of an FUO in a clt with HIV? a. drug fever b. upper respiratory infection c. nothing specific; this is a systemic disease manifestation d. urinary tract infection
d. bleeding gums and delayed wound healing
a clt with a nutritional deficiency of vitamin C may have: a. dry skin and loss of skin color b. thickened skin that is dry or rough c. flaky skin, sores in the mouth, and cracks at the corners of the mouth d. bleeding gums and delayed wound healing
d. red, flat, nonblanchable petechia
a clt with a platelet abnormality may present with: a. red to blue macular plaques b. multiple freckle-like macular lesions in sun-exposed areas c. numerous small brown nonscaly macules that become more prominent with sun exposure d. red, flat, nonblanchable petechia
c. analgesia and by the use of a "donut" cushion when sitting
a coccygeal fx is tx with: a. traction b. surgical repair c. analgesia and by the use of a "donut" cushion when sitting d. prolonged bedrest for 6 wks
fluticasone and salmeterol (advair diskus) pg 354
a combination of inhaled corticosteroid and long-acting B2 agonist is
a. internal tibial torsion
a common cause of in-toeing in childhood is: a. internal tibial torsion b. femoral retroversion c. external tibial torsion d. flat feet
nocturnal awakening with one or more of the following sxs: dyspnea, cough, and wheezing (pg 340)
a common finding with asthma is
4-6 months or no longer than 12 months. Persons at risk for incomplete recovery are those older than age 55, hypertensive, with pain other than ear pain, with complete facial paralysis, and those who have changes in lacrimation, such as involuntary shedding of tears pg 141
a complete recovery from Bell's Palsy usually occurs in:
an acute or chronic infection that is accompanied by factors that complicate the infection, e.g. catheter, pregnancy, or chronic disease (pg 594)
a complicated lower uti is either
b. interstitial cystitis
a dancer from an adult club down the street comes in for a renewal of ther birth control pill prescription. She says that everything is fine. On exam, you find grayish-white vaginal discharge, greenish cervical discharge, and cervical motion tenderness. Which of the following differential dx is most UNlikely? a. gonorrhea b. interstitial cystitis c. bacterial vaginosis d. chlamydia
a. syphilis
a darkfield exam is used to cutaneous dx which disease? a. syphilis b. viral blisters c. scabies d. candidiasis
c. syphilis
a darkfield microscopic exam is used to dx: a. scabies b. leprosy c. syphilis d. infections
renal failure
a decreased serum calcium occurs in
nephrotic syndrome
a decreased serum level of albumin occurs and a decreased serum level of sodium occurs in
a. bacterial infection of the scalp
a differential dx of scarring alopecia may be: a. bacterial infection of the scalp b. Addison's disease c. drug-induced hair loss d. androgenetic baldness
a decreased estrogen level
a factor contributing to stress incontinence is
an unexplainable fever that persists for one week pg 908
a fever of unknown origin (FUO) in a hospitalized pt is
a fever greater than 101.3F (38.5C) that occurs on at least three occasions in a 3-week period pg 908
a fever of unknown origin (FUO) in an ambulatory pt is
True
a few as 3 days of malnutrition in the form of inadequate protein-calorie intake can impair normal wound-healing mechanisms True False
proteinuria, bile salts (pg 583)
a foamy appearance of urine indicates
mild exacerbation *PEFR >80% predicted or personal best *no wheezing or breathlessness *responses to SABA sustained for 4 hrs *for pts on glucocorticoid, double dose for 7-10 days AND contact provider for followup instructions (pg 348)
a good response in self management of asthma exacerbation is:
only diagnostic tool available at the ONSET of therapy, caution should be used as expectorated material is frequently contaminated by pathogenic bacteria that colonize the upper respiratory tract (pg 362)
a gram stain of sputum remains the
the standard method of diagnosis of AOM is by
a hand-held otoscope, the pneumatoscopy allows evaluation of tympanic membrane motion and is highly recommended for diagnosis (uptodate)
a. irritative voiding sxs, low back pain, and perineal pain.
a hx of UTI in males is often seen in men with chronic bacterial prostatitis. other s/s of chronic bacterial prostatitis include: a. irritative voiding sxs, low back pain, and perineal pain. b. nausea and vomiting as well as fever c. loss of appetite and wt loss d. irritative voiding sxs, inability to ambulate, and fever
renal outline and identify lower rib fractures
a kidneys, ureters, and bladder (KUB) film can visualize the
sensorineural hearing loss is defined as
a lesion in the organ of Corti or in the central pathways including the CM VIII pg 276
b. zosteriform
a linear arrangement along a nerve distribution is a description of which type of skin lesion? a. annular b. zosteriform c. keratotic d. linear
an incision into the bladder or ureter to remove calculi or place a ureteral stent (pg 606)
a lithotomy is
removal of a single lobe (pg 387)
a lobectomy is
life threatening condition and requires emergency medical care (pg 335)
a loss of 600ml of blood in 24hrs is a
d. dysplasia
a loss of DNA control over differentiation that occurs in response to adverse conditions is referred to as: a. hyperplasia b. metaplasia c. anaplasia d. dysplasia
True pg 135
a lumbar puncture is essential for the diagnosis of encephalitis True False
d. topical nasal steroid
a medication considered first line for a pt with allergic rhinitis is a: a. decongestant b. non-sedating antihistamine c. leukotriene blocker d. topical nasal steroid
d. Virchow's node
a metastatic tumor from below the diaphragm is suspected when you palpate which of the following nodes in the left supraclavicular space? a. Wringer's node b. Sim's node c. Wiskott-Aldrich node d. Virchow's node
c. harlequin sign
a mother c/o that her newborn infant lying on his/her side may appear red on the dependent side of the body while appearing pale on the upper side. When she picks up the baby, this coloring disappears. You explain to her: a. a temporary hemangioma b. hyperbilirubinemia c. harlequin sign d. mongolian spots
c. cephalosporins
a patient has a penicillin allergy. He describes an anaphylactic reaction. Which medication class should be specifically avoided in him? a. quinolone b. macrolides c. cephalosporins d. tetracyclines
a. nephrectomy (g 608)
a patient is seen for stage I renal cancer. What should the provider refer to a nephrologist for which appropriate tx? a. nephrectomy b. chemotherapy c. palliative tx d. radiation
b. what medications are you current taking? (p583)
a patient is seen in the clinic with a cc of hematuria. Which of the following data should be collected in order to make a differential dx? a. do you have a hix of liver disease? b. what medications are you current taking? c. have you noticed swelling in your ankles? d. all of the above?
hypertension (pg 603)
a patient is seen in the office c/o severe flank pain. The clinican should assess this patient for what risk factor for kidney stones?
a. NSAIDs (pg 583)
a patient is seen with hematuria confirmed on microscopic exam. The clinician should ask about taking which of these substances that might be the cause of the hematuria? a. NSAIDs b.Beets c. Vitamin A d. Red meat
pyelonephritis (pg 601)
a patient presents with CVA tenderness and a multiple day history of high fever, chills and dysuria. What is the most likely diagnosis with the given information?
b. mild obese pg 895
a patient who is 5 feet tall and weighs 150 pounds (29.3kg/m2). How would the healthcare provider classify this patient? a. overweight b. mild obese c. moderate obese d. morbid obese
c. moderate obese pg 895
a patient who is 5 feet tall and weighs 170 pounds (33.2kg/m2). How would the healthcare provider classify this patient? a. overweight b. mild obese c. moderate obese d. morbid obese
d. morbidly obese pg 895
a patient who is 5 feet tall and weighs 205 pounds (40kg/m2). How would the healthcare provider classify this patient? a. overweight b. mild obese c. moderate obese d. morbidly obese
a respiratory fluoroquinolone OR a beta-lactam IV ceftriaxone (rocephin), IV cefotaxime (claforan) (pg 366)
a patient with CAP + influenza can be tx with
b. hypersplenism
a platelet count less than 150,000/mm3 may indicate: a. possible hemorrhage b. hypersplenism c. polycythemia vera d. malignancy
removal of a whole lung (pg 387)
a pneumonectomy is
severe exacerbation *PEFR <50% predicted or personal best *marked wheezing or SOB add oral glucocorticoid repeat beta 2 agonist at once call clinician at once if no response and proceed to ER or call 911 (pg 348)
a poor response in self management of asthma exacerbation is:
b. 33%
a positive Homan's sign is present in approximately what percentage of pts with DVT? a. 25% b. 33% c. 50% d. 75%
b. induration
a positive TST reading is dependent upon which of these reactions? a. erythema b. induration
treated to minimize the possibility of reactivation TB (Pg 372)
a positive TST without clinical TB infection (negative chest x-ray and no symptoms) should be
TB infection (pg 372)
a positive reaction in a TST represents only the presence of
c. he has an infection of unknown origin
a pt demonstrates leukocytosis. This means: a. he has a bacterial infection b. he has a viral infection c. he has an infection of unknown origin d. he does not have an infection
d. doxycycline
a pt has been dx with MRSA. She is sulfa allergic. Which medication could be used to tx her? a. augmentin b. trimethoprim-sulfamethoxazole c. ceftriaxone d. doxycycline
a. cough
a pt has cough, pharyngitis, nasal discharge, and fever. Which sx is most common in a pt with acute bronchitis? a. cough b. pharyngitis c. nasal discharge d. fever
b. recent heat stroke (pg 616)
a pt is dx with acute renal failure (ARF), which of the following info obtained from the hx should alert the provider this is a case of prerenal azotemia? a. nephrolitiasis b. recent heat stroke c. recent infection where gentamicin was used in tx d. all of the above
c. how often do your sxs occur?
a pt is dx with asthma. Which question is most important to ask when deciding on medication management? a. do you smoke? b. how severe are your sxs? c. how often do your sxs occur? d. do you ever wheeze?
a. asthma exacerbation
a pt is found to have eosinophilia. An expected finding is: a. asthma exacerbation b. bronchitis c. hepatitis d. osteoporosis
c. order a stool specimen
a pt recently received an antibiotic for 10 days for pneumonia. His respiratory sxs have resolved but today he calls the office. He reports having severe watery diarrhea, abdominal cramping and low-grade fever. What should be done? a. give an anti-diarrheal agent b. encourage the pt to force fluids c. order a stool specimen d. wait 24 hrs for resolution for sxs
b. add a long acting anticholinergic he is using it too frequently, should be used only a few times weekly because if it used to frequently it will lose its effectiveness over time
a pt with COPD has been using albuterol with good relief for SOB. He is using it 3-4 times daily over the past 4 weeks. How should the NP manage this? a. encourage its use b. add a long acting anticholinergic c. tell him to use it only once daily d. add an oral steroid
a respiratory fluoroquinolone OR an advanced macrolide PLUS a beta-lactam IV ceftriaxone (rocephin), IV cefotaxime (claforan) (pg 366)
a pt with a suspected aspiration inpatient (non ICU) with CAP can treat with:
b. continue the original plan
a pt with acute bronchitis and cough for 5 days calls to report that his cough is productive of discolored sputum. He has no other new sxs. How should the NP manage this? a. consider pneumonia; prescribe a macrolide antibiotic b. continue the original plan c. encourage the pt to return to the clinic for a recheck d. order a chest x-ray and tx accordingly
c. decongestant and antitussive
a pt with acute bronchitis was dx at an urgent care center 10 days ago. He reports he was given an anti-tussive for nighttime cough, a steroid injection and oral steroids, an antibiotic, and a nasal decongestant. Which of these interventions was actually indicated for acute bronchitis? a. steroid injection and oral steroids b. antibiotic c. decongestant and antitussive d. antibiotic and steroids
a. fever
a pt with asthma presents with chest tightness, wheezing,c oughing, and fever. He is wheezing in the upper right lobe. His cough is non-productive, and he denies nasal sxs. Which sxs are not likely related to his asthma? a. fever b. coughing c. wheezing d. chest tightness
c. increase the dose of the inhaled steroid, refill the albuterol
a pt with asthma uses one puff twice daily of an inhaled steroid and has an albuterol inhaler for PRN use. He requests a refill on his albuterol inhaler. His last prescription was filled 5 weeks ago. What action by the NP is appropriate? a. refill the albuterol only b. prescribe a longer acting bronchodilator, continue the steroid c. increase the dose of the inhaled steroid, refill the albuterol d. prescribe a long acting bronchodilator and increase the steroid
dependent edema, venous engorgement, and localized pain, darkened skin color in the lower extremities, along with dryness and scaling of the sking (pg 486)
a pt with chronic venous insufficient will complain of
Vabcintcub (vancocin) OR linezolid (zyvox) (pg 366)
a pt with community-acquired MRSA pneumonia can be tx with:
a. pneumonia
a pt with cough and fever is found to have infiltrates on chest x-ray. What is his likely dx? a. pneumonia b. heart failure c. TB d. pneumonitis
b. an inhaled steroid this pt is using his "rescue" medication more than twice weekly, this is a signal that the patient's asthma is not well controlled.
a pt with intermittent asthma is using his "rescue" medication once daily. How should this be managed. He must receive a prescription for: a. a bronchodilator b. an inhaled steroid c. a long-acting beta agonist d. a leukotriene blocker
a. lymphocytosis
a pt with mononucleosis would most likely have: a. lymphocytosis b. eosinophia c. leukocytosis d. monocytosis
intermittent leg pain (intermittent claudication) in both calves that increases in severity until the pt stops walking. Pain is severe, grabbing, and cramplike. The pt may also have thick toenails with cornlike material under the nail. Eventually the lower legs and ankles assume a purple-black color characteristic of cyanosis and gangrene (pg 486)
a pt with peripheral artery disease with complain of
d. streptococcus pneumoniae
a pt with pneumonia reports that he has rust colored sputum. What pathogen should the NP suspect? a. mycoplasma pneumonia b. chlamydophila pneumoniae c. staphylococcus aureus d. streptococcus pneumoniae
non-Hodgkin's lymphome pg 855
a rare type of thyroid cancer that should be consider in person with a rapidly growing goiter is:
recurrence within two weeks of the original infection
a recurrent lowr uti is characterized by
a. the malignancy risk is related to the size of the mass
a renal mass was accidentally found on George, a 70 y/o male during his hospitalization for an episode of diverticulitis. Which of the following statements is true? a. the malignancy risk is related to the size of the mass b. because of an increase in the discovery of tumors, rates of death from kidney cancer continue to decline c. the total mortality rate at 5 years is less than 10% in patients like George d. the tx of choice is a radical nephrectomy
relaxed and supported extremity and ends with purposeful movement pg 83
a resting tremor occurs in a
low risk - consider home tx (pg 364)
a score of 0 to 1 on the CURB-65 is considered:
short in-patient hospitalization or closely monitored outpatient tx (pg 364)
a score of 2 on the CURB-65, consider:
a severe pneumonia, hospitalize and consider ICU (pg 364)
a score of 3 or greater on the CURB-65 is considered:
an early age, those with clinical emphysema who have not smokes and those with a family hx of young onset of COPD (pg 353)
a serum alpha1-antitripsin level should be done in patients who develop COPD at
PZA (pyrazinamide) Pg 375
a serum uric acid level should be done before starting
PZA (pg 375)
a serum uric acid level should be measured before starting
a. may be transmitted to a partner or newborn even in the absence of lesions because of viral shedding
a sexually active woman should be aware that genital herpes simplex virus: a. may be transmitted to a partner or newborn even in the absence of lesions because of viral shedding b. is suppressed during menstruation, physical or emotional stress, immunosuppression, sexual intercourse, and pregnancy c. recurrences usually last the same length of time as the initial outbreak d. requires the use of condoms only during outbreaks
a. COPD
a short-acting anticholinergic medication can be used alone or in combination with a short-acting beta agonist to manage sxs of which disease? a. COPD b. asthma c. glaucoma d. tachyarrhythmias
d. atrophy
a thinning of skin that appears white or translucent is defined as: a. a scale b. a cyst c. a fissure d. atrophy
c. myasthenia gravis
a thymectomy is usually recommended in the early tx of which disease? a. Parkinson's disease b. MS c. myasthenia gravis d. Huntington's choria
a. laparoscopic resection or ablation of the lesions
a tx used to improve the chance of pregnancy in an infertile woman who has minimal or mild endometriosis is: a. laparoscopic resection or ablation of the lesions b. dilation and curettage c. the use of gonadotropin-releasing hormone analogues d. the use of birth control pills for 3 months and then abruptly stopping
urine culture
a urinary tract infection is best detected by a
false
a urine osmolality is > 500m0sm/L in patients with post renal acute renal failure true or false?
d. a healthy woman of reproductive age
a vagina Ph of 4.2 is an expected finding in: a. a healthy prepubertal age girl b. a woman with trichomoniasis vaginalis c. a postmenopausal woman with atropic vaginitis d. a healthy woman of reproductive age
The most common causes of acute rhinosinusitis (ARS) is
a viral infection (uptodate)
c. apply cold for 20 min, then take it off for 30-45 min, repeat for the first 24-48 hrs while awake
a., age 18 sprained his ankle playing ice hockey. he is confused as to whether to apply heat or cold. what do you tell him? a. use continuous heat for the first 12 hrs, then use heat or cold to your won preference b. use continuous cold for the first 12 hrs, the use heat or cold to your own preference c. apply cold for 20 min, then take it off for 30-45 min, repeat for the first 24-48 hrs while awake d. alternate between cold and heat for 20 min each for the first 24-48 hrs
Risk factor for metabolic syndrome are:
abdominal obesity; triglycerides >150mg/dl, HDL-C (men <40mg/dl; women <50mg/dl) BP >130/85mm/Hg Fasting Blood Glucose >110mg/dl must have at least 3 of the above pg 439
d. tuberculous epididymitis
abnormalities of the scrotum are usually painless or nontender. which of the following is an exception and is usually tender? a. hydrocele b. tumor of the testis c. spermatocele d. tuberculous epididymitis
Escherichia coli
about 80% of all cases of dysuria are caused by ascending bacterial infection of the urinary tract by which organism?
without pathological cause og 81
about 90% of all headaches are
d. with presence of refractory retention and bladder stones
according to the American Urological Association Guideline of the Management of BPH DX and TXment Recommendations, when is referral for invasive surgery automatically warranted? a. with an AUA sx index of 7 or lower b. with an AUA sx index 8 or greater c. with irritative sxs such as urgency, frequency, or nocturia d. with presence of refractory retention and bladder stones
carbonic anhydrase inhibitor
acetazolamide (Diamox) is a
A patient presents with sudden eye pain and blurred vision. Has nausea and vomiting. Reports seeing rainbow halos around lights, there is corneal cloudiness, with diffuse conjunctival hyperemia. The pupil of the affected eye will be moderately dilated and completed UNRESPONSIVE to light. This patient is most likely experiencing
acute closed-angle glaucoma and should be referred immediately to an opthalmologist pg 262
a sudden onset (pg 331)
acute cough is
chemotherapy for TB (pg 375)
adults should have serum bilirubin, hepatic enzymes, BUN, creatinine, CBC w platelet count before starting
azithromycin and clarithromycin (pg 365)
advanced macrolides refer to
d. easy bruising
adverse effects from prolonged or high-potency topical corticosteroid use on an open lesion may include: a. epidermal proliferation b. striae c. vitilgo d. easy bruising
avoid strenuous activities for at least 2 weeks
after a renal biopsy, a patient should be instructed to:
malaise, fatigue, headache, neck pain, and stiffness and generalized pains may reflect presenting sxs, if untreated will include multiple joint arthritis pg 959
after the early symptoms of Lyme disease occur, the next symptoms are:
d. corticosteroids pg 940
after the initial tx for anaphylaxis what medication can you give to prevent late-stage anaphylactic rxs? a. albuterol b. diphenhydramine c. an H2 blocker d. corticosteroids
a. hypertension
all of the following are associated with secondary Raynaud phenomenon except: a. hypertension b. scleroderma c. repeated use of vibrating tools d. use of beta-adrenergic antagonists
b. minoxidil (rogaine)
all of the following medications may cause alopecia except: a. warfarin (cumadin) b. minoxidil (rogaine) c. levonorgestrol (norplant) d. acetalsalicylic acid (aspirin)
asthma (pg 341)
allergic rhinitis and eczema often accompany
asthma (pg 341)
allergic rhinitis and eczema often accompany a dx of
P. jirovecii during the first decade of life (pg 361)
almost all people are infected with the fungus:
scalp hair loss in patches pg 145
alopecia areata is
autoimmune endocrinopathies such as Hashimoto's thyroiditis, Addison's disease, and pernicious anemia pg 146
alopecia areata is associated with
baldness and can occur anywhere on the body where hair is present pg 145
alopecia is
hair loss over the whole body pg 145
alopecia universalis is
d. cerebellar function
alternately touching the nose with the index finger of each and and repeating the motion faster and faster with the eyes closed tests? a. CN X b. CN XI c. CN XII d. cerebellar function
inhaled corticosteroids (pg 345)
always rinse mouth with what type of inhaler?
d. polycystic kidney disease
among the following diseases, which is the most common cause of chronic renal disease: a. cystic fibrosis b. sickle cell anemia c. huntington's chorea d. polycystic kidney disease
a. anxiety or depression
an 80 y/o with COPD is likely to have concurrent: a. anxiety or depression b. thyroid disease c. diabetes d. obesity
b. levofloxacin - a respiratory quinolone
an 83 y/o healthy adult is dx with pneumonia. he is febrile but in no distress. Wha tis the preferred tx for him? a. supportive measures, it is probably viral b. levofloxacin c. azithromycin d. doxycycline
claudication (pg 487)
an ABI of 0.6 - 0.9 indicates
severe ischemia (pg 487)
an ABI of < 0.5 indicates
ABG test
an O2 sat of 90% or less warrants an
False
an abnormally elevated D-dimer test is highly sensitive and specific for the dx of thromboembolic disease True or False
cystine stones
an acidic urine pH favors precipitation of which type of kidney stone?
the onset of a UTI in a previously symptom free patient (pg 594)
an acute lower uti is characterized by
d. Pertussis
an adult has upper respiratory sxs and cough for the past 14 days. What should be considered? a. H. influenza b. S. pneumoniae c. Viral agents d. Pertussis
>0.9 = good (pg 487)
an ankle-brachil index (ABI) compares the arm BP to the ankle pressures using a BP cuff and a Doppler. The normal ratio of ankle to brachial pressure is:
a. penicillin
an eczematous skin reaction may result from: a. penicillin b. allopurinol (zyloprim) c. an oral contraceptive d. phenytoin (dilantin)
a blister pg 154
an example of a bulla is
cheilosis, a linear crack extending from the epidermis to the dermis pg 154
an example of a fissure is
a. levalbuterol other examples are: albuterol, pirbuterol, or bitolterol
an example of a short-acting beta agonist is: a. levalbuterol b. salmeterol c. mometasone d. beclomethasone
3 or more extrainguinal lymphadenopathies pg 908
an example of systemic lymphadenopathy is human immunodeficiency virus infection, in which there may be
5-7 lbs occurs with smoking cessation (pg 412)
an expected weight gain of
moderate exacerbation *PEFR 50%-80% predicted or personal best *persistent wheezing/SOB add oral glucocorticoid continue beta 2 agonist AND contact provider at once for instructions (pg 348)
an incomplete response in self management of asthma exacerbation is:
d. IgE
an increase of which immunoglobulin (Ig) signifies atopic disorders such as allergic rhinitis, allergic asthma, atopic dermatitis, and parasitic infestation? a. IgG b. IgM c. IgA d. IgE
the pt attempts voluntary movement pg 83
an intentional tremor occurs when
d. prostate cancer
an obstructive cause of lower urinary tract sxs in males is: a. bladder cancer b. bladder stones c. infection d. prostate cancer
c. expected to be completely resolved within 21 days (for the primary lesion)
an occurrence of genital herpes is: a. cured with acyclovir b. best managed with trichloroacetic acid 80%-90% applied directly to the lesion c. expected to be completely resolved within 21 days (for the primary lesion) d. not a factor in continuing with intercourse
c. guafenesin
an older adult has a cold. She calls your office to ask for advice for an agent to help her runny nose and congestion. She has hypertension, COPD, and glaucoma. What agent is safe to use? a. pseudoephedrine b. oxymetazoline nasal spray c. guafenesin d. diphenhydramine
respiratory fluoroquinolone (moxifloxacin, gemifloxacin, levofloxacin) OR azithromycin or clarithromycin PLUS high dose amoxicillin OR azithromycin or clarithromycin PLUS high dose amoxicillin-clavulanate (pg 366)
an uncomplicated CAP with recent antibiotic tx can be treated with:
macrolide (azithromycin, clarithromycin, erythromycin) OR doxycycline (vibramycin) (pg 366)
an uncomplicated CAP without recent ***antibiotic*** tx can be treated with: (antibiotic tx within the last 3 months)
can be resolved without addressing complicating factors pg 594)
an uncomplicated lower uti is one that
hospital and community setting individuals predisposed to aspiration are at risk alcoholism is the most frequent predisposing factor, CVA, seizures, and general anesthesia, periodontal disease the onset is insidious
anaerobic pneumonia occurs in both the
hair loss over the entire scalp pg 145
androgenetic alopecia is
microcytic or normocytic anemia pg 911
anemia of chronic disease may cause
immunoglobulin E (IgE) dependent anaphylactic reaction, which involves bronchospasm, laryngeal edema, and shock pg 156
angiodema and urticaria can be part of a life threatening
urticaria that involves not only edema of the dermis but the subcutaneous tissues too pg 156
angiodema is
all diabetic patients should be seen by an opthalmologist at least
annually pg 273
viral conjunctivitis does not require
antibiotics unless there is a 2nd infection, use lubrication for comfort pg 259
d. women who are several weeks postpartum after a normal labor and delivery
antibodies (inhibitors) directed against factor VIII can arise spontaneously in a number of situations. These include a. clts who have mitral regurgitation b. as sequelae to a strep throat c. clts on antibiotics d. women who are several weeks postpartum after a normal labor and delivery
asthmatic individuals (pg 333)
antihistamines are most helpful for those with allergic upper airway disease but are usually avoided in
traction or inflammatory headache and is most often due to intracranial hemorrhage, a medical emergency pg 81
any headache that is abrupt, explosive, severe, and described as the worst headache of the patient's life is suggestive of a:
colonoscopy to r/o GI cancer pg 915
any pt over the age of 50 with heme-positive stools or evidence of iron-deficiency should be referred for a
over 60 years have age or have a coexisting medical condition (pg 358)
approximately 70-80% of individuals who develop CAP are:
3-4% pg 930
approximately how much Hct is lost with each 500ml removal of blood?
90% pg 81
approximately what percentage of headaches are without pathological cause?
symmetrical pg 93
are the symptoms of ALS unilateral or symmetrical?
the brain, liver, bone, and adrenal glands (pg 381)
areas of metastases in adenocarcinoma commonly are
initially to hilar and mediastinal lymph nodes, then to the liver, adrenas, bones, and brain (pg 381)
areas of metastases in squamous cell carcinoma commonly are
b. three pg 909
as a general rule of thumb, the estimated level of hct is how many times the value of hgb? a. two b. three c. four d. five
oxalates
asparagus, beer, beets, cabbage, celery, chocolate, and cocoa, fruits, green beans, nuts, tea, colas, and tomatoes are foods high in
hand flapping upon hyperextension of the wrists with complete forward extension of the upper extremities and is an objective finding in CRF (pg 619)
asterixis is
chronic inflammatory disease characterized by reversible hyper-reactivity of the bronchi and bronchioles (pg 339)
asthma is a
d. 1 yr
at what age does the foreskin become fully retractable? a. 3 months b. 6 months c. 9 months d. 1 yr
<6.5mg/dL pg 830
at what level should you immediately treat hypocalcemia?
d. by 12 wks gestational age
at what point of fetal development does sexual differentiation occur? a. as soon as fertilization occurs b. by 4 wks gestational age c. by 8 wks gestational age d. by 12 wks gestational age
older adults and in pulmonary patients of any age (pg 353)
atrial arrhythmias are common in
older adults and in pulmonary pts of any age (pg 353)
atrial arrhythmias are common in
women, older adults, and those with diabetes likely will have
atypical or minimal symptoms with acute MI pg 451
gradual onset, dry hacking cough, fever, and prominence of constitutional sxs, e.g. pounding headaches, coryza, sore throat, shaking chills, and myalgia (pg 361)
atypical pneumonia sxs are
d. you may have difficulty salvaging iron from old red blood cells for reuse
barbie, age 27, had her spleen removed after an automobile accident. You are seeing her in the office for the first time since her dc from the hospital. She ask you how her surgery will affect her in the future. How do you respond? a. your red blood cell production will be slowed b. your lymphatic system may have difficulty transporting lymph fluid to the blood vessels c. you will have difficulty storing the nutritional agents needed to make red blood cells d. you may have difficulty salvaging iron from old red blood cells for reuse
most common skin cancer pg 236
basal cell carcinoma is the
c. confirm the dx with biopsy
before initiating cancer tx, the first crucial step is to: a. stage the cancer b. define the goals of tx c. confirm the dx with biopsy d. choose a tx plan from the many tx options
a baseline spirometry should be done, than may take 40mg of prednisone daily for 1-2 weeks. another spirometry is done, if there is a 15% improvement in the FEV1, the dose can be tapered to 7.5 to 15mg daily or every other day (pg 355)
before starting oral steroids,
lead poisoning pg 912
besides chronic alcoholism, what is another common cause of acquired sideroblastic anemia?
nephritis (pg 583)
beta-lactams, sulfonamides, NSAIDS, rifampin, cipro, allopurinol, tagamet and dilantin can cause
b. higher bone mass
black men have a relatively low incidence of osteoporosis because they have: a. increased bone resorption b. higher bone mass c. wide and thick long bones d. decreased bone deposition
d. none of the above; there is no early detection
bladder cancer can be detected early by: a. an annual urine culture b. a bladder tumor marker blood test c. an annual cystocopy d. none of the above; there is no early detection
a. cancer of the penis
bloody penile discharge is most likely to be associated with which of the following? a. cancer of the penis b. herpes zoster c. epididymitis d. Peyronie's disease
a. all women age 65 and older regardless of risk factors
bone mineral density (BMD) testing is recommended by the National Osteoporosis foundation for which fo the following clt populations to assess whether they are at high risk for osteoporosis? a. all women age 65 and older regardless of risk factors b. all men age 65 and older regardless of risk factors c. all women in their 30s for baseline d. all women of menopausal age
Bacterial conjunctivitis should be treated with
broad spectrum topic agents pg 259
myoglobin, bile pigments, melanin, cascara, and iron preparation (pg 583)
brown/black appearance of urine indicates
3mm or less (about 50% stenosis) pg 113
bruits become audible when the distal lumen is narrowed to approximately:
oral antistaphylococcal drugs for a total of 10 days pg 182
bullous impetigo should be tx with
c. itching may continue after successful tx for up to a wk
clt teaching is an integral part of successfully txing pediculosis. Which of the following statements would you incorporate in your teaching plan? a. it's ok to resume sharing combs, headsets, and so on, after being lice free for 1 month b. soak your combs and brushes in rubbing alcohol for 8 hrs c. itching may continue after successful tx for up to a wk d. spraying of pesticides in the immediate environment is essential to prevent recurrence
a. cognitive, motor, and behavioral changes
clts with AIDS typically experience the neurological symptomatic triad consisting of: a. cognitive, motor, and behavioral changes b. seizures, paresthesias, and dysesthesias c. Kaposi's sarcoma, cryptococcal meningitis, and depression d. seizures, depression, and paresthesias
a. pneumonia
clts with senile dementia of the Alzheimer's type often die of: a. pneumonia b. suicide c. pressure sores d. malnutrition
b. insert a rectal suppository and then eventually perform digital stimulation
clts with spinal cord injures often have bowel incontinence and need to have a bowel program instituted. What is the most effective way to stimulate the rectum to evacuate in the quadriplegic clt? a. administer stool softener nightly b. insert a rectal suppository and then eventually perform digital stimulation c. administer laxative every other night d. administer enemas on a regular basis
d. a thickening and broadening of the ends of the fingers
clubbing is defined as: a. elongation of the toes b. broadening of each thumb c. a birth deformity of the feet d. a thickening and broadening of the ends of the fingers
a. interrupt the cycle of urate crystal deposition and inflammatory response
colchicine may be used to terminate an acute attack of gouty arthritis, as well as to prevent recurrent episodes. the mechanism of action is to: a. interrupt the cycle of urate crystal deposition and inflammatory response b. increase serum uric acid levels c. potentiate the excretion of uric acid d. inhibit the tubular reabsorption of urate, promoting the excretion of uric acid
moderate to severe asthma
combination inhaled corticosteroids and LABAs are used to tx
closed comedone (whiteheads) and open comedones (blackheads) pg 204
comedones are noninflammatory lesions and are classified into two types:
pulmonary, cardiac, hematologic, psychogenic (pg 333)
common causes of dyspnea are
most are gram negative enteric aerobic gram negative bacili klebsiella pneumoniae pseudomonas aeruginosa staphylococcus aureus (+) oral anaerobes legionella pneumoniae (travel in colling systems, condensers, and shower heads) (pg 358
common causes of nosocomial pneumonia are:
c. ED and a decrease in bone mineral density
common clinical sxs of testosterone deficiency include: a. erectile dysfunction and a decrease in visceral fat b. an increase in lean body mass and infertility c. ED and a decrease in bone mineral density d. loss of libido and increased body hair
outside the hospital or is dx within two days after hospitalization in a patient who has not resided in a long-term care facility for 2 weeks or more prior to the onset of symptoms (pg 358)
community acquired pneumonia occurs
a. more antiplatelet effect
compared with unfractionated heparin, characteristics of low-molecular-weight- heparin (LMWH) include all of the following EXCEPT: a. more antiplatelet effect b. decreased need for monitoring of anticoagulant effect c. longer half-life d. superior bioavailability
c. application of a topical antimicrobial to the affected area
comprehensive tx for a person with peripheral occlusive arterial disease and diabetes mellitus includes all of the following EXCEPT: a. daily ASA use b. lipid lowering with an HMG-CoA reductase inhibitor (statin) c. application of a topical antimicrobial to the affected area d. maintenance of glycemic control
a. bitemporal hemianopsia
compression of the optic chiasm by a pituitary tumor will cause what type of vision changes? a. bitemporal hemianopsia b. no visual changes c. eye floaters d. cloudy vision
not a disease process or disease state but rather a sx pg 77
confusion is
clear with cough (pg 332)
crackles which are caused by fluid do not
measure of renal function (pg 620)
creatinine clearance is a far more informative diagnostic tool as a
b. testicular cancer
cryptochidism is a risk factor for: a. cancer of the prostate b. testicular cancer c. bladder cancer d. a benign testicular tumor
c. interferon B (betaseron)
current pharmacological therapy for relapsing-remitting multiple sclerosis involves: a. high dose steroids b. baclofen or diazepam c. interferon B (betaseron) d. benzodiazepines
rare autosomal recessive disorder called cystinuria, acct for approximately 1-3% of renal calculi (pg 603)
cystine stones are created because of a
rigid extension of the arms and legs, downward pointing of the toes, and backward arching of the head, this posturing is characteristic of a lesion of the midbrain, pons, or diencephalon
decerebrate posturing presents as:
pt is rigid with flexed arms, clenched fists, and extended legs, this posturing is characteristic of a lesion at or above the brainstem
decorticate posturing presents as:
urge urinary incontinence
decreased bladder capacity; bladder irritation from a UTI, turmor, stones, or irritants such as caffeine, alcohol; and CNS disorders or spinal cord lesions are all contributing factors to
c. CN VII
decreased facial strength indicates a lesion of which CN? a. CNIII b. CN V c. CN VII d. CN VIII
b. vitamin B12
deficiency of which nutritional source usually presents with an insidious onset of paresthesias of the hands and feet that are usually painful? a. thiamine b. vitamin B12 c. folic acid d. vitamin K
Recent, abrupt onset (<48 hrs) of symptoms Middle ear effusion, and Signs or symptoms of inflammation of the middle ear All 3 criteria must be present - A red TM is not enough Risk factors: tobacco smoke, URIs Complications: perforation, mastoiditis, local brain abscess, serous OME, cholesteatoma
definition of Acute Otitis Media (AOM) (ppt week 2)
decline in mental functioning, affecting memory, cognition, language, and personality pg 77
dementia is a
a. pneumocystis jiroveci pneumonia (PCP)
despite successful primary prophylaxis, which infection remains a common AIDS-defining dx? a. pneumocystis jiroveci pneumonia (PCP) b. cryptococcosis c. cryptosporidiosis d. candidiasis
ESRD (pg 618)
diabetic nephropathy is the most common cause of
limiting protein intake
dietary management for the clt in acute renal failure includes:
a viral, mycoplasma, or chlamydia infection (pg 362)
diffuse interstitial infiltrates are suggestive of
sensation of unsteadiness and a feeling of movement within the head pg 79
dizziness is the
inadequate blood flow and oxygen supply to the brain and spinal cord pg 79
dizziness occurs as a result of
no, it affects the rate of decline, so if you restart Aricept, the pt will not reach the level of function that he/she was at prior to stopping the drug pg 108
does Aricept prevent progression of Alzheimer's disease?
no, does not blanch with pressure, almost all causes of palpable purpura are serious and vasculitis must be rules out pg 155
does purpura blanch?
orthostatic hypotension
doxazosin is associated with a risk of:
alpha-adrenergic receptor blockers that relax the bladder nect and prostate smooth muscle
doxazosin, prazosin, and terazosin are all:
d. beta-adrenergic antagonist
drug therapy that had previously thought to worsen sxs in lower-extremity arterial vascular disease includes the use of: a. beta2-agonists b. calcium channel blockers c. direct thrombin inhibitors d. beta-adrenergic antagonist
a. the reduction of sweat and oil glands
dry, itchy skin in older adults results from: a. the reduction of sweat and oil glands b. loss of subcutaneous tissue c. dermal thinning d. decreased elasticity
a. what does a rolling stone gathers no moss mean?
during a mental status exam, which question might you ask to assess abstraction ability? a. what does a rolling stone gathers no moss mean? b. start with 100 and keep subtracting 7 c. what do you think is the best tx for your problem? d. what would you do if you were in a restaurant and a fire broke out?
d. what was your mother's maiden name?
during a mental status exam, which question would be the most helpful to assess remote memory? a. how long have you been here? b. what time did you get her today? c. what did you eat for breakfast? d. what was your mother's maiden name?
a. a cystocele
during a pelvic exam, you ask Mrs. K to Valsalva (strain). While doing this, a pouching is seen on the anterior wall of the vagina. This is indicative of: a. a cystocele b. a rectocele c. an enterocele d. a uterine prolapse
a. calf symmetry and leg strength
during a sports pre-participation physical exam, when you ask the clt to rise up on his toes and raise his heels, you are observing for: a. calf symmetry and leg strength b. hip, knee, and ankle symmetry c. hip, knee, and ankle motion d. scoliosis, hip motin, and hamstring tightness
True
during the first 6 weeks of the postpartum period, the childbearing woman is at increased risk for venous thrombus formation True or False
a. normal foot
during your assessment of your clt's foot, you note that the foot is in alignment with the long axis of the lower leg and that wt-bearing falls on the middle of the foot, from the hell, along the midfoot, to between the second and third toes. what do you dx? a. normal foot b. Hallux valgus c. talipes equinovarus d. hammertoes
b. anovulation
dysfunctional uterine bleeding is usually associated with: a. pregnancy b. anovulation c. genital tumor d. inflammation
Heart failure can result from a
dysfunctional ventricle that is unable to eject an adequate amount of blood (systolic dysfunction) OR inability of the ventricle to fill with a sufficient amount of blood (diastolic dysfunction) pg 461
flulike illness, fever, chills, myalgia, may report a rash or red spot that grew in size pg 958
early symptoms of Lyme disease are:
d. taking emergency contraceptive pills (ECPs)
emergency contraception refers to: a. an induced abortion inan emergency room b. quickly starting on birth control pills in anticipation of sexual intercourse c. having a medroxyrogesterone injection in the ER q12 wks d. taking emergency contraceptive pills (ECPs)
b. authentic active listening
emotional support is best given to the clt with a sexually transmitted infection by: a. offering many alternatives b. authentic active listening c. assuring the clt that everything will be okay d. emphasizing the duration of the disease
alveolar walls due to an imbalance of proteinase-antiproteinase enzymatic activity (pg 340)
emphesema is characterized by destruction of
d. persistent anovulation
endometrial cancer, hirsutism, acne, breast cancer, increased risk of diabetes,infertility, menstrual bleeding problems, and an increased risk of cardiovascular disease are clinical consequences of: a. mastalgia b. menorrhagia c. endometriosis d. persistent anovulation
allergic intersitial nephritis due to renal toxic drugs (pg 615)
eosinophiluria is characteristic of
along the temporal cranial wall and result from tears in the middle meningeal artery pg 111
epidural hematomas are caused by severe head injuries and are most common:
b. vascular
erectile dysfunction is a complex phenomenon with a variety of causes. the predominant cause is: a. psychological b. vascular c. neurogenic d. drug related
Differential diagnosis - distinguish between conductive and sensorineural causes Determine if problem with spoken word Medication history, family history, noise exposure PE - HEENT, hearing tests, Weber, Rinne If screening tests suggest impairment, refer to audiologist USPSTF - recommends periodic hearing assessment of elderly
evaluation of hearing loss (ppt week 2)
b. twice a year pg 926
every patient with sickle cell anemia should be evaluated by a hematologist at least: a. once a year b. twice a year c. every four months d. every three months
History - URI, otalgia, fever +/-, diminished hearing on the affected side, purulent drainage (if TM perforated) Pneumatic Otoscopy Organisms: pneumoniae, H. influenzae, Moraxella catarrhalis, viruses
exam findings in AOM (ppt week 2)
metabolic acidosis
excessive alcohol ingestion can cause
Epiphoria is
excessive tearing pg 245
acute exacerbations of systemic lupus erythematosus pg 964
exposure to ultraviolet B and A rays can trigger
struvite stones
extracorporeal shock wave lithotripsy is not recommended for which type of kidney stone?
allergic conjunctivitis occurs more commonly during the
fall and spring, sneezing and rhinorrhea are common pg 247
bacterial conjunctivitis occurs more commonly during the
fall and winter pg 247
c. cancers in the sigmoid colon
fecal occult blood testing (FOBT) is most effective in identifying: a. cancers in the right colon b. polyps c. cancers in the sigmoid colon d. cancers in the transverse colon
polycystic ovary syndrome pg 205
females with acne conglobata should be evaluated for
b. have the clt shrug his/her shoulders while you resist the movement
how is CN XI tested? a. ask the clt to say "ah" b. have the clt shrug his/her shoulders while you resist the movement c. have the clt stick out his/her tongue and move it from side to side d. touch the pharynx with a cotton-tipped applicator
by a Prussian blue stain of a bone marrow aspirate pg 913
how is sideroblastic anemia dx?
with calcium gluconate (94 mg of elemental calcium) in bolus and maintenance infusion pg 831
how is tetany, dyrhythmias, or seizures tx with hypocalcemia?
d. 3 hrs
how long can the vaginal contraceptive ring (Nuvaring) be out of the vagina before an additional form of contraception is necessary? a. 3o min b. one hr c. 2 hrs d. 3 hrs
2-8 wks pg843
how long does it take for the medications PTU or MMI to achieve their peak levels?
a series of at least 3 single specimens on different days should be collected (pg 372)
how many sputum specimens are needed to test for pulmonary TB?
6-8 eight ounces a day (pg 607)
how much water should a patient drink in order to prevent kidney stones? unless prevented by cardiac complications?
a. at every visit
how often does the agency for Healthcare Research and quality recommend that providers ask clts about their tobacco-use status? a. at every visit b. at least every 6 months c. once a year d. at the initial h/p examination
annually pg 874
how often should a diabetic have his/her eyes checked?
twice a year or every six months pg 92
how often should drug levels be monitored when a seizure medication has controlled the seizures and the drug level is adequate?
every 4-6 months pg 915
how often should someone with mild iron deficiency be followed up?
if the patient requires erythropoietin-alpha injections, then every 30 days for the next six months pg 915
how often should you followup with a patient with anemia chronic disease?
every 4-6 months pg 915
how often should you followup with a patient with mild iron-deficiency?
a. severe pg 913
how severe is the anemia in patients with beta-thalassemia major? a. severe b. mild c. moderate d. not at all
b. a TB skin test
how should a 20 y/o college age student who presents with cough, night sweats, and weight loss be screened for TB? a. a chest x-ray b. a TB skin test c. a sputum specimen d. questionnaire about sxs
MI, stroke, peripheral arterial disease, and sudden cardiac death (is only part of a total cardiac risk assessment)
hs-CRP has been shown to be an independent predictor of risk of pg 445
c. cervical dysplasia
human papillomavirus may lead to: a. pelvic inflammatory disease b. molluscum contagiosum c. cervical dysplasia d. genital herpes
normal urine
hyaline casts may be seen in
thiazide diuretic
hydrochlorothiazie (hydroDIURIL) is a
life threatening and is a cause of cardiac death (pg 617)
hyperkalemia in ARF can be
stone formation, reasons are unknown (pg 603)
hypertension doubles the risk for
renal failure (pg 618)
hypertensive nephropathy is the second most common cause of
spontaneously and from result of excessive intake of thyroid hormones pg 837
hyperthyroidism can occur
a. 20 to 40 yrs pg 837
hyperthyroidism peaks at what ages? a. 20 to 40 yrs b. 40-60 yrs c. 15 to 20 yrs d. no peaks, can affect at any age
a. screen for prostate or rectal malignancy
if a clt appears with sxs of BPH, a DRE is indicated in order to: a. screen for prostate or rectal malignancy b. evaluate for hypospadias c. r/o any neurological problems that may cause the sxs d. detect the presence of urethritis
c, 900 ml
if a clt with acute renal failure excretes 400ml of urine on Tuesday, how much fluid intake (both oral and IV) should the clt have on Wednesday? a. 400ml b. 600ml c. 900ml d. 1200ml
gastric aspiration is done following a period of fasting 8-10 hrs and performed before the patients arises, about 50ml of gastric content is required (pg 372)
if a patient is unable to produce a sputum culture to test for TB, what other test can be done?
refer to a nephrologist (pg 585)
if a patient's 24 hr urine protein was 4.2 g/day what should you do?
refer to a nephrologist (p584)
if a patient's 24-hr urine for protein was 4.3g/d, what should you do?
two blood samples should be obtained for culture as well as CBC and serologic analysis of sodium, urea, nitrogen, creatinine, and glucose (pg 363)
if a pneumonia is severe enough to require hospitalization, at least
erythromycin pg 182
if a pt is allergic to penicillin, what antibiotic can be used to tx bullous impetigo?
then a high suspicion is imperative and the patient should be referred to a breast specialist for further evaluation pg 155
if a rash on the nipple or areolar region lasts two weeks or longer and does not resolve with topical steroids,
a. from the beginning of the menstrual cycle until the BBT has been elevated for 3 days
if a woman is using the basal body temperature (BBT) method of birth control and does not want to become pregnant, when would you tell her to avoid unprotected intercourse? a. from the beginning of the menstrual cycle until the BBT has been elevated for 3 days b. whenever the BBT is elevated c. whenever the BBT is lowered d. fromt he end of the menstrual cycle until the BBT has been low for 5 days
multiple sclerosis pg 95
if a young adult develops trigeminal neuralgia, what diagnosis should be considered?
vena-cava filter may be used to trap emboli before they rech the lungs and cause a PE (pg 489)
if anticoagulant therapy is contraindicated, a filtering device such as a:
d. a goniometer
if any limitation or any increase in range of motion occurs when assessing the musculoskeletal system, the angles of the bones should be measured by using: a. Phalen's test b. skeletometry c. the Thomas test d. a goniometer
mitral stenosis, TB, bronchiectasis, or lung abscess (pg 335)
if hemoptysis occurs in patients age 45 or less, the likely cause is:
bronchogenic carcinoma, bronchitis, TB, pulmonary embolus with infarction (pg 335)
if hemoptysis occurs in patients age 45 or older, the likely cause is:
cervical and sometimes preauricular and submandibular nodes pg 181
if impetigo is on the face, which lymph nodes are enlarged?
axillary nodes pg 181
if impetigo is on the upper extremities, which lymph nodes are affected?
skin testing or radioallergosorbent tests (RASTs)/in vitro testing to determine sensitivity to optimize tx (pg 341)
if persistent asthma is present, what test(s) does the EPR-3 recommend?
ABG (pg 334)
if the O2 sat is <90%, what should be done?
a. diphenhydramine pg 940
if the pt is conscious and without laryngospasm after the initial tx for anaphylaxis what medication can you give to relieve cutaneous sxs a. diphenhydramine b. ibuprofen c. aspirin d. lidocaine
bronchial provocation testing may be useful with histamine, metacholine or exercise (pg 341)
if the spirometry is non-diagnostic for asthma, what other testing can be done?
should be advised to remove the patch before going to bed and then apply a new patch upon rising (pg 414)
if vivid dreams or insomnia occur with the nicotine patch, the patient
a. confront the clt
if you suspect that your clt abuses alcohol, the most appropriate action would be to: a. confront the clt b. obtain further confirmatory information c. consult with family members d. suggest Alcoholics Anonymous (AA)
warm agglutinins as this occurs at warmer temperatures pg 916
immune -mediated hemolysis occurs when RBC specific antibodies cover the RBCs - when IgG are attaching to the RBC this is known as
cold agglutinins as this occurs at cooler temperatures pg 916
immune -mediated hemolysis occurs when RBC specific antibodies cover the RBCs -when IgM are attaching to the RBC this is known as
c. hct
in a burn trauma, which blood measurement rises as a secondary result of hemoconcentration when fluid shifts from the intravascular compartment? a. hgb b. sodium c. hct d. blood urea nitrogen (BUN)
b. alkaline phosphatase to be elevated
in a clt with osteomalacia, you would expect levels of: a. serum calcium to be elevated b. alkaline phosphatase to be elevated c. creatinine excretion to be elevated d. serum phosphorus to be elevated
d. 12,000 or greater pg 925
in a patient during or after a sickle cell crisis, clinicians should expect the WBC to be elevated to a. 9,000 b. 10,000 c. 11,000 d. 12,000 or greater
True
in a patient with suspected superficial thrombophlebitis in the calf, the abnormalities in the lower-extremity exam are potentially enhanced by having the pt stand for approx. 2 min True or False
a. cigarette smoking
in a premenopausal woman, the biggest heart attack risk factor is: a. cigarette smoking b. family hx c. sedentary lifestyle d. obesity
d. chlamydia
in a pt dx with cervical gonococcal infection, you would also suspect a co-infection with: a. candidiasis b. syphilis c. trichomoniasis d. chlamydia
a respiratory fluoroquinolone OR an advanced macrolide - azithromycin, clarithromycin PLUS a beta lactam - amoxicillin-calvulanate, cefpodoxime (vantin), cefprozil (cefzil), cefuroxime (ceftin) (pg 366)
in a pt with CAP + a comorbidity, tx can be:
a. lymphocytosis and atypical lymphocytes
in a pt with monoucleosis, which laboratory abnormality is most common? a. lymphocytosis and atypical lymphocytes b. elevated monocytes c. a decreased total white count d. elevated liver enzymes
less than 100 mL
in an assessment of renal function, what is the maximum amount of urine in a 24 hr period that a clt would produce for a dx of anuria to be considered?
d. rheumatoid arthritis
in analyzing synovial fluid, a yellow-green color may indicate which of the following? a. trauma b. gout c. bacterial infection d. rheumatoid arthritis
d. Ortolani maneuver
in assessing an infant for developmental dysplasia of the hip (DDH), the NP places the infant supine, flexes the knees by holding the thumbs on the inner midthighs, with fingers outside on the hips touching the greater trochanters, stabilizes one hip, and abducts and gently pulls anteriorly on the other thigh. if this external rotation feels smooth with no sound present, there is no hip dislocation. this is: a. the Allis test b. Lasegue's sign c. the McMurray test d. Ortolani maneuver
a. supination
in assessing the skeletal muscles, you turn the forearm so that the palm is up. this is called: a. supination b. pronation c. abduction d. eversion
c. temporomandibular joint syndrome
in assessing your clt, you place the tips of your first two fingers in front of each ear and ask him to open and close his mouth. then you drop your fingers into the depressed area over the joint and note for smooth motion of the mandible. with this action, you are assessing for: a. maxillomandibular integrity b. well-positioned permanent teeth or well-fitting dentures c. temporomandibular joint syndrome d. mastoid inflammation
c. it is a bactericidal agent that acts on the cell membrane and cell wall of susceptible bacteria and binds to cellular DNA
in burn trauma, silver sulfadiazine (silvadene), a sulfonamide, is the most commonly used topical agent. What is its mechanism of action? a. it is a synthetic antibiotic that appears to interfere with the metabolism of bacterial cells b. it is a bacteriostatic agent that inhibits a wide variety of gram+ and gram -organisms by altering the microbial cell wall and membrane c. it is a bactericidal agent that acts on the cell membrane and cell wall of susceptible bacteria and binds to cellular DNA d. it is a protective covering that prevents light, air, and invading organisms from penetrating its surface
d. 7 or lower
in deciding whether to tx M., who has BPH, you use the American Urological Association scale. no txment is indicated if the AUA score is: a. 36 or higher b. 20-35 c. 8-19 d. 7 or lower
b. renal failure
in ordering imaging studies in a patient with peripheral vascular disease,t he use of radiocontrast medium can potentially result in: a. hepatic failure b. renal failure c. bone marrow suppression d. thrombocytopenia
d. saphenous vein
in patients with varicose veins, which vessel is most often affected? a. femoral vein b. posterior tibial vein c. peroneal vein d. saphenous vein
b. the linear pattern of induration can help differentiate the process from cellulitis or other inflammatory processes
in providing care for a pt with superficial thrombophlebitis, the NP considers that: a. it is a benign, self-limiting disease b. the linear pattern of induration can help differentiate the process from cellulitis or other inflammatory processes c. a chest x-ray should be obtained d. limited activity enhances recovery
a. taking a daily hot shower to relax
in teaching a clt with MS, the provider should emphasize all of the following points EXCEPT: a. taking a daily hot shower to relax b. exercising to maintain mobility c. getting plenty of rest d. seeking psychological and emotional support
a sore that does not heal
in teaching your clt about the American Cancer Society's CAUTION model, which identifies signs of many cancers, you teach her that the A stands for:
change in bowel or bladder habits
in teaching your clt about the American Cancer Society's CAUTION model, which identifies signs of many cancers, you teach her that the C stands for:
indigestion or difficulty in swallowing
in teaching your clt about the American Cancer Society's CAUTION model, which identifies signs of many cancers, you teach her that the I stands for:
b. nagging cough or hoarseness
in teaching your clt about the American Cancer Society's CAUTION model, which identifies signs of many cancers, you teach her that the N stands for: a. night sweats b. nagging cough or hoarseness c. nausea and vomiting d. noxious odor
obvious change in wart or mole
in teaching your clt about the American Cancer Society's CAUTION model, which identifies signs of many cancers, you teach her that the O stands for:
thickening or lump in the breast or elsewhere
in teaching your clt about the American Cancer Society's CAUTION model, which identifies signs of many cancers, you teach her that the T stands for:
unusual bleeding or discharge
in teaching your clt about the American Cancer Society's CAUTION model, which identifies signs of many cancers, you teach her that the U stands for:
d. myocardial infarction
in the Physicians' Health Study, middle aged men who suffer from migraine headaches are 42% more likely to have which condition when compared with nonsufferers? a. congestive heart failure b. peripheral vascular disease c. abdominal aortic aneurysm d. myocardial infarction
tx to minimize the possibility of reactivation (secondary) TB (pg 372)
in the U.S. a positive reaction in any pt but has a negative chest x-ray and no clinical sxs, it is recommended to
the herpes simplex virus pg 134
in the United States, the most common cause of encephalitis is
c. sleep disturbances
in the depressed clt, antidepressants are most effective in alleviating: a. suicidal feelings b. interpersonal problems c. sleep disturbances d. anxiety disorders
a. rheumatoid arthritis pg 944
in the general population of the U.S., what is the second most common connective tissue disease and the most destructive to the joints? a. rheumatoid arthritis b. osteoarthritis c. calcific tendonitis d. sprains
decreased creatine clearance
in the older adult, which physiological change affects pharmacokinetics?
ischemic heart disease and MI, it is considered a frequent anginal equivalent (pg 333)
in the older patient, dyspnea is the major ATYPICAL presentation for
b. bargaining
in the stages of Elisabeth Kubler-Ross's anticipatory grieving, which stage follows that of anger? a. denial b. bargaining c. depression d. acceptance
True
in the tx of a venous stasis ulcer that is not responding to standard therapy, additional therapeutic options include hyperbaric O2 therapy True False
d. Munchausen syndrome
in which syndrome does the clt often undergo multiple invasive procedures with negative findings? a. masochist syndrome b. malingering syndrome c. Munchausen syndrome by proxy d. Munchausen syndrome
Potential causes of secondary hyperlipidemia are:
inactivity alcohol abuse diabetes hypothyroidism thiazide diuretic use (high dose) chronic renal insufficiency pg 440
most cost effective means of liquefying secretion and the patient does not have heart failure (pg 333)
increasing water intake to 3-4 L/day is the
A B-type natriuretic peptide (BNP) over 500 pg/ml is
indicative of heart failure pg 464
d. both a & b pg 950
infectious mononucleosis results from an acute infection from: a. Epstein-Barr virus b. cytomegalovirus c. hepatitis d. both a & b
d. inability to conceive after 1 full yr of unprotected intercourse
infertility is best defined as the: a. inability to conceive with multiple sex partners b. inability to conceive for 9 months of unprotected intercourse when both partners are younger than 30 yrs of age c. state of voluntary childlessness d. inability to conceive after 1 full yr of unprotected intercourse
>65 and for any person who is at risk for adverse complications - for the very elderly and those who are institutionalized, revaccination should occur every 5 years (pg 367-368)
influenza vaccination is strongly recommended for those
GOLD - Global Initiative for Chronic Obstructive Lung Disease, stage I (pg 354)
inhaled short-term acting beta-2 agonist are the first line of therapy in the
*measure PEFR: >50% personal best or predicted value indicates severe exacerbation *note s/s such as cough, wheezes, chest tightness, accessory muscle use and suprasternal retractions suggest severe exacerbations (pg 348)
initial assessment of severity of self management of asthma includes:
inhaled SABA up to 3 tx of 2-4 puffs by metered-dose inhaler at 60 min intervals or single nebulizer (pg 348)
initial tx of self management of asthma exacerbation is:
pentosan polysulfate sodium (Elmiron) when tends to reduce the bladder wall inflammation (pg 599)
interstitial cystitis (IC) can be treated with
women, there is no cure and antibiotics do not help (pg 594)
interstitial cystitis (IC) is found primarily in
renal parenchyma itsels, such as glomerulonephritis, acute tubular necrosis (often caused by ischemic insult of nephrotoxic drugs such as aminoglycosides, intersitial nephritis (often an allergic type of rx to various drugs or transfusion, and tubular obstruction. Immune-mediated phenomena may lead to ARF following bacterial infections, e.g. thrombotic thrombocytopenic purpura, hemolytic uremic syndrome following gastroenteritis caused by Escherichia coli (pg 612)
intrarenal refers to disorders that affect the
true
is cigarette smoking is a risk factor for chronic renal failure true or false?
yes, and should be regarded as an emergent condition, is more commonly seen in children pg 193
is periorbital cellulitis potentially life-threatening?
yes, as long as the pt does not have advanced liver disease pg 913
is serum ferritin a reliable test of low stores of iron?
If a patient with bacterial conjunctivitis wears contacts,
it is important to treat her with a more broad-spectrum antibiotic drop, such as macrolide. handout
allergic conjunctivitis symptoms of conjunctival abnormalities are
itching and burning, mild,and diffuse pg 247
d. impetigo
large flaccid bullae, with honey-colored crusts around the mouth and nose are characteristic of: a burn b. rocky mountain spotted fever c. measles d. impetigo
10-15% of cases, the tumors are large and peripheral and are very aggressive and most often found at a late stage, have an ability to metastasize with a predilection for the SMALL INTESTINE (pg 381)
large-cell carcinoma are the least common type of lung cancer, representing about
memory loss, cognitive disturbances, mood changes, and peripheral neuropathy, plus arthritis pg 959
late symptoms of Lyme disease are:
a. within the uterine wall
leiomyomas are found: a. within the uterine wall b. on the vaginal wall c. within the cervix d. on the fallopian tube
The American Diabetes Association sets an acceptable level of HgbAIC at
less than 7% pg 273
severe infections, alcoholics, older adults and in malnutrition (pg 362)
leukopenia may be seen in
six-legged, wingless insects from 1-4mm in length that move extremely fast pg 162
lice looks like
increase blood pressure (pg 333)
licorice is suppose to calm a cough and has expectorant qualities, however licorice may
a. discontinuing cigarette smoking
lifestyle modification for patients with Raynaud phenomenon include: a. discontinuing cigarette smoking b. increasing fluid intake c. avoiding placing hands in warm water d. discontinuing ASA use
CNS sxs such as irritability, seizures and in cases of ingestion or overdose, death pg 159
lindane use has been associated with
*intermittent sxs < than once a week, brief exacerbations - lasting from a few hrs to a few days *night time asthma sxs less than twice a week *asymptomatic and normal PEF between exacerbations *PERF or FEV1 >80% predicted; PFT variability 20% (pg 342)
mild intermittent clinical features before tx are:
b. symptoms occurring more than twice weekly
mild persistent asthma is characterized by: a. limitation in activity due to bronchoconstriction b. symptoms occurring more than twice weekly c. wheezing and coughing during exacerbations d. shortness of breath with exercise
*symptoms more than 2x per week but less than 1x per day, may be several times at night/month *exacerbations may affect activity and sleep *night time asthma sxs more than twice a week *PERF or FEV1 >80% predicted; PFT variability 20%-30% (pg 342)
mild persistent clinical features before tx are:
children younger than 1 year (pg 369)
miliary tuberculosis is seen most often in
*symptoms daily, but not continual, > 1 week, nighttime *exacerbations affect activity and sleep *daily use of inhaled SABA *PERF or FEV1 60%-80% predicted; PFT variability >30% (pg 342)
moderate persistent clinical features before tx are:
a. arterial embolism with underlying atrial fibrillation
more common etiologies of acute lower extremity atherosclerotic arterial disease include: a. arterial embolism with underlying atrial fibrillation b. chronic venous insufficiency c. extension of venous thrombosis d. vessel damage
primary hyperparathyroidism
most calcium phosphate kidney stones are caused by
200-250 (pg 361)
most cases of PCP are dx when the CD4+T count drops to
b. genital herpes
most lesions of the penis are nontender and painless. which of the following conditions begins with a tender, painful lesion? a. syphilitic chancre b. genital herpes c. carcinoma of the penis d. Peyronie's disease
c. lower back pain and hypercalcemia
multiple myeloma is a plasma cell malignancy in which the bone marrow is replaced, and there is bone destruction and paraprotein formation. Myeloma is a disease of older adults overall (median age at presentation, 65 yrs). Common presenting sxs include: a. nausea and vomiting and chronic cough b. fatigue and splenomegaly c. lower back pain and hypercalcemia d. nausea and vomiting and fatigue
Atypical pneumonia or walking pneumonia because of the predominance of constitutional sxs. Children over age 5 and young adults are at greatest risk, outbreaks occur in populations living in close proximity (pg 360)
mycoplasma pneumonia is also known as an
d. a disease with extrapulmonary manifestations
mycoplasma pneumoniae is: a. a dx of exclusion b. a typical respiratory pathogen c. only identifiable on chest x-ray d. a disease with extrapulmonary manifestations
c. narcotics
nalxone (narcan) is the antidote for an overdose of: a. acetaminophen b. benzodiazepine c. narcotics d. phenothiazines
flesh-eating bacteria and loss of limb is a potential complication pg 191
necrotizing fasciitis is also known as?
As the preproliferative diabetic disease progress,
nerve fiber layer infarctions (cotton wool spots), venous beading and dilation, edema and in some cases extensive retinal hemorrhage will be noted pg 273
b. melanocytes
nevi arise from: a. plugged follicles b. melanocytes c. capillary occlusion d. epithelium
every am and worn for 16-24 her per day for usually 6-8 weeks (pg 414)
nicotine patches range from 5-22mg of nicotine and are applied
noncicatricial alopecia pg 147
non-scarring alopecia is also called
85% of all primary lung carcinoma in the US (pg 380)
non-small-cell lung cancer comprises approximately
diabetic retinopathy is a
noninflammatory disorder of the retina that develops in patients with diabetes pg 272
MI - NSTEMI is a result of a
nonocclusive thrombus that partially interrupts perfusion and only affect part of the myocardial wall rather than full thickness pg 451
ankle/brachial index (ABI) reading of >0.9 is
normal pg 487
3.3 to 4.5 g/dL
normal values for Serum albumin are
4.5 to 5.5 mEq/L
normal values for Serum calcium are
0.5 to 1.0 mg/dL
normal values for Serum creatinine are
135 to 145 mEq/L
normal values for Serum sodium are
MCV=81-99 fL pg 915
normocytic anemia is defined as anemia associated with normally sized RBCs, what is the MCV?
15% of all hospital acquired infections (pg 358)
nosocomial pneumonia is second to UTI's in hospital acquired infections and account for
is composed of cells with round to oval nuclei, the tumors are soft in consistency and have shiny gray cut surfaces (pg 380)
oat-cell carcinoma is one of the three types of SCLC and
risk factors for renal cell carcinoma (pg 607)
obesity; exposure to asbestos, cadmium, and/or gasoline; use of phenacetin and aspirin containing analgesics; and chronic hemodialysis for polycystic kidney disease and smoking are all
peripheral edema, ascites, ausculatory crackles, pericardial rub, elevated BP, hard and rapid pulse (pg 619)
objectiving findings in CRF are:
a. before age 15
obsessive-compulsive disorder sxs usually occur a. before age 15 b. during midlife crisis c. during late adolescence and early adulthood d. in later life
expiratory flow rate is impaired (pg 349)
obstructive lung diseases are those in which the
a. thrombotic
of the four types of strokes, which one is the most common and has a gradual onset? a. thrombotic b. embolic c. lacunar d. hemorrhagic
a. fatigue
of the sxs listed below, the most commonly expressed sx of women with premenstrual syndrome is: a. fatigue b. depression c. breast tenderness d. swelling of the extremities
c. stage IV
on PE, J., has pubic hair that spreads over her mons pubis with a slt lateral spread. In addition, her breast development shows breast enlargement with secondary mound formation by the developing areola. Which Tanner best describes J.'s development? a. stage II b. stage III c. stage IV d. stage V
chest x-ray AP/lateral and a sputum culture for examination for mycobacterial
once a positive TB skin test is determined, the next step is to get a
d. host modification
one major approach to cancer prevention is: a. colonoscopy b. new drug trials c. Pap smears for women of all ages d. host modification
True
one of the potential serious adverse effects of unfractionated heparin is thrombocytopenia True or False
20-30 and is more common in women pg 93
onset of MS peaks between the ages of
Definition - fluid in middle ear without signs of inflammation Caused by blockage of Eustachian tube Decreased hearing may result Difficult to distinguish from AOM May persist 3 months to 1 year following AOM
otitis media with effusion (OME) (ppt week 2)
in viral conjunctivitis, preauricular lymph nodes are
palpable pg 247
c. scarlet fever
pastia lines are present in which disease? a. toxic shock syndrome b. rocky mountain spotted fever c. scarlet fever d. meningococcemia
2-3 weeks after initiation of drug therapy (pg 379)
patients are typically considered infectious for TB approximately
a. excruciating pg 925
patients undergoing a sickle cell crisis describe the pain as: a. excruciating b. tolerable c. moderate d. minor
lung cancer (pg 352)
patients who have smoked for 20 years+ and have chronic bronchitis should be evaluated for
nephrologist (pg 622)
patients with CRF should be referred to a
c. can cough or wheeze
patients with asthma: a. all wheeze b. all cough c. can cough or wheeze d. have dyspnea
vitamin B12 deficiency alone pg 921
peripheral neuropathy is part of the differential among subjective symptoms for
arterial and venous disease (pg 483) the mechanisms of PVD are different
peripheral vascular disease (PVD) consists of what two distinct types of disorders?
44% of all cases of dizziness and vertigo pg 79
peripheral vestibular disease accts for up to
a. scabies
permethrim (elimite) applied over the body overnight from the neck down is the preferred tx for: a. scabies b. eczema c. herpes simplex d. psoriasis
b. not enough intrinsic factor
pernicious anemia is a result of: a. not enough folic acid b. not enough intrinsic factor c. not enough vitamin D d. not enough iron
two or more consecutive occasions (pg 582)
persistent hematuria occurs on
acute bacterial rhinosinusitis should be suspected in patients presenting with the following three features:
persistent symptoms or signs of ARS lasting 10 or more days with no clinical improvement onset with severe symptoms fever >39C/102F and purulent nasal discharge or facial pain lasting at least three consecutive days at the beginning of illness onset with worsening symptoms following a viral URI that lasted 5-6 days and was initially improving (uptodate)
>10mm to 14mm are considered positive (pg 372)
persons who are recent arrivals (<5 yrs), foreign-born from high risk countries, medically underserved low income and high risk racial or ethnic minority, IV users, residents/employees of high risk congregate settings, mycobacteriology lab personnel, medical conditions, 10% or < below ideal body weight who test between
>5mm to 9mm are considered positive (pg 372)
persons with HIV infection , recently exposed to clinically active TB, organ transplants who test between
a. SPT I
persons with which skin phototype (SPT) sunburn easily after 30 minutes in the sun, but never tan? a. SPT I b. SPT II c. SPT III d. SPT IV
acute meningitis rather than encephalitis pg 135
photophobia and severe headache may point to
a. an increase in immunoglobulin A and G antibodies
physiological changes in the immune system of older adults include: a. an increase in immunoglobulin A and G antibodies b. a high rate of T-lymphocyte proliferation c. an increase in the number of cytotoxic T cells d. an increase in CD8, which affects regulation of the immune system
sever dyspnea, butusually present with relatively normal ABGs, are typically very thin and typically have the barrel chest - emphysema (pg 352)
pink puffers s/s
AIDS (pg 358)
pneumocystis jirovecii (formerly carinii) pneumonia (PCP) is the leading cause of death in pts with
parenchymal disease (pg 358)
pneumonia is an infection of the alveoli, distal airways and interstitium of the lungs and thus is predominantly a:
parenchyma, usually infectious in orgin (pg 358)
pneumonia is typically an acute inflammation of the lung
1. engorgement 2. red hepatization 3. gray hepatization 4. resolution (pg 359)
pneumonococcal pneumonia typically includes four responsive stages of infection:
estrogen depletion-related changes
postmenopausal women tend to have more recurrent urinary tract infections because of:
obstruction of flow of urine from the kidneys, including ureteral obstruction, bladder neck obstruction, or urethral obstructions. Causes include BPH, prostate or bladder cancer. USE CAUTION of OTC decongestants as the alpha-agonist properties may lead to worsening prostatic hypertroy with resultant anuria (pg 612)
postrenal azotemia refers to any etiology that might lead to an
c. puberty starts before age 9.5 yrs
precocious puberty is present if: a. a delay in any of the Tanner stages takes longer than 2 yrs from one stage to the next b. the adolescent has had sexual relations c. puberty starts before age 9.5 yrs d. an adolescent rushes through all Tanner stages in less than 2 yrs
1.comorbidity (cardio or pulmonary disease) 2.current use of or recent withdrawal from systemic glucocorticoids 3. difficulty perceiving airflow obstruction or its severity 4. hx of suden severe exacerbations 5. hospitalization or emergency care within last month 6.illicit drug use 7. low socioeconomic status and URBAN residence 8. prior intubation for asthma 9. sensitivity to Alternaria 10. serious psychiatric or psychsocial problems 11. 3 or more ER visits to the ER in the past year 12. 2 or more hospitalizations in the past year 13. use of 3> canisters of inhaled, short-acting beta2 adrenergic agonists per month (pg 347)
risk factors for a fatal asthma attack are:
dizziness, N/V, diaphoresis, difficulty with balance, fluctuating hearing loss, feelings of pressure in the ear and diplopia pg 77
s/s of vestibular disease include:
cicatricial alopecia pg 147
scarring alopecia is also called
c. 9 months
screening infants for anemia should occur at what age? a. 6 months b. no screening is recommended c. 9 months d. 12 months
a benign, warty appearing growth and commonly found in adults pg 232
seborrheic keratosis is
aminoglycosides (amikacin, kanamycin) polypetitdes (capreomycin, viomycin, enviomycin) fluoroquinolone (cipro, levofloxacin, moxifloxacin) thioamides (ethionamide, prothionamide cycloserine p-Aminosalicylic acid (Pg 375)
second line TB drugs are:
b. a rise in the ESR is a normal part of aging
select a statement that is true about the erythrocyte sedimentation rate (ESR) a. it is a very specific indicator of inflammation b. a rise in the ESR is a normal part of aging c. it is useful in detecting pancreatic cancer d. it is diagnostic for rheumatoid arthritis
1. assess severity 2. initial tx 3. good response; incomplete response; poor response 4. contact provider for followup with good response; contact provider at once for instruction for incomplete response; and proceed to ER or call 911 with poor response (pg 348)
self management of asthma exacerbation includes:
males and always results in scar formation pg 205
severe acne is more common in
ankle/brachial index (ABI) reading of 0.5 or less indicates
severe ischemia pg 487
acute glaucoma presents with
severe pain, the anterior chamber may appear narrow with penlight exam, the pupil is mid-dilated nonreactive or sluggish, nausea, vomiting and headache may be present pg 247
*continuous daily sxs, frequent nighttime sxs *frequent exacerbations *physical activities limited by asthma *PERF or FEV1 <=60% predicted; PFT variability >30% (pg 342)
severe persistent clinical features before tx are:
first line defense for acute asthma attack, may be used prophylactically prn before exercise an increase in need, indicates a change in tx regime (pg 344)
short acting beta-2 agonists such as: albuterol (ventolin, proventil) Pirbuterol (maxair) are
yes, because the condition may result in blindness **pts with ophthalmic herpes will usually have a lesion on the tip of the nose** pg 137
should a pt with ophthalmic herpes affecting the first branch of the trigeminal nerve be referred to an ophthalmologist? If so, why?
yes
should pts who are dx with chronic renal failure should limit their intake of MILK, BANANAS, SOY SAUCE yes or no
c. 42 yrs
sickle cell anemia affects African Americans. Approximately 1 in 400 African Americans in the US has sickle cells disease (SCD). Advances in tx have been made, but life expectancy is still limited. The mean survival time for men with the disease is approximately: a. 24 yrs b. 34 yrs c. 42 yrs d. 52 yrs
autosomal recessive disorder in which an abnormal hemoglobin leads to chronic hemolytic anemia with a variety of severe clinical consequences
sickle cell anemia is an
autosomal-recessive disorder pg 923
sickle cell anemia is an
b. approximately 8% of american blacks
sickle cell anemia is an autosomal recessive disorder caused by the hemoglobin S gene. An abnormal hemoglobin leads to chronic hemolytic anemia with numerous clinical manifestations and becomes a chronic multisystem disease, with death from organ failure, usually between ages 40 and 50. the hemoglobin S gene is carried by: a. approximately 4% of the US population b. approximately 8% of american blacks c. approximately 4% of latinos d. approximately 12% of native americans
patients with background retinopathy should be followed up with the opthalmologist every
six months pg 273
temporary pause in breathing during sleep that lasts at least 10 seconds to 90 seconds (pg 398)
sleep apnea is defined as
Lisinopril is the only pharmacologic agent found to
slow the progression of diabetic retinopathy pg 273
15-20 % of lung cancers, invades the submucosa and is CENTRALLY located developing around a main bronchus as a whitish gray growth that invades surrounding structure (pg 380)
small-cell lung cancer comprises approximately
c. overflow incontinence
small-quantity incontinence with nearly continous dribbling is symptomatic of which kind of incontinence? a. stress incontinence b. urge incontinence c. overflow incontinence d. functional incontinence
40% less MAO B non smokers (pg 407)
smokers have
non smokers (pg 407)
smokers weight and BP are lower, heart rate faster than
b. anticonvulsants, tricyclic antidepressants, and corticosteroids
some pharmacological adjuncts to analgesics in clts with uncontrolled cancer pain include: a. anticonvulsants and tricyclic antidepressants b. anticonvulsants, tricyclic antidepressants, and corticosteroids c. selective serotonin receptor inhibitors d. benzodiazepines
b. plasmapheresis
some providers have successfully induced remission in clts with MS by using adrenocorticotropic hormone therapy or other pharmcological therapy along with: a. chelation therapy b. plasmapheresis c. bone marrow transplantation d. intravenous lipids
c. responsive to laser obliteration
spider varicosities are: a. usually symptomatic b. a potential site for thrombophlebitis c. responsive to laser obliteration d. caused by sun exposure
obstructive or restrictive disease (pg 332)
spirometry is helpful to determine the presence of
diagnosing and then in evaluating the management of asthma (pg 341)
spirometry measurements are helpful in
yellow but is not tenacious or foul smelling (pg 369)
sputum is characteristically
2nd most common skin cancer pg 240
squamous cell carcinoma is the
2nd most common lung cancer and accts for 24-35% of cases, is more common in males and occurs almost entirely in cigarettes smokers (pg 381)
squamous-cell carcinoma is the
T1, N0, M0; T2 N0, M0 pg 385
stage I =
T1, N1 M0; T2, N1, M0 pg 385
stage II =
T1, N2 M0; T2, N2 M0; T3 N0 M0; T3 N1 M0 pg 385
stage IIIA=
any T N3 M0; T4 any N M0 pg 385
stage IIIB=
any T any N M1 pg 385
stage IV
Tis, N0, M0 pg 385
stage O =
9 months of INH 4 months of rifampin OR 2 months of rifampin plus PZA (pg 372)
standard regimes in the U.S. for a positive TB skin test but negative chest x-ray and clinical sxs are
INH for 9 months rifampin for 4 months or rifampin plus pyrazinamide for 2 months (most hepatotoxic regimen)
standard tx for TB are
healthy adults, is a gram positive bacterium (pg 360)
staphylococcus aureus rarely causes pneumonia in
pregnant women or women who could become pregnant (pg 375)
streptomycin (SM, STM, S) should not be given to
it can cause congenital deafness in the fetus (pg 374)
streptomycin should not be prescribed for pregnant women because
allergic conjunctivitis discharge is
stringy, mucoid pg 247
effective bronchodilator (pg 348)
strong coffee continues to be used as an
carcinomas may mimic
styes or chalazions pg 251
epidural and subdural bleeding, they can also happen without external provocation pg 111
subarachnoid and intraparenchymal hemorrhages can be caused by trauma and can coexist with:
that knocks the brain against the skull. Movement of the brain relative to the skull tears the thin superior cerebral veins, which drain the external cerebral veins into the superior sagittal sinus venous subdural hematomas expand more slowly than the higher pressure arterial epidural hematoma pg 111
subdural hematomas are usually a result of blunt trauma:
pruritis, dry skin; nausea, vomiting, anorexia, hiccupping, emotional lability or depression, fatigue, confusion, headache, seziures, coma, urine odor to breath, SOB, metallic taste, impotence, nocturia, muscle cramp (pg 619)
subjective findings in CRF are:
first two years of life, at puberty, and during adolescence (pg 368)
susceptibility to TB is greater during the
Acute rhinosinusitis (ARS) is defined as
symptomactic inflammation of the nasal cavity and paranasal sinueses lasting less than four weeks (uptodate)
tachypnea, orthopnea, paroxysmal nocturnal (pg 333)
symptoms associated with dyspnea are
infection, hypokalemia, hypernatremia, glucose intolerance, bone demineralization (pg 346)
systemic corticosteroids may mask
a. positive antinuclear antibody (ANA), malar rash, and photosensitivity
systemic lupus erythematosus is dx on the basis of: a. positive antinuclear antibody (ANA), malar rash, and photosensitivity b. positive ANA, weight loss, and night sweats c. negative ANA, photosensitivity and renal disease d. leukopenia, negative ANA, and photosensitivity
herpes simplex conjunctivitis requires
systemic or topical agents and should be referred to an opthalmologist pg 259
myasthenia gravis there are concerns r/t hepatotoxicity, loss of consciousness, and visual disturbances textbook advises against using telithromycin (pg 365)
telithromycin (Ketek) should never be used in a pt with
c. increased destruction of RBC
thalassemia is caused by: a. blood loss b. impaired production of all blood-forming elements c. increased destruction of RBC d. autoimmune antibodies
a. pityriasis rosea
the "herald patch" is present in almost all cases of: a. pityriasis rosea b. psoriasis c. impetigo d. rubella
b. fever, fatigue, and pharyngitis
the 3 most common signs and sxs of primary HIV infection are: a. wt loss, pharyngitis, and fatigue b. fever, fatigue, and pharyngitis c. night sweats, rash, and headache d. myalgias, fatigue, and fever
1. identification of factors that exacerbate sxs 2. daily monitoring of peak expiratory flow with a sx record 3.written instruction on managing an acute asthma attack 4. intensive education and f/u emphasizing joint decision making (pgs 341-342)
the 4 principles of management of asthma include:
Ask Advice Advise Assess Assist (pg 411)
the 5 A's in smoking cessation questions are:
pain pulselessness paresthesia paralysis poikilothermia (coolness) pallor (pg 487)
the 6 Ps of PAD are:
lung cancer (pg 386)
the AP and lateral chest x-ray remains the simplest method for identifying patients with
a. women should try to exercise moderately for at least 30 min on most, if not all, days of the week
the American College of Obstetricians and Gynecologists guidelines for exercise during pregnancy and after delivery include which of the following? a. women should try to exercise moderately for at least 30 min on most, if not all, days of the week b. exercise in the supine position is the position of choice c. anaerobic exercise during pregnancy is preferred over aerobic exercise d. exercise should be discontinued upon discovery of pregnancy and be resumed after delivery
c. shoulder abduction and elbow flexion
the C5 myotome innervates: a. wrist extension b. elbow extension c. shoulder abduction and elbow flexion d. ulnar deviation at the wrist along with finger flexion and abduction
c. Kaposi's sarcoma in clt's younger than age 60
the CDC definition of AIDS includes the presence of which of the following disorders, with or without lab evidence of HIV? a. pneumonia in clts younger than age 60 b. dementia in clts younger than age 60 c. Kaposi's sarcoma in clt's younger than age 60 d. primary brain lymphoma in clts older than age 60
c. hemoglobin electrophoresis
the GOLD standard for definitive dx of sickle cell anemia is: a. reticulocyte count b. sickle cell test c. hemoglobin electrophoresis d. peripheral blood smear
a. infiltrates
the Gold Standard for dxing pneumonia on chest x-ray is the presence of: a. infiltrates b. interstitial fluid c. cavitation d. "pooling"
rotating the pt's head to one side and then lowering it slowly to 30 degrees below the bodyline. The pt should be observed for nystgmus during head rotation and vertical position pg 79
the Hallpike maneuver is performed by
b. vertigo
the Hallpike maneuver is performed to elicit: a. a seizure b. vertigo c. syncope d. a headache
benign vertigo and vertigo resulting from a CNS legion pg79
the Hallpike maneuver is used to distinguish between
d. male and female infertility rates are almost the same in the majority of cases
the Jameses are thinking about going for infertility counseling because they have been married for 5 yrs and have been unable to conceive. They ask you whether the man or the woman is usually the cause of the infertility. What do you tell them about the etiology of infertility? a. in most cases, infertility is related to a female factor b. in most cases, infertility is related to a male factor c. in the majority of cases, the etiology cannot be identified d. male and female infertility rates are almost the same in the majority of cases
b. bacterial vaginosis
the Mobiluncus species is responsible for which sexually transmitted infection? a. condylomata acuminata b. bacterial vaginosis c. human papillomavirus d. lymphogranuloma venereum
b. primary tumor
the T in the TNM staging system refers to: a. tolerance b. primary tumor c. tumor marker d. turgor
b. reduce action of androgens in the prostate
the action of a 5-alpha-reductase inhibitor in the txment of BPS is to: a. relax smooth muscle of the prostatic capsule b. reduce action of androgens in the prostate c. relieve bladder obstruction d. improve urinary flow rates
c. bladder capacity is decreased with increased obstructive changes from the prostate
the aging lower urinary tract in men undergoes which changes that can result in increased urinary sxs? a. bladder capacity is increased with lower postvoid residuals b. bladder capacity is increased with increased detrusor contractility c. bladder capacity is decreased with increased obstructive changes from the prostate d. detrusor contractility is decreased, resulting in lower post-void residuals
40 years
the aging process begins to affect the kidneys with a progressive loss of nephron units by age:
d. prevention of peripheral arterial disease
the anticipated result of debridement as part of the tx of venous stasis ulcers includes all of the following EXCEPT: a. enhanced tissue granulation b. encouragement of reepithelialization c. reduction of bacterial burden d. prevention of peripheral arterial disease
c. 50 yrs
the average age of menopause int he US is: a. 45 yrs b. 48 yrs c. 50 yrs d. 53 yrs
40-50 years pg 923
the average lifespan for someone who is homozygous for the hemoglobin S gene is
inoperability (pg 389)
the basic indication of radiation therapy is
GFR, glomerular filtration rate
the best index of kidney function is
a. hysteroscopy
the best method to dx uterine polyps is a: a. hysteroscopy b. dilation and curettage c. colposcopy d. laparoscopy
an early morning spot urine collection
the best test to determine microalbuminuria to assist int he dx of diabetic nephropathy is:
d. symptomatic sexually transmitted disease
the bladder tumor antigen test may also be positive with: a. testicular torsion b. the use of steroids for bodybuilding c. scrotal trauma d. symptomatic sexually transmitted disease
raised pg 192
the borders in erysipelas are
a. pain pg 925
the cardinal subjective symptom of sickle cell crisis is what? a. pain b. nausea c. light-headedness d. palpitations
unknown pg 94
the cause of multiple sclerosis is
pain starting in the flank and localizing in the CVA
the clinical presentation of a clt with urolithiasis would include:
Tumor-Node-Metastasis (TNM) pg 385
the current accept system for staging of lung cancer is the
six month. The initial phase of a 6 month regimen should consist of a 2 month course of INH, rifampin, pyrazinamide, and ethambutol. The second phase should consist of INH and rifampin for a total of 4 months (pg 374)
the current minimal acceptable duration of tx for all children and adults with culture-positive TB is
overnight polysomnogram, performed in a sleep center and is the GOLD STANDARD for identifying the presence, type, and severity of sleep apnea (pg 402)
the definitive test for sleep apnea is an
15% or > increase in the FEV1 after 2 puffs of a beta-adrenergic agonist has been inhaled (pg 341)
the diagnosis of asthma is made by demonstrating the reversibility of the airway obstruction from the pre/post PFTs Reversibility is defined as a:
FEV1 <70%
the diagnostic criteria for COPD is
FEV1/FVC ratio that is less than 70% and the FEV1 is the most useful parameter to assess the severity of obstruction (pg 353)
the diagnostic criterion for COPD is a
c. clotrimazole troches (10mg) 5x daily or nystatin (mycostatin) suspension 500,000-1,000,000 unites 3-5x daily
the first choice of tx for a clt who is positive for HIV and has oral candidiasis is: a. fluconazole (diflucan) 100mg PO daily b. ketoconazole (nizoral) 200mg PO daily c. clotrimazole troches (10mg) 5x daily or nystatin (mycostatin) suspension 500,000-1,000,000 unites 3-5x daily d. griseofulvin (grisactin) 500mg bid
b. lichen planus
the five Ps, purple, polygonal, planar, pruritic papules - are present in: a. ichthyosis b. lichen planus c. atopic dermatitis d. seborrheic dermatitis
children under 2 and those over the age of 65 pg 86
the highest incidence of seizures are among
overflow incontinence
the inability to empty the bladder, resulting in overdistention and frequent loss of small amts of urine, describes which type of urinary incontinence?
Non-small-cell-lung- cancer (pg 383) 2-12%, removal of the pulmonary lesion may give a dramatic remission of the arthralgia and peripheral edema
the incidence of HPO is most exclusively found in
b. casts
the increased presence of which of the following in a urinalysis indicates the presence of bacteria or protein, which is seen in severe renal disease and could also indicate urinary calculi? a. crystals b. casts c. nitrates d. ketones
1-4 days (pg 336)
the incubation period for most viral URIs are
pulmonary function tests (spirometry, diffusing capacity, and ABGs) (pg 353)
the initial diagnostic evaluation should include
d. FBS, lipid profile, TSH, testosterone
the initial dx and or lab testing that you should order to r/o organic causes of ED in men includes: a. color doppler sonography b. CBC, blood chemistry profile, TSH, and PSA c. nocturnal penile tumescene and rigidity test d. FBS, lipid profile, TSH, testosterone
d. duplex ultrasonography
the initial dx evaluation of a clinically stable pt with suspected DVT most often includes obtaining a: a. impedance plethysmography b. iodine 125 fibrinogen scan c. contrast venography d. duplex ultrasonography
The FIRST PHASE is a2 month course of INH, rifampin, pyrazinamide, and ethambutol or streptomycin in children who are too young to be monitored for visual acuity The SECOND PHASE is tx of INH and rifampin for 4 months (daily tx or 2-3 times per week) (pg 374)
the initial phase of a 6-month tx should be a
respiratory alkalosis
the kidneys excrete increased amts of HCO3 to lower the pH as a mod of compensation for which acid/base disturbance?
c. condylar joint
the knee is an example of a: a. spheroidal joint b. hinge joint c. condylar joint d. fibrous joint
b. leukocytosis
the major laboratory abnormality noted in pts with pneumonia is: a. eosinophila b. leukocytosis c. Gram stain positive d. leukopenia
b. leukocytosis
the major laboratory abnormality noted in pts with pneumonia is: a. eosinophilia b. leukocytosis c. leukoplakia d. leukopenia
diabetes mellitus and primary hypertension - ***hypertension*** is present in at least 85% of pts with CRF (pg 618)
the major underlying diseases leading to ESRD are
caused by a virus, bacterial cases are abut only 25% (pg 336)
the majority of URI are
b. in the upper outer quadrant
the majority of breast carcinomas are found in which anatomical site in the breast? a. around the areola b. in the upper outer quadrant c. in the lower half of the breast d. toward the sternum
a. superficially spreading
the majority of malignant melanomas are: a. superficially spreading b. lentigo maligna c. acra-lentiginous d. nodular
c. anthrax
the morphology of which lesion begins as an inflammatory papule that develops within several days into a painless, hemorrhagic, nad necrotic abscess, eventually with a dense, black, necrotic eschar forming over the initial lesion? a. furuncle-carbuncle b. hidradenitis suppurativa c. anthrax d. cellulitis
d. needle biopsy
the most accurate dx tool for prostate cancer is: a. DRE b. PSA c. transrectal ultrasound exam d. needle biopsy
b. mycoplasma pneumoniae
the most common cause of Atypical Pneumonia in adults is: a. streptococcus pneumoniae b. mycoplasma pneumoniae c. chlamydophila pneumoniae d. staphylococcus aureus
S. pneumoniae and is a gram positive pathogen (pg 359
the most common cause of CAP is
a. diabetes and hyperthyroidism
the most common cause of androgen deficiency in older men is decreased testosterone production by the Leydig cells within the testes. various systemic d/o can also affect circulating testosterone levels. these d/o include: a. diabetes and hyperthyroidism b. HIV and urinary tract infection c. HTN and heart disease d. tobacco use and cholecystitis
c. lumbar intervertebrak disk herniation
the most common cause of cauda equina syndrome is: a. fx b. hematoma c. lumbar intervertebrak disk herniation d. space-occupying lesion
diabetes mellitus
the most common cause of nephrotic syndrome is
a. S. pneumoniae
the most common cause of pneumonia in people of ALL ages is: a. S. pneumoniae b. Group A strept. c. S. aureus d. mycoplasma sp.
UTI
the most common cause of sepsis in older adults is
c. BPH
the most common cause of urinary incontinence in men is: a. urethritis b. prostate cancer c. BPH d. chronic bacteriuria
ureterolithiasis
the most common cause of urinary tract obstruction is:
renal hypoperfusion or nephrotoxins (pg 611)
the most common causes of ARF are associated with intrarenal injury caused by
b. ataxia
the most common central nervous system side effect of mumps in children is: a. diplopia b. ataxia c. decreased hearing d. reduced IQ
c. urinary stasis
the most common factor predisposing a woman to a urinary tract infection is a. use of an oral contraceptive b. the use of a diaphragm c. urinary stasis d. calculi
c. escherichia coli
the most common gram-negative bacteria that causes both acute and chronic bacterial prostatitis is: a. staphylococcus aureus b. klebsiella c. escherichia coli d. enterobacteriaceae
idiopathic hypercalciuria
the most common metabolic condition that predisposes to the formation of kidney stones is
b. LMWH
the most common method of preventing venous thromboembolism in higher risk surgical pts is the use of: a. vit K b. LMWH c. vena cava filter d. warfarin
a. back and shoulder pain
the most common reason for surgery for macromastia is: a. back and shoulder pain b. breast cancer c. fibrocystic breast disease d. inability to breastfeed
b. cough
the most common sx associated with acute bronchitis is: a. fever b. cough c. pharyngitis d. purulent sputum
c. retrograde ejaculation
the most common type of genitourinary dysfunction after a transurethral resection of the prostate is: a. erectile dysfunction b. urinary incontinence c. retrograde ejaculation d. decreased libido
a. indirect inguinal hernia
the most common type of hernia is a(n) a. indirect inguinal hernia b. direct inguinal hernia c. femoral hernia d. umbilical hernia
a. infiltrating ductal carcinoma
the most common type of invasive breast carcinoma is: a. infiltrating ductal carcinoma b. medullary carcinoma c. lobular carcinoma d. infiltrating papillary carcinoma
d. bacterial vaginosis
the most common type of vaginal infection is: a. candidiasis b. trichomoniasis c. gonorrhea d. bacterial vaginosis
hypotension
the most frequent complication during hemodialysis is
urinary tract infection
the most frequent reason why a person would come to your office complaining of burning on urination is:
hematuria
the most frequent sign of bladder cancer is
d. pregnancy
the most likely cause of amenorrhea is: a. an anatomical deviation b. a genetic factor c. an endocrine abnormality d. pregnancy
b. CASPAR (Classification of Psoriatic Arthritis) criteria
the most widely accepted screening tool for psoriatic arthritis (PsA) is the: a. ACR (American College of Rheumatology) criteria b. CASPAR (Classification of Psoriatic Arthritis) criteria c. psoriasis area and severity index d. rome criteria
aerobic and are attracted to the apical sections of the lungs (pg 369)
the mycobacteria tuberculosis organism is
c. agitation and restlessness
the older adult with delirium would present with which of the following behaviors? a. fatigue, apathy, and occasional agitation b. agitation, apathy, and wandering behavior c. agitation and restlessness d. slowness and absence of purpose
c. 3-5 days
the onset of anticoagulation effect of warfarin usually occurs how soon after the initiation of therapy? a. immediately b. 1-2 days c. 3-5 days d. 5-7 days
d. phobia
the persistent and irrational fear of a specific object, activity, or situation that results in a compelling desire to avoid the dreaded object, activity, or situation is defined as: a. depression b. obsession-compulsion c. agoraphobia d. phobia
c. brachytherapy
the placement of a high dose of radioactive material directly into a malignant tumor and giving a lower dose to the normal tissues is referred to as: a. radiotherapy b. teletherapy c. brachytherapy d. ionization therapy
deep venous thrombophlebitis or deep venous thrombosis (pg 486)
the presence of a thrombus within a deep vein with an accompanying inflammatory response is termed?
reddish-brown (pgs 582-583)
the presence of porphyrins, hemoglobin or myoglobin can color the urine
Coomb's test pg 917
the presence of spherocytes necessitates obtaining a
c. allow for the prevention of septicemia with prophylactic medication
the primary reason for newborn screening for sickle cell disease is to: a. present the parents with the option for genetic screening in the future b. test siblings if it is proved that the newborn has sickle cell disease c. allow for the prevention of septicemia with prophylactic medication d. prevent a sickle cell crisis
anti-inflammatory agent (pg 354)
the primary role of inhaled corticosteroids in COPD is an
b. prevent skin breakdown and the entrance of moisture and bacteria but allow permeability of )2 and moisture vapor
the purpose of a transparent dressing such as Tegaderm applied over a pressure ulcer is to: a. toughen intact skin and preserve skin integrity b. prevent skin breakdown and the entrance of moisture and bacteria but allow permeability of )2 and moisture vapor c. allow necrotic material to soften d. use the proteolytic enzymes in the dressing to serve as a debriding agent
delayed cellular hypersensitivity reaction (IV T-cell-mediated immune response) pg 370-371)
the reaction to intradermally injected tuberculin is the classic example of what type of reaction?
flat and diffused pg 192
the red borders in cellulitis are
45-70% (pg 484)
the risk for pts undergoing hip surgery or knee reconstruction is:
c. fluid rich in fructose
the seminal vesicles secrete: a. clear mucus b. urine c. fluid rich in fructose d. semen
free T4 assay when the TSH level is either undetectable or 10 mU/L or higher pg 837
the sensitive TSH level should be followed by a
c. gradually wean the baby to a bottle or a cup over a 3 wk period
the simplest and safest method of suppressing lactation after it has started is to: a. wear a snug brassiere b. use ice packs c. gradually wean the baby to a bottle or a cup over a 3 wk period d. begin oral hormones or long-acting hormonal injections
b. having the clt increase his fluid intake
the single-most effective method of treating urinary calculi is: a. prescribing an antibiotic b. having the clt increase his fluid intake c. performing lithotripsy d. performing cystoscopy
shock, denial, anger, bargaining, depression and acceptance
the stages of Elisabeth Kubler-Ross are:
a. nerve root compression
the straight leg raising maneuver can be used to dx: a. nerve root compression b. a fx hip c. an anterior cruciate ligament tear d. tendinitis
millet seed, approximately 2mm in diameter (pg 373)
the term miliary is appied to the appearance of nodules that are about the size of a
b. during the 3rd trimester
the tests in male infants descend from the retroperitoneal space through the inguinal canal and into the scrotal sacs. this most commonly occurs: a. during the 2nd trimester b. during the 3rd trimester c. during the neonatal period d. by age 6 months
5-15mcg/mL (pg 355)
the therapeutic level of theophylline is
1. chest x-ray 2. leukocyte count 3. gram stain of sputum specimens (pg 362)
the three most helpful test to dx pneumonia include the
c. alopecia universalis
the total loss of hair on all parts of the female body is referred to as: a. female pattern alopecia b. alopecia areata c. alopecia totalis c. alopecia universalis
a. has a hx of being a victim of abuse
the typical perpetrator in a domestic violence situation is one who: a. has a hx of being a victim of abuse b. has a criminal record c. is involved in a new relationship d. is on a lower socioeconomic scale
b. cardiac arrhythmias
the use of tricyclic antidepressants in the elderly increases the risk of: a. suicide b. cardiac arrhythmias c. reactive depression d. shortness of breath
a. rubeola
the viral exanthem of Koplik's spots is present in: a. rubeola b. rubella c. fifth disease d. varicella
c. 11 yrs
thelarche is the first sign of puberty in most girls. It usually occurs at about what age? a. 8 yrs b. 10 yrs c. 11 yrs d. 13 yrs
moderate and severe persistent asthma, occasionally is used as mild persistent asthma (step 2)
theophylline is a long-acting bronchodilator used for
tea leaf (pg 349)
theophylline's name means
d. hypothalamus
there are many causes of amenorrhea. In ballet dancers or marathon runners, which anatomical structure is the probable cause? a. outflow tract b. ovary c. anterior pituitary d. hypothalamus
a. high estrogen states, such as with oral contraceptive
there are many precursors of deep venous thrombosis (DVT), such as decreased blood flow, injury to the blood vessel wall, and altered blood coagulation. which clinical risk factor for DVT is the result of hypercoagulability? a. high estrogen states, such as with oral contraceptive b. trauma that results in orthopedic surgery c. an indwelling intravenous catheter of a lower extremity d. prolonged sedentary position related to immobility
obstructive and restrictive (pg 349)
there are two primary forms of lung disease, they are:
True pg 104, advise vaccinations
there is an increase risk of mortality from influenza and pneumonia among patients with Parkinson's disease True False
calcium phosphate and/or oxalate stones
thiazide diuretics are used for the tx of which type of renal calculi?
bacterial conjunctivitis has a
thick purulent discharge, crust eyelids in the morning pg 247
c. anemia
thin, spoon-shaped nails are usually seen in: a. trauma b. fungal infection c. anemia d. psoriasis
cefdinir, (omnicef) cefoperazone, (cefobid) ceftibuten (cedax) are very effective against common respiratory pathegen and have few adverse effects and interactions with other drugs. they have two advantages, once a day dosing for COPD and few GI effects (pg 355)
third generation oral cephalosporins such as
1. allergens and environmental factors 2. infections, uri's are common precursors to asthma attack 3. psychologic factors - stress at work/home/school (pg 339)
three principal triggers for exacerbations of asthma are:
For BEST therapeutic effect
thrombolytics should be administered within the first 3 hrs and ideally within 30 mins of presentation of an MI pg 458
d. onchomycosis
tinea unguium is also known as: a. tinea capitis b. pityriasis versicolor c. tinea manuum d. onchomycosis
d. apley's compression test
to aid in the dx of meniscus damage, which test should you perform? a. bulge test b. lachman test c. drawer sign d. apley's compression test
on most days for a minimum of 3 months of the year, for at least 2 consecutive years (pg 349)
to dx chronic bronchitis, a chronic or recurrent cough must be present:
a. the occiput, low cervical, trapezius, and supraspinatus
to dx fibromyalgia, there must be tenderness on digital palpation in at least 11 of 18 (nine pairs) tender-point sites, which would include: a. the occiput, low cervical, trapezius, and supraspinatus b. the proximal interphalangeal (PIP), metacarpophalangeal (MCP) joints of thehands, and the metatarsophalangeal (MTP) and PIP joints of the footthe face joints of the cervical, thoracic, and lumbar spine d. the radial and ulnar styloids and the medial and lalteral malleoli
c. computer programmers
to plan for a community education program, the NP needs to know that persons at highest risk for developing thoracic outlet syndrome (TOS) are: a. bicycle riders b. dancers c. computer programmers d. swimming instructors
b. bladder cancer
tobacco has been linked to which of the following types of cancer? a. colon cancer b. bladder cancer c. prostate cancer d. cervical cancer
seizures pg 88
too much gluatamate or not enough GABA or vitamin B6 can lead to
a. tampon contamination with staphylococcus aureus
toxic shock syndrome (TSS) may be causes by which of the following? a. tampon contamination with staphylococcus aureus b. a short vaginal canal c. the use of superabsorbent tampons d. a UTI involving the bladder and kidneys
one occasion (pg 582)
transient hematuria occurs on
the use of anti-cholinergic agents
transient urinary incontinence in women may be caused by
b. a hydrocele
transillumination of fluid in the scrotum may be seen with: a. a varicocele b. a hydrocele c. testicular torsion d. testicular cancer
b. low-dose steroids
treatment of choice for polymyalgia rheumatica (PMR) is. a. acetaminophen or NSAIDs b. low-dose steroids c. tricyclic antidepressants d. antibiotics
a. compression stockings
treatment of superficial venous thrombophlebitis in a low-risk, stable pt includes use of: a. compression stockings b. acetaminophen c. warfarin d. heparin
d. applying a moisture-retaining dressing
treatment options for venous stasis ulcers in the lower extremities include: a. cleansing with hydrogen peroxide b. applying Burow solution c. prescribing a systemic corticosteroid d. applying a moisture-retaining dressing
physiologic, essential, toxic, cerebellar, Parkinsonian, resting, and intentional pg 83
tremors are sometimes classified into seven groups. What are the 7 groups?
a. CN V pg 138
trigeminal neuralgia affects which cranial nerve? a. CN V b. CN VII c. CN X d. CN III
d. a transparent, semipermeable membrane dressing
tx for a stage I pressure ulcer may include: a. an enzymatic preparation b. systemic antibiotics c. surgical tx with muscle flaps d. a transparent, semipermeable membrane dressing
a. edema
typically, the earliest sign of lower-extremity venous insufficiency is: a. edema b. altered pigmentation c. skin atrophy d. shiny skin
d. acute blood loss
under which of the following circumstances is the reticulocyte count elevated? a. aplastic anemia b. iron-deficiency anemias c. poisonings d. acute blood loss
BMI <18 or 15-20% below ideal weight pg 835
underweight is defined as:
a. an intraductal papilloma
unilateral galactorrhea may be present with: a. an intraductal papilloma b. a woman who is lactating c. a ruptured breast implant d. pregnancy
sexual or violent abuse pg 954
up to 50% of patients with fibromyalgia syndrome have a history of
d. nausea and vomiting
urinary stone disease, or urolithiasis, affects a lg number of clts every yr, exceeded in frequency as a urinary tract d/o only by infections and prostate disease. this d/o affects men more frequently than women with a ratio of 3:1. common presenting s/s include which of the following? a. guarding of the abdomen b. fever c. pain that is present during the daytime hrs d. nausea and vomiting
d. 30 days
urine drug screening tests, often performed in the workplace, are frequently effective in finding a person who smokes marijuana because a urine test will be positive for marijuana for up to how long after a person stops smoking the drug? a. 24 hrs b. 1 wk c. 2 wks d. 30 days
dehydration
urine specific gravity is increased in clients with
72 hrs
urine tends to become colonized when indwelling urinary catheters are left in place for more than
b. bile
urine that appears brownish in color may result from which of the following? a. beets (red) b. bile c. rifampin (orangish) d. phenazopyridine (reddish/orange)
generalized eruption of pale, evanescent wheals or papules that is associated with severe itching pg 155-156
urticaria is defined as a sudden
clot migration from the legs to the lungs (pg 489)
vena-cava filters do not prevent clot formation, however they prevent
Virchow's Triad
venous stasis vessel injury hypercoagulability pg 485
Presentation Painless or mild discomfort "gritty" Watery to serous discharge Unilateral at onset then bilateral w/in 1-2 days Preauricular lymphadenopathy Treatment Adenovirus is most common pathogen Antihistamine Lubricant drops Cool compresses Often treated with antibiotic drops dt inability to diagnostically rule out bacterial involvement
viral conjunctivitis (ppt week 2)
The most common condition of red eye is
viral conjunctivitis pg 246
5-15% of adult CAP and are restricted to the upper respiratory system, secondary pneumonia is most frequently caused by S. pneumoniae and S. aureus are transmitted by hand to hand or by coughing/sneezing (pg 360)
viral pneumonia infections acct for
ethambutol (pg 375)
visual acuity and red-green color perception tests should be done before starting
ethambutol (pg 375)
visual acuity and red-green color testing should be done prior to using
A hallmark of macular degeneration is
visual difficulties in low light pg 275
oxaluria by facilitating oxalate metabolism - reduce oxalate stone (pg607)
vitamin B6 and magnesium are both known to decrease
Abdominal obesity = women
waist circumference >35 inches pg 439
Abdominal obesity = men
waist circumference >40 inches pg 439
viral conjunctivitis discharge is
watery pg 247
memantine (Namenda) by improving cognitive function and has additive effects to acetylcholinesterase inhibitors pg 108
what N-methyl-D-asparate receptor antagonist has been effective in txing moderate to severe Alzheimer's disease?
urine output less than 500mL/day
what amt of urine in a 24 hr period in an adult represents oliguria?
SSRIs (pg 356)
what antidepressant class is safe for those with lung disease?
superficial skin infections predominantly caused by three fungal species pg 169
what are dermatophytoses or tinea?
EnoxaPARIN (Lovenox) daltePARIN (Fragmin) tinzaPARIN (Innohep) and are given subcutaneously (pg 488)
what are examples of LMWH:
exercise, anxiety, elevated body temperature, hot baths or showers, will usually resolved 30 min after the offending activity is stopped pg 156
what are factors that trigger cholinergic urticaria?
montelukast (singulair) zafirlukast (accolate) cromolyn sodium (intal) nedocromil sodium (tilade) pg 345)
what are leukotriene receptor antagonist drugs?
reversible airflow limitation and diurnal variation as measured by peak flow meter, nasal discharge mucosal swelling frontal tenderness nasal polyps allergic shiners check for eczema use of accessory muscles auscultation of inspiratory and expiratory wheezes may be present (wheezing during forced exhalation is no longer considered a reliable factor) (pg 340
what are objective signs of asthma?
kidneys, brain, and bones are the most common sites of progression (pg 369)
what are other areas TB may progress to besides the lungs?
zafirlukast montelukast
what are some leukotriene blockers?
minoxidil (rogaine) finasteride (propecia) pg 146-147
what are some medical treatments for alopecia?
1. distinguish between bacterial vs viral (lobar infiltrates suggest bacterial while diffuse interstitial infiltrate are suggestive of a viral infection) 2. r/o a pleural effusion, a complication in 1/3 of pneumococcal pneumonia pts 3. cavities may be seen on films in pts with pneumonia caused by anaerobes, S aureus, S. pneumoniae serotype III, mycobacterium TB, aerobic gram neg bacilli, and fungi (pg 362)
what are the 3 important reasons for a Chest x-ray to r/o pnuemonia?
1. allergens and environmental factors 2. infections - common URI, viral infections 3. psychologic factors - stress at work/home or crisis (pg 339)
what are the 3 principle triggers for exacerbation of asthma?
muscle contraction = tension mixed = combination of muscle contraction and vascular vascular = migraine and cluster traction or inflammation pg 81
what are the four (types) classes of headaches?
1. iron deficiency 2. anemia of chronic disease (ACD) 3. thalassemia 4. siderblastic anemia pg 910
what are the four causes of microcytic anemia?
1. vitamin B12 deficiency 2. folate deficiency 3. antimetabolite drugs such as methotrexate 4. miscellanceous etiologies pg 919
what are the four general categories of cause of macrocytic anemia?
1. initiating stage - kidney is injured 2. oliguric stage - can last 5-15 days or weeks * the longer this stage, the poorer the outcome* 3. diuretic stage - beginning when urine output is >400mL/day and BUN begins to fall 4. recovery phase - extends from the time the BUN is stable, urine output is normal and the pt returns to normal activities (pg 614)
what are the four stages of ARF?
type 1: IgE-mediated immediate hypersensitivity type 2: antibody-mediated cellular cytotoxicity response type 3: antibody-allergen complex response type 4: delayed -type cellular hypersensitivity response pg 939-940
what are the four types of immune responses?
relapsing-remitting primary progressive secondary progressive progressive relapsing pg 93
what are the four types of multiple sclerosis?
peripheral blood smear, CBC, platelet count pg 935
what are the initial tests for acute leukemia and chronic leukemia?
antithyroid drugs, radioactive iodine, and surgery pg 842
what are the three tx options for Graves' disease?
PTU propylthiouracil MMI methimazole pg 843
what are the two antithyroid medications used today?
1)whether it is scarring or nonscarring alopecia 2)whether hair loss is in a small, well-circumscribed area or generalized pg 145
what are the two factors a NP should address in the evaluation of alopecia?
bullous and nonbullous pg 180
what are the two forms of impetigo?
b. Alzheimer's disease and vascular disorder
what are the two most common causes of dementia in older pts? a. polypharmacy and nutritional disorder b. Alzheimer's disease and vascular disorder c. metabolic disorders and space-occupying lesions d. infections affecting the brain and polypharmacy
1) nonpharmacologic measures to reduce bacterial colonization on the skin 2) antibiotics to eradicate the responsible pathogen and prevent recolonization and complications pg 182
what are the two principles of therapy in the management of impetigo?
ischemic and hemorrhagic pg 109
what are the two types of cerebral accidents?
Trousseau's sign (carpal spasm) Chvostek's sign (spasm of the facial muscle) pgs 830-831
what are two neuromuscular signs that can be tested and are indicative of hypocalcemia?
HIV, AIDS, or chronic renal failure pg 914
what are typical causes of anemia of chronic disease?
d. C-reactive protein pg 950, normal C-reactive protein is <0.8mg/dL pg 950
what blood test is a nonspecific method and most helpful for evaluating the severity and course of an inflammatory process? a. erythrocyte sedimentation rate b. WBC c. polymorphonuclear cells d. C-reactive protein
very high WBC >500,000, blurred vision, respiratory distress, and occasionally priapism, more commonly found in CML pg 935
what characterizes leukostasis and between CML or CLL which is it more commonly found in?
red (pg 582)
what color will urine change to with 100 RBCs per hpf?
pink (pg 582)
what color will urine change to with 20-30 RBCs per hpf?
ACEi, (-pril) drugs (pg 332)
what common antihypertensive drug is known to cause a dry cough?
hemodynamically induced acute renal failure
what condition may result from clients takings NSAIDs on a long-term basis?
d. sxs of prostate cancer in general tend to progress more rapidly than those of BP
what differentiates prostate cancer sxs from BPH sxs? a. urinary frequency, hesitancy, and intermittency are much worse with prostate cancer b. nocturia is worse with BPH c. dribbling and a weak stream are more indicative of BPH d. sxs of prostate cancer in general tend to progress more rapidly than those of BPH
the induration and ulceration, pg 235
what differentiates squamous cell carcinoma from actinic keratosis?
c. polymyalgia rheumatica
what do affects older individuals, particularly women and is characterized by pain and stiffness in the cervical spine and shoulder and hip girdles, along with signs of systemic infection such as malaise, wt loss, sweats, and low-grade fever? a. fibromyalgia syndrome b. myofascial somatic dysfunction c. polymyalgia rheumatica d. Reiter's syndrome
are too small to be seen on a regular microscope, they will appear as extremely small dark specks on the slide under both low and high power pg 167
what do bacteria look like under the microscope?
round to oval and are the size of nuclei in epithelial cells and are often seen in candidal and trichomonal infections because of inflammation pg 167
what do leukocytes look like under the microscope?
small and oval to round in shape, seen either alone or in clusters pg 167
what do spores look like under the microscope?
forced expiratory volume in one second
what does FEV1 mean?
light in color; resemble rbcs in shape and size or may be a larger dumbbell form (pg 604)
what does a calcium stone look like?
c. a narrowing of the carotid artery because of atherosclerosis of the vessel
what does a carotid bruit heard on auscultation indicate? a. a normal finding in adults recovering from a carotid endarterectomy b. an evolving embolus c. a narrowing of the carotid artery because of atherosclerosis of the vessel d. a complete occlusion of the carotid artery
lemon yellow, hexagonal, and sparkle under light microscopy. Medications such as topiramate, triamterene, and sulfadiazine and protease inhibitor indinavir may cause cystine stones (pg 604)
what does a cystine stone look like?
d. expiratory flow
what does a peak flow meter measure? a. exercise capacity b. oxygen saturation c. peak flow capacity d. expiratory flow
have thin translucent walls that have septa dividing each segment, like a bamboo stem pg 167
what does a pseudohyphae or hyphae look like under the microscope?
the adult female measures 0.3-0.5mm long, round body with 4 pairs of short legs pg 157
what does a scabie look like?
flat, consist of hexagon-shaped crystal that are radioaque and of form secondary to a uti caused by Proteus mirabilis (pg 604)
what does a struvite stone look like?
are radiolucent, red-orange with a teardrop or flat square shape, may be associated with a hereditary etiology of gout or with idiopathic causes (pg 604)
what does a uric acid stone look like?
it is one of the neuromuscular signs indicating hypocalcemia and is a significant sign of tetany pg 830
what does carpopedal spasm indicate?
should raise suspicion that the dx may be paroxysmal nocturnalhemoglobinuria (PNH) pg 917
what does dark red urine first thing in the morning indicate?
an ominous sign, indicates a poor prognosis because the immune system is not responding to a potentially fatal pathogen
what does leukopenia mean in a pt with pneumonia?
c. rise in neutrophils
what does shift to the left or left shift mean? a. rise in basophils b. rise in monocytes c. rise in neutrophils d. rise in lymphocytes
usually resistive and ropy in texture pg 831
what does the glandular enlargement of gynecomastia feel like?
nitrofurantoin (macrobid, macrodantin) (pg 332)
what drugs may cause interstitial fibrosis and associated cough?
kegel exercises (pg 588)
what exercise can be taught to female patients who are experiencing stress or urge incompetence?
chest x-ray (332)
what followup test should occur if the Mantoux PPD test is positive?
beta-hemolytic streptococcal infection
what hx is commonly found in a clt with glomerulonephritis?
eliminate caffeine from your diet (pg 590)
what instruction should be given to a patient with overactive bladder?
exposure to moldy han can result in sxs of pneumonia, e.g. coughing, fever, chills, malaise, and dyspnea within 4-8 hrs following exposure, this disease may become chronic (pg 364)
what is "farmer's lung"?
cerumen or hairs impinging on the tympanic membrane which may cause a cough (pg 332)
what is Arnold's reflex?
deletions or severe mutations in both beta-globin genes result in beta-thalassemia major resulting in a severe, transfusion-dependent, lifelong anemia with skeletal abnormalities pg 912
what is Cooley's anemia?
directly observed therapy, because of the difficulty in predicting which patient will adhere to a prescribed regime (pg 374)
what is DOT and why is it important?
Bamberger-Marie disease (pg 383)
what is HPO also known as?
patients with Parkinson's disease have: Tremor at rest, Rigidity Akinesia or bradykinesia Postural disturbances pg 101
what is TRAP?
It is a urinary diversion procedure because of a urinary tumor. It is a continent internal ileal reservoir in which nipple valves are formed on the skin. the filling pressure closes the valves, preventing leakage and reflux
what is a Koch pouch?
seborrheic keratosis pg 232
what is a benign, warty appearing growth and commonly found in adults
a large, multiloculated abscess made up of multiple furuncles pg 188
what is a carbuncle?
c. sexual abuse
what is a common factor in pts with noneliptiform seizures? a. stress b. anxiety c. sexual abuse d. malingering
a reduced number of RBCs form from ring sideroblast precursors found in the bone marrow pg 912
what is a diagnostic hallmark of sideroblast anemia?
paresthesia pg 83
what is a early symptom of herpes zoster?
a deep bacterial infection of a hair follicle with abscess formation pg 188
what is a furuncle?
a hair on the center of the pustule sometimes perforates the lesion pg 185
what is a hallmark presence for folliculitis?
the reversibility of the phenomenon (pg 341)
what is a key feature in making the dx of asthma?
polypharmacy pg 77
what is a major contributor of confusion in older adults?
d. cryosurgery
what is a noninvasive method of txing skin cancer (other than melanoma) that uses liquid nitrogen? a. Moh's micrographic surgery b. curettage and electyrodessication c. radiation therapy d. cryosurgery
poststreptococcal glomerulonephritis pg 183
what is a possible complication from nonbullous impetigo?
pulmonary embolism (PE) pg 486)
what is a potential complication of deep venous thromboembolism?
seizure-like sxs but are not associated with EEG changes, up to 80% are associated with early sexual abuse, especially among females pg 89
what is a pseudoseizure?
persistent slow-growth enlargement pg 909
what is a reason to consider lymph node aspiration and cytologic evaluation?
serum ferritin, a value less than 30mg/L is considered pathological pg 913
what is a reliable test of low iron stores in a pt who does not have advanced liver disease?
if there is an actual DVT, it may cause the thrombus to release (pg 487)
what is a risk with the ascending venogram?
a. coal tar preparations
what is a safe and effective tx for mild psoriasis? a. coal tar preparations b. systemic steroids c. topical antibiotics d. systemic antihistamines
it is a progressive neurologic disorder that involves destruction of the anterior horn motor cell in the spinal cord or the brain stem pg 93
what is amyotrophic lateral scloerosis?
it falls under vascular lesions and is bleeding into the skin layers and surrounding tissue as a result of trauma or coagulation defects pg 155
what is an ecchymosis?
freckles pg 153
what is an example of a macule?
moles (nevi), less than 1cm in diameter, elevated and palpable on top of the skin pg 153
what is an example of a papule?
a large bruise (can be punctate to larger-sized lesions) pg 155
what is an example of a purpura?
a bruise pg 155
what is an example of an ecchymosis?
pharyngitis pg 908
what is an example of regional lymphadenopathy?
HIV pg 908
what is an example of systemic lymphadenopathy?
a. keloid
what is an excessive amt of collagen that develops during scar formation called? a. keloid b. skin tag c. angioma d. keratosis
variable RBC size pg 913
what is anisocytosis?
a. St. Anthony's fire pg 191
what is another name for erysipelas? a. St. Anthony's fire b. Georgia's fire c. St. Mary's fire d. erythema
a streptococcal infection of the superficial layers of the skin and does not involve the subcutaneous layers, unlike the more typical cellulitis pg 191
what is erysipelas?
b. balanoposthitis
what is inflammation of the glans and prepuce called? a. balanitis b. balanoposthitis c. phimosis d. paraphimosis
a hyperpigmentation caused by increased levels of estrogen, progesterone, and of melanocyte-stimulating hormone during pregnancy pg 148
what is melasma or chloasma?
b. actinic keratosis
what is most common precancerous skin lesion found in Caucasians: a. a skin tag b. actinic keratosis c. a melanoma d. a basal cell lesion
renal blood flow reduced by 50% after an ischemic episode (pg 613)
what is no-reflow phenomenon?
chronic alcoholism pg 912
what is one of the most common causes of acquired sideroblastic anemia?
iron deficiency pg 910
what is one of the most common causes of microcytic anemias in the world?
weakness, it may be local to a single extremity, face or involve more than one extremity, it may develop suddenly or gradually, may be permanent or transient pg 81
what is paresis?
an abnormal sensation described as numbness or tingling, cramping or pain without a known stimulus and is felt along peripheral nerve pathways pg 81
what is paresthesia?
infestation by lice pg 161
what is pediculosis?
variable RBC shape pg 913
what is poikilocytosis?
breathe deeply and clear the lungs every 2-4 hrs regularly (pg 335)
what is pulmonary toiletry?
b. ankylosis
what is stiffness or fixation of a joint called? a. contracture b. ankylosis c. dislocation d. subluxation
test for scabies by using blue or green felt tip pens, rub the pen over the suspected burrow, remove the excess ink with an alcohol wipe, the remaining ink concentrates in the tunnel and indicates the location of the burrow pg 159
what is the Burrow Ink test?
renal angiography (pg 620)
what is the GOLD STANDARD to test for renal vascular flow for arterial stenosis?
points are allotted for the three major risk factors of age,number of cigarettes smoked per day, and FEV1 as a percentage of the value predicted.
what is the Tecumseh Index?
venous stasis vessel wall injury coagulation abnormalities
what is the Virchow triad?
a. 2-15 yrs pg 931 table 17.2
what is the age of onset for acute lymphocytic leukemia (ALL)? a. 2-15 yrs b. 15-20 yrs c. 30-45 yrs d. 60-75 yrs
all patients, but more prominent >60 yrs, the median age is 42 pg 931 table 71.2
what is the age of onset for chronic granulocytic leukemia (CGL) or chronic myelogenous leukemia (CML) ?
b. > 60 yrs pg 931 table 17.2
what is the age of onset for chronic lymphocytic leukemia? a. 2-15 yrs b. > 60 yrs c. 30-35 yrs d. 20-25 yrs
b. all adults but more prominent >40 years pg 931 table 17.2
what is the age of onset for nonlymphocytic leukemia (ANLL) or acute myelogenous leukemia (AML) and acute granulocytic leukemia (AGL) ? a. 2-15 yrs b. all adults but more prominent >40 years c. 30-35 yrs d. 20-25 yrs
asthma, allergic rhinitis and eczema pg 212
what is the atopic triad?
b. a sinus
what is the connection between the surface of the skin and an underlying structure called? a. an ulcer b. a sinus c. an erosion d. an abscess
6 months (pg 374)
what is the current minimal acceptable duration of tx for all children and adults with culture positive TB?
b. epinephrine 1:1,000 sq (0.3-0.5ml) for adults
what is the drug of choice for acute anaphylaxis? a. benadryl 25-100mg PO qid for adults b. epinephrine 1:1,000 sq (0.3-0.5ml) for adults c. prednisone (2mg/kg q 24 hrs) PO in one initial daily dose, tapered off over 1-2 wks d. amlodipine besylate (norvasc) 5mg qid for 4 wks
up to 5-6 years unless the problem begins in the brain stem, then there is problems with aspiration and the course is more rapid pg 93
what is the duration of ALS?
3 months
what is the duration of sxs and cough for Pertussis?
b. mucosal erythroplasia
what is the earliest visual sign of oral and pharyngeal squamous cell carcinomas? a. leukoplakia b. mucosal erythroplasia c. loss of sensation in the tongue d. difficulty chewing or swallowing
IV replacement of calcium and is guided by the severity of hypocalcemia paired with the pt's s/s pg 831
what is the emergency tx of hypocalcemia?
encouraging hydration
what is the first step in the tx of uric acid kidney stones?
c. rest tremor
what is the first sx seen int he majority of clts with Parkinson's disease? a. rigidity b. bradykinesia c. rest tremor d. flexed posture
45% pg 930
what is the goal Hct% goal in those with absolute polycythemia?
to control the disease so the pt no longer feels physically or psychologically hindered by the skin lesions pg 227
what is the goal of therapy for psoriasis?
a sudden onset, interruption of ongoing activities, a blank stare and possibly a brief upward rotation of the eye, the attack may last from a few seconds to a half a minute pg 86
what is the hallmark of an absence seizure?
the rapid progress and severity of symptoms, progresses in hours as opposed to days pg 191
what is the hallmark of necrotizing fasciitis?
7-10 days
what is the incubation period for Pertussis?
2-3 weeks of incubation (pg 360)
what is the incubation period for mycoplasma pneumonia?
d. eliminate the heat source
what is the initial emergency measure to limit burn severity? a. stabilzie the clt's condition b. identify the type of burn c. prevent heat loss d. eliminate the heart source
rehydration pg 929
what is the initial management of relative polycythemia?
b. 2.0 -3.0
what is the international normalized ratio (INR) range recommended during warfarin therapy as part of the management of a pt with DVT? a. 1.5 - 2.0 b. 2.0 -3.0 c. 2.5 - 3.5 d. 3.0 - 4.0
c. the knee
what is the largest join in the body? a. the hip b. the shoulder c. the knee d. the elbow
c. streptococcus pneumoniae
what is the leading cause of bacterial meningitis in adults? a. haemophilus influenzae b. meningococcal c. streptococcus pneumoniae d. gram-negative pneumonia
ultrasound imaging
what is the least expensive method for evaluating renal mass size?
death
what is the likeliest outcome in clts with CKD?
b. to keep the clt functioning independently as long as possible
what is the main overall goal of therapy for the clt with Parkinson's disease? a. to halt the progression of the disease b. to keep the clt functioning independently as long as possible c. to control the sxs of the disease d. to ease the depression associated with the disease
progressive phlebotomies pg 929
what is the management for absolute polycythemia if the Hct is > than 55-60%?
d. they alter the host environment for cell growth
what is the mechanism of action of steroid hormones in cancer chemotherapy? a. they interfere with DNA or RNA synthesis b. they interfere with DNA replication by attacking DNA synthesis throughout the cell cycle c. they inhibit protein synthesis d. they alter the host environment for cell growth
c. clomipramine (anafranil)
what is the medication of choice for obsessive-compulsive disorders? a. alprazolam (xanax) b. carbamezipine (tegretol) c. clomipramine (anafranil) d. buspirone (buspar)
24 hr collection (pg 585)
what is the most accurate way to quantify the amount of protein in the urine?
b. S. pneumoniae
what is the most common cause of CAP world-wide? a. Group A strept. b. S. pneumoniae c. S. aureus d. mycoplasma sp.
d. alcoholism
what is the most common cause of cerebellar disease? a. hypothyroidism b. use of drugs such as 5-fluorouracil or phenytoin c. cerebellar neoplasm d. alcoholism
diabetic nephropathy
what is the most common cause of chronic renal failure?
cardiovascular failure
what is the most common cause of death in dialysis patients?
c. diabetic nephropathy
what is the most common cause of end-stage renal disease (ESRD)? a. hypertensive nephropathy b. glomerulonephritis c. diabetic nephropathy d. acute tubular necrosis
c. a varicocele
what is the most common cause of male infertility? a. azoospermia b. a problem with sperm motility c. a varicocele d. antisperm antibodies
d. staphylococcus aureus
what is the most common cause of mastitis in breastfeeding women? a. escherichia coli b. streptococcus c. mycobacterium tuberculosis d. staphylococcus aureus
c. iron-deficiency anemia pg 910
what is the most common cause of microcytic anemia? a. anemia of chronic disease b. sideroblastic anemia c. iron-deficiency anemia d. thalassemia
an enlarged thyroid gland pg 833
what is the most common cause of neck size?
Grave's disease pg 837
what is the most common cause of spontaneous hyperthyroidism in the U.S.
pneumothorax is the most common complication (pg 387)
what is the most common complication associated with a needle guided by CT or fluoroscopy?
pneumonia (pg 338)
what is the most common complication of influenza?
a. non-bacterial prostatitis
what is the most common prostatitis syndrome found in males of any age? a. non-bacterial prostatitis b. prostatodynia c. acute bacterial prostatitis d. chronic bacterial prosttitis
d. sun exposure
what is the most common rosacea trigger? a. alcohol b. cold weather c. skin care products d. sun exposure
b. spine
what is the most common site of prostate metastasis? a. lungs b. spine c. heart d. blood vessels
Ziehl-Neelsen (pg 373)
what is the most common staining method for direct microscopy of sputum?
a. pruritus from the lesions pg 180
what is the most common symptom associated with both types of impetigo? a. pruritus from the lesions b. nausea c. fever d. red, moist rash
a. cytomegalovirus
what is the most common virus to be transmitted in utero? a. cytomegalovirus b. rubella c. varicella d. toxoplasmosis
a. tinea capitis, is seen more frequently in children until puberty pg 169
what is the most contagious of all dermatophytoses? a. tinea capitis b. tinea versicolor c. tinea manuum d. tinea corporis
c. avoidance of the offending agent
what is the most effective tx for urticaria? a. an oral antihistamine b. dietary management c. avoidance of the offending agent d. a glucocorticosteroid
clubbing of the fingers, also associated with hypertrophic pulmonary osteoarthropathy (HPO) pg 383, often resembles RA
what is the most frequent peripheral sign of lung cancer?
c. avoid smoking
what is the most important thing a woman can do to have youthful, attractive skin? a. keep well hydrated. b. use sunscreen with an SPF of at least 45 c. avoid smoking d. use mild defatted or glycerin soaps
d. y-glutamyltransferate (GGT)
what is the most sensitive dx test for identifying an alcoholic clt? a. aspartate transaminase (AST, also called SGOT) b. mean corpuscular volume c. alkaline phosphatase d. y-glutamyltransferate (GGT)
c. altered mental status
what is the most sensitive indicator of increase intracranial pressure and the first sx to change as the pressure rises? a. dilation of the pupil b. hyperventilation c. altered mental status d. development of focal neurological signs such as hemiparesis
c. alcohol decreases the ability of persons to adhere to a prescribed medical regime
what is the most significant reason alcohol use is discouraged in persons with HIV infection or AIDS? a. alcohol interferes with the pharmacokinetics of most AIDS b. filling up on the empty calories of alcohol replaces the desire for food c. alcohol decreases the ability of persons to adhere to a prescribed medical regime d. if clts become addicted to alcohol, when AIDS advances, they will become addicted to painkillers
the aggressiveness, resulting in a more rapid growth and early local and distant metastasis via lymphatic and blood vessels (pg 380)
what is the most striking difference between small-cell-lung cancer and other malignant lung cancers?
b. vitiligo
what is the name of the acquired disorder characterized by complete loss of pigment of the involved skin? a. tinea versicolor b. vitiligo c. tuberous sclerosis d. pityriasis alba
a. 9-11 mg/dL pg 830
what is the normal serum calcium values in adults? a. 9-11 mg/dL b. 6-8 mg/dL c. 13-14 mg/dl d. 0.2-.09 mg/dl
its reversibility (pg 341)
what is the one key feature in making the diagnosis of asthma?
dullness
what is the percussion tone heard over a distended bladder?
d. anteverted
what is the positionof the uterus when the cervix is on the anterior vaginal wall? a. midposition b. retroverted c. retroflexed d. anteverted
fiberoptic bronchoscopy (pg 363)
what is the preferred invasive method to obtain lower respiratory secretions>
d. magnetic resonance imaging (MRI) pg 842
what is the preferred test assess for ophthalmopathy resulting from Graves' disease? a. fine needle aspiration b. thyroid ultrasound c. computed tomography (CT) scan d. magnetic resonance imaging (MRI)
start at the lowest dose that brings adequate symptom reversal pg 102
what is the rule of thumb to administer levodopa therapy for patients with Parkinson disease?
simple partial seizure pg 86
what is the seizure called when the patient does NOT lose consciousness?
bone marrow aspirate pg 935
what is the subsequent test for acute leukemia and chronic leukemia after the blood draws and results?
venous thromboEmbolism (pg 486)
what is the term when a thrombus breaks away from the wall of the vein and travels upward toward the heart?
topical 5-fluorouracil (5-FU) cream for eradicating actinic keratoses on the face and for multiple lesions pg 235
what is the treatment of choice for txing actinic keratosis?
c. diarthrosis joint
what is the type of joint that is freely movable, such as the shoulder joint, called? a. synarthrosis joint b. amphiarthrosis joint c. diarthrosis joint d. juxtarthrosis joint
damage to the parietal lobe, thalamus, spinothalamic tract, or the spinal or peripheral nerves that are the usual pathways for transmission and interpretation of sensory stimuli pgs 81-82
what is the usual cause for paresthesia?
an infection of the beard area and is more common in men who work with animals pg 169
what is tinea barbae?
chronic idiopathic urticaria pg 156
what is urticaria that lasts longer than 6 weeks classified as?
total loss of skin color in patchy areas (rarely over the entire body) pg 148
what is vitiligo?
b. chest x-ray
what laboratory test could help differentiate acute bronchitis from pneumonia in a patient with a productive cough? a. CBC b. chest x-ray c. sputum specimen d. pulmonary function tests
serum creatinine greater than 4mg/dL
what laboratory value is associated with kidney disease?
Darbepoetin (Aranesp), weekly initially, than adjusted to every 2 week dosing Erythropoietin (Epogen, Procrit) is given 3xweekly, then adjusted pg 914
what medication can be given for anemia of chronic disease and how often?
beta blockers (pg 332)
what medication can cause a hypersensitive airway, wheezing, and cough?
Diamox pg 81
what medication decreases edema in the labyrinth?
meclizine, diphenhydramine or promethazine = they suppress the vestibular end organ receptors and inhibit activation of vagal responses pg 81
what medications are the most commonly prescribed for vertigo?
ibuprofen, aspirin pgs 911-912
what medications can cause occult blood in the stool?
a. the 4th and 5th fingers
what part of the body is affected by Dupuytren's contracture? a. the 4th and 5th fingers b. the great toe c. the tibia d. the penis
a. increased lactic acid production, muscle breakdown, and minor inflammation
what pathophysiology associated with transient pain after exercising usually begins a few hrs after exercise with soreness and may last up to a week? a. increased lactic acid production, muscle breakdown, and minor inflammation b. mild musculotendinous inflammation c. major musculotendinous inflammation, periostitis, and bone microtrauma d. breakdown in soft tissue and stress fx
streptococcus A pg 180
what predominant bacteria causes ecthyma?
c. upper lateral thigh pg 940
what site should you give the initial injection of epinephrine for tx of anaphylaxis? a. IV b. abdomen c. upper lateral thigh d. deltoid
there is no specific test to diagnosis MS pg 95
what specific test is used to diagnosis multiple sclerosis (MS)?
the Flexible fiberoptic bronchoscopy (FFB) pg 386)
what test is the most important procedure for determining the endobronchial extent of disease?
creatinine clearance (pg 620)
what test is the most useful in determining renal function in a patient suspected with chronic renal failure (CRF)?
a CBC and Hgb electrophoresis pg 913
what tests are required to determine thalassemia?
serum bilirubin, hepatic enzymes, BUN, creatinine, CBC w platelet count (pg 375)
what tests should be measured before instituting tx for TB?
pins and needles pg 81
what type of associated feelings are described in paresthesia?
c. allogeneic pg 937
what type of bone marrow transplant is obtained from a compatible donor such as a parent or sibling with a similar tissue type? a. autogenic b. autologous c. allogeneic d. syngeneic
b. autologous pg 936
what type of bone marrow transplant is obtained from a individuals own marrow before tx? a. autogenic b. autologous c. allogeneic d. syngeneic
d. syngeneic pg 936
what type of bone marrow transplant is obtained from an identical twin? a. autogenic b. autologous c. allogeneic d. syngeneic
c. her father had the disease and her mother was a carrier
when Jane tells you that she has hemophilia, you know that: a. both of her parents also have the disease b. her maternal grandfather probably had the disease and it skipped a generation c. her father had the disease and her mother was a carrier d. her mother had the disease
a. is an injury to the ligaments that attach to bone in a joint
when M., age 12 slid into home plate while playing baseball, he injured his ankle. you are trying to differentiate between a sprain and a strain. you know that a sprain: a. is an injury to the ligaments that attach to bone in a joint b. is an injury to the tendons that attach to the muscles in a joint c. is an injury resulting in extensive tears of the muscles d. does not result in joint instability
a. what would it take for you to consider quitting?
when a clt is in the precontemplation stage of smoking cessation, which question should the health-care provider ask? a. what would it take for you to consider quitting? b. what would it take for you to quit now? c. what technique do you think will work best for you? d. what do you think will be your biggest challenge to quitting?
c. natural passive
when a neonate is initially protected against measles, mumps, and rubella because the mother is immune, this is an example of which type of immunity? a. natural active b. artificial active c. natural passive d. artificial passive
b. emergency care pg 831
when a patient is exhibiting either Trousseau's or Chvostek's signs and accompanied by respiratory distress(stridor, loud crowing noises and cyanosis), what type of care do they need? a. home care b. emergency care c. ibuprofen d. rest
a. endometriosis
when a woman complains of dyspareunia in the lower back during orgasm, you should consider: a. endometriosis b. cystitis c. vaginitis d. causes r/t pelvic inflammatory disease
b. knee-chest position while prone
when a woman has extreme spasticity, which position should she assume for a Pap smear? a. OB stirrups position b. knee-chest position while prone c. v-shaped position without stirrups d. side-lying position
c. is a medium-weight to heavy-weight prescription product
when advising a woman with varicose veins about the use of support stockings, you consider that the preferred type: a. can be purchased in the hosiery section of a dept. store b. is a lightweight pair and available over-the-counter c. is a medium-weight to heavy-weight prescription product d. is used in the form of panty hose
d. all of the above pg 946
when analyzing synovial fluid, if it is transparent, xanthochromic, has 200-2,000 leukocytes/mL , <50% PMN and a low RBC count, it may indicate which of the following conditions? a. osteoarthritis b. scleroderma c. Polyarteritis nodosa d. all of the above
c. rheumatoid arthritis pg 946
when analyzing synovial fluid, it it is opaque, white, or translucent, has 5,000 leukocytes/mL, 80% PMN, and a lower RBC count, it may be indicative of which of the following conditions? a. normal result b. scleroderma c. rheumatoid arthritis d. sickle cell disease
a. normal result pg 946
when analyzing synovial fluid,, if it is clear, yellow, has <200 leukocytes/mL , <25% PMN and a low RBC count, it may indicate which of the following conditions? a. normal result b. scleroderma c. rheumatoid arthritis d. sickle cell disease
d. septic arthritis pg 946
when analyzing synovial fluid,, if it is opaque, white, has 50,000-300,000 leukocytes/mL , 50-100% PMN and a low RBC count, it may indicate which of the following conditions? a. normal result b. scleroderma c. rheumatoid arthritis d. septic arthritis
a. 3
when assessing a clt's deep tendon reflexes, you note they are more brisk than normal. You document them as: a. 3 b. 2 c. 1 d. 0
usually within the first year of life for those born with sickle cell anemia pg 923
when do the first symptoms of sickle cell anemia occur?
b. full ROM against gravity with some resistance
when grading muscle strength on a scale of 1-5, a grade of 4 indicates: a. full ROM against gravity with full resistance b. full ROM against gravity with some resistance c. full ROM with gravity d. full ROM with gravity eliminated (passive motion)
when it is accompanied by other breast abnormalities, especially if it is unilateral, a disc > than 4cm in diameter, does not resolve after two years after puberty, occurs in the presence of abnormal serum levels of free testosterone and luteinizing hormones or abnormal testis pg 831-832
when is a mammogram indicated in gynecomastia?
when there is documented failure of therapy with oral iron supplements pg 914
when is supplemental iron by parental route indicated?
a. induration
when measuring the TB skin test, are you measuring the: a. induration b. erythema c. induration and erythema
b. common in thin older adults
when palpating the skin over the clavicle of J., age 84, you notice tenting which is: a. indicative of dehydration b. common in thin older adults c. a sign of edema d. indicative of scleroderma
d. epispadias
when performing a newborn assessment of a male infant, you note that the urethral opening is on the dorsal side of the glans. this is referred to as: a. hypospadias b. Peyronie's disease c. priapism d. epispadias
d. bacterial prostatitis
when performing a prostate exam, you note a tender, warm prostate. what do you suspect? a. BPH b. prostatic abscess c. prostate cancer d. bacterial prostatitis
c. decrease in sphincter tone
when performing a rectal exam on the aging man, you may normally note: a. fissures b. small prostate c. decrease in sphincter tone d. longer anal canal
skin testing or radioallergosorbent tests (RASTs)/in vitro testing to determine sensitivity to allergens (pg 341)
when persistent asthma is present, the EPR-3 recommends:
a. antidepressants
when premenstrual syndrome sxs do not respond to dietary and nonmedical therapies, which of the following drugs might you try? a. antidepressants b. antihistamines c. corticosteroids d. anticholinergics
when the thyroid gland is hard and enlarged or when nodules are palpated pg 842
when should thyroid cancer be considered?
c. in the case of infection
when should you order a CBC for your clt? a. routinely b. before dental work c. in the case of infection d. if she is pregnant
a. cholestyramine
when take concomitantly with warfarin, which of the following causes a possibly decreased anticoagulant effect? a. cholestyramine b. allopurinol c. cefpodoxime d. zolpidem
a. clarithromycin
when taken with warfarin, which of the following causes a possible increased anticoagulant effect? a. clarithromycin b. carbamazepine c. pravastatin d. sucralfate
b. advance the cane with the ipsilateral leg
when teaching A., age 67 yo use of a cane because of osteoarthritis of her left knee, an important point to stress is to tell her to: a. carry the cane int he ipsilateral hand b. advance the cane with the ipsilateral leg c. make sure that the cane length equals the ht of the iliac crest d. use the cane to aid in joint protection and safety
d. make an appt if you note any hard lumps directly on the testicle, whether they are tender or not
when teaching T. how to do a testicular self-exam, which of the following do you tell him? a. examine your testicles when you are cold because them makes them more sensitive b. make sure your hands are dry to create friction c. if you feel firmness above and behind the testicle, make an appt d. make an appt if you note any hard lumps directly on the testicle, whether they are tender or not
a. isograft
when the donor and recipient of a transplant are identical twins, this is referred to as a(n): a. isograft b. autograft c. allograft d. xenograft
yes (pg 372)
when treating TB, are LFTs necessary?
a. tennis elbow
when wrist and finger extension causes pain over the extensor carpi radialis brevis tendon, the extensor carpi radialis longus tendon, and the extensor digitorum communis, you would suspect: a. tennis elbow b. golfer's elbow c. deQuervain's disease d. intersection syndrome
d. the proprioceptive system
when you ask a clt to walk a straight line placing heel to toe, you are assessing: a. sensory function b. cerebellar function c. cranial nerve function d. the proprioceptive system
c. deQuervain's tenosynovitis
when you elicit a painful Finkelstein's sign, you are testing for: a. carpal tunnel syndrome b. bursitis of the shoulder c. deQuervain's tenosynovitis d. tennis elbow
c. doll's eyes
when you move a clt's head to the left and the eyes move to the right in relation to the head, this is referred to as: a. extraocular eye movements b. oculomotor degeneration c. doll's eyes d. decerebrate posturing
a. stereognosis
when you place a key in the hand of a clt whose eyes are closed and ask him to identify the object, what are you assessing? a. stereognosis b. graphesthesia c. two-point discrimination d. position sense
the face (especially the cheeks) and lower legs pg 191
where are the most common sites for erysipelas?
the pituitary pg 148
where does adrenocorticotropic hormone (ACTH) come from?
the liver (pg 350)
where in the body is the protein, alpha-antitrypsin (ATT) made?
d. in the inner condyle of the humerus
where is the epitroclear lymph node located? a. in front of the ear b. halfway between the angle and the tip of the mandible c. in the posterior triangle along the edge of the trapezius muscle d. in the inner condyle of the humerus
in the hospital with parenteral antibiotics pg 193
where should erysipelas be treated at, home or the hospital?
c. chancroid and genital herpes
which STDs are cofactors for HIV transmission? a. syphilis and chlamydia b. herpes and chlamydia c. chancroid and genital herpes d. chancroid and gonorrhea
a. topical antifungals
which agent is ineffective against psoriasis? a. topical antifungals b. systemic medications c. phototherapy d. topical corticosteroids
PTU, because it is less likely to cross the placenta pg 843
which antithyroid medication would you use in a pregnant women and why?
c. FAQ
which appropriate test for the initial assessment of Alzheimer's disease provides performance rating on 10 complex, higher-order activities? a. MMSE b. CAGE questionnaire c. FAQ d. Homes and Rahe Social Readjustment Scale
b. Breslow's
which assessment system for a malignant melanoma is used to determine the thickness of a lesion and help in determining prognosis? a. Clark's b. Breslow's c. Brown's d. Dermatoscope's
c. Glasgow Coma Scale
which assessment tool rates the level of consciousness by assigning a numerical score to the behavioral components of eye opening, verbal response, and motor response? a. MMSE b. Brudzinski sign c. Glasgow Coma Scale d. CAGE questionnaire
Streptococcus pneumonia (pg 336)
which bacteria is the most common cause of community acquired pneumonia?
b. CA 125
which blood tumor marker is highly specific to epithelial ovarian cancer? a. PSA b. CA 125 c. CA 15-3 d. CA 19-9
b. chondrosarcoma
which bone tumor arises from cartilage and is usually located in the pelvis, femur, proximal humerus, or ribs? a. osteosarcoma b. chondrosarcoma c. Ewing's sarcoma d. fibrosarcoma
lower spine and weightbearing joints (pg 370)
which bones are most often affected with skeletal TB?
d. testicular cancer
which cancer can be cured with chemotherapy alone? a. breast cancer b. malignant melanoma c. bladder cancer d. testicular cancer
a. CD8+T
which cells are predominately present in COPD? a. CD8+T b. CD4=Thelper
ACEi
which class of hypertensive drugs are contraindicated in clts with renal artery stenosis?
b. Jeff, age 11, who recently had an orchiectomy after a traumatic accident
which clt will most likely never develop prostate cancer? a. J., age 79 who had a transurethral resection of the prostate for BPH b. Jeff, age 11, who recently had an orchiectomy after a traumatic accident c. Sid, age 70 who has a normal prostate specific antigen level d. John, age 32, who is taking steroids for bodybuilding
d. renal cysts
which condition does not cause flank pain? a. pyelonephritis b. uretorolithiasis c. vascular occlusion of the kidney (renal vein thrombosis) d. renal cysts
urinary calcium output of > than 300mg/24hr
which diagnostic finding(s) may lead to a dx of nephrolithiasis?
Serum hemoglobin and hematocrit
which diagnostic studies for the evaluation of renal function assesses for the lack of erythropoietin?
a. erythema infectiosum (fifth disease)
which disease usually starts on the cheeks and spreads to the arms and trunk? a. erythema infectiosum (fifth disease) b. rocky mountain spotted fever c. rubeola d. rubella
furosemide (lasix)
which diuretic acts by inhibiting sodium chloride reabsorption in the thick ascending limb of the loop of Henle?
c. leukotriene blockers
which drug class is never used to tx COPD? a. long-acting bronchodilator b. long-acting anticholinergic c. leukotriene blockers d. systemic steroids
c. varenicline (chantix)
which drug has been shown to be an effective aid to smoking cessation? a. oxazepam (serax) b. clorazepate dipotassium (tranxene) c. varenicline (chantix) d. alprazolam (xanax)
d. ramipril
which drug may be associated with a cough? a. dextromethorphan b. guaifenesin c. albuterol d. ramipril
c. finasteride (Proscar)
which drug reduces the size of the prostate, reduces the risk of urinary retention by increasing urinary flow rate, and reduces some fo the sxs of BPH? a. doxazosin (cardura) b. prozosin (minipress) c. finasteride (Proscar) d. terazosin (hytrin)
d. african americans
which ethnic grp has the highest overall cancer incidence rate? a. native americans b. asian and pacificislanders c. hispanics d. african americans
African Americans (pg 344
which ethnicity has a negative rx to long-term-beta-blockers?
b. acne rosacea
which form of acne is more common in the middle-aged to older adult and causes changes in skin color, enlarged pores, and thickening of the soft tissues of the nose? a. acne vulagaris b. acne rosacea c. acne conglobata d. nodulocystic acne
b. estradiol (E2)
which form of estrogen is secreted in the greatest amount by the ovaries during the reproductive yrs and considered most potent? a. estrone (E1) b. estradiol (E2) c. estriol (E3) d. the potency and secretion of all of the above are in equal amts
males pg 86
which gender is more likely to have a seizure?
a. Bartholin's glands
which glands are posterior on each side of the vaginal orifice and open onto the sides of the vestibule int he groove between the labia minora and hymen? a. Bartholin's glands b. Skene's glands c. paraurethral glands d. cystocele
a. white men
which groups are most prone to develop nephrolithiasis? a. white men b. white women c. black men d. back women
African Americans
which grp of persons experience more rapid age-related decreases in glomerular filtration rate than do Caucasians?
c. naproxen sodium (anaprox DS)
which headache preparation has GI distress as a side effect? a. sumatriptain (imitrex) b. nadolol (corgard) c. naproxen sodium (anaprox DS) d. ergot preparations (cafergot)
c. immune complex-mediated reaction
which hypersensitivity reactions results in a skin test that is erythematous with edema within 3-8 hrs? a. anaphylactic reaction b. cytotoxic reaction c. immune complex-mediated reaction d. delayed hypersensitivity reaction
d. pityrosporum orbiculare pg 169
which is NOT one of the three fungal species that predominantly cause superficial skin infections? a. trichophyton b. ephidermophyton c. microsporum d. pityrosporum orbiculare
a hematoma pg 907
which is larger? a hematoma or bruise
c. ferritin
which is the best serum test to perform to spot an iron-deficiency anemia early before it progresses to flull-blown anemia? a. hemoglobin b. hematocrit c. ferritin d. reticulocytes
b. oral griseofulvin (grisactin)
which is the drug of choice for tinea capitis? a. a topical corticosteroid b. oral griseofulvin (grisactin) c. a topic antifungal d. an antibiotic
a. IgG
which is the most abundant immunoglobulin (Ig) found in the blood, lymph, and intestines? a. IgG b. IgA c. IgM d. IgD
c. psoriasis
which lesion results in scales or shedding flakes of greasy, keratinized skin tissue? a. eczema b. impetigop c. psoriasis d. herpes
b. long-acting bronchodilator
which medication below is contraindicated for one use in treating asthma? a. short-acting brochodilator b. long-acting bronchodilator c. inhaled steroid d. oral steroid
a. cough suppressant with codeine
which medication below would be contraindicated in a pt with COPD? a. cough suppressant with codeine b. dextromethorphan c. mucolytics d. decongestant with guaifenesin
a. bethanechol chloride (Urecholine)
which medication is used primarily in the tx of acute postoperative and postpartum urinary retention and for neurogenic bladder atony with urinary retention? a. bethanechol chloride (Urecholine) b. neostigmine (Prostigmin) c. Propantheline bromide (Pro-Banthine) d. Oxybutynin (ditropan)
d. atypical antipsychotics like risperidone (risperdal)
which medication should be avoided in clts with Alzheimers disease who have concurrent vascular dementia or vascular risk factors? a. acetycholinesterase inhibitors like donepezil (aricept) b. N-methyl-D-aspartate (NMDS) receptor c. anxiolytics like bupirone (buspar) d. atypical antipsychotics like risperidone (risperdal)
c. creatine kinase
which muscle enzyme is elevated in polymyositis? a. aldolase A b. aspartate aminotransferase c. creatine kinase d. lactate dehydrogenase
NSAIDs
which nephrotoxic agent should be avoided in clts with CRF?
c. depakote
which of the antiepileptic drugs are associated with spina bifida? a. dilantin b. lamictal c. depakote d. keppra
b. tinea versicolor - caused by pityrosporum orbiculare (malassezia furfur) pg 169
which of the below is caused by a yeast and not a dermatophytose or tinea? a. tinea capitis b. tinea versicolor c. tinea manuum d. tinea corporis
b. pregnancy
which of the following are predisposing factors for pyelonephritis? a. dehydration b. pregnancy c. alkaline urine d. smoking
a. repetitive nerve stimulation
which of the following can assist in the dx of myasthenia gravis? a. repetitive nerve stimulation b. the presence of cogwheel rigidity c. Chvostek's sign d. Trousseaus' sign
c. adhesions from infection
which of the following can cause phimosis? a. paraphimosis b. smegma c. adhesions from infection d. priapism
a. Burkitt's lymphoma
which of the following cancers is associated with Epstein-Barr virus? a. Burkitt's lymphoma b. Kaposi's sarcoma c. lymphoma d. adult T-cell leukemia
a. beta blockers
which of the following cardiac drugs is used to tx migraine headaches? a. beta blockers b. nitrates c. ACEIs d. alpha-adrenergic blockers
b. obstructed airways
which of the following characteristics is always present in a pt with COPD? a. productive cough b. obstructed airways c. SOB d. hypercapnia
b. pneumonia
which of the following complications is the leading cause of death shortly after a stroke? a. septicemia b. pneumonia c. pulmonary embolus d. ischemic heart disease
d. heart disease
which of the following conditions is a contraindication to using the copper IUD? a. hx of ectopic pregnancy b. nulliparity c. tx cervical dysplasia d. heart disease
b. fetal alcohol syndrome
which of the following conditions is most responsible for developmental delays in children? a. cerebral palsy b. fetal alcohol syndrome c. down syndrome d. meningomyelocele
a. cytoscopy with biopsy (pg 610)
which of the following diagnostic tests should be ordered for a patient suspected of bladder cancer? a. cytoscopy with biopsy b. KUB (kidneys, ureter, bladder) xray c. MRI d. urine tumor marker
a. methotrexate (Rheumatrex) pg 949
which of the following disease-modifying antirheumatic drugs is a folic acid antagonist? a. methotrexate (Rheumatrex) b. Etanercept (Enbrel) c. rituximab (rituxan) d. Anakinra (Kineret)
d. Anakinra (Kineret) pg 949
which of the following disease-modifying antirheumatic drugs is a interleukin-1 receptor antagonist? a. methotrexate (Rheumatrex) b. Etanercept (Enbrel) c. Rituximab (Rituxan) d. Anakinra (Kineret)
c. Rituximab (Rituxan) pg 949
which of the following disease-modifying antirheumatic drugs is a monoclonal antibody? a. methotrexate (Rheumatrex) b. Etanercept (Enbrel) c. Rituximab (Rituxan) d. Anakinra (Kineret)
b. Etanercept (Enbrel) pg 949
which of the following disease-modifying antirheumatic drugs is a tumor necrosis factor blocker? a. methotrexate (Rheumatrex) b. Etanercept (Enbrel) c. Rituximab (Rituxan) d. Anakinra (Kineret)
d. Raynaud disease
which of the following does not directly contribute to the development of varicose veins? a. leg crossing b. pregnancy c. heredity d. Raynaud disease
c. oral contraceptives
which of the following drugs given to nursing mothers may cause a reduction in the milk supply? a. antihistamines b. antithyroid medication c. oral contraceptives d. laxatives
d. phenazopyridine (pyridium)
which of the following drugs is an analgesic to tx dysuria? a. hyoscyamine (levsin) b. oxybutyin (ditropan) c. propantheline (pro-banthine) d. phenazopyridine (pyridium)
d. Erythromycin (e-mycin)
which of the following drugs is not associated with acute renal failure? a. ACEi b. cimetidine (tagamet) c. NSAIDs d. Erythromycin (e-mycin)
c. antibiotics
which of the following drugs may diminish the effectiveness of oral contraceptives? a. beta blockers b. oral anticoagulants c. antibiotics d. oral hypoglycemis
b. oral anticoagulants
which of the following drugs may have their effects diminished when used in combination with an oral contraceptive? a. corticosteroids b. oral anticoagulants c. antibiotics d. anticonvulsants
a. beta blockers
which of the following drugs may have their effects enhanced when used in combination with an oral contraceptive? a. beta blockers b. oral anticoagulants c. antacids d. anti-convulsants
c. bromocriptine
which of the following drugs used for parkinsonism mimics dopamine? a. anticholinergics b. levodopa c. bromocriptine d. tolcapone
d. ausculatory crackles (pg 619)
which of the following exam findings should be expected in a pt with chronic renal failure? a. weak, thready pulse b. hypotension c. pleural friction rub d. ausculatory crackles
a. gait of sensory ataxia
which of the following gaits in older adults includes brusqueness of the movements of the leg and stamping of the feet? a. gait of sensory ataxia b. Parkinsonian gait c. antalgic gait d. cerebellar gait
b. eosinophils
which of the following in the urinary sediment signifies an allergic reaction in the kidney? a. leukocytes b. eosinophils c. crystals d. erythrocytes
b. cigarette smoking, diabetes, and a high fat diet
which of the following increases the risk of pancreatic cancer? a. high carbohydrate diet b. cigarette smoking, diabetes, and a high fat diet c. diabetes and lack of activity d. yo-yo dieting
c. removable white plaques
which of the following indicates that John, a 32 y/o clt with AIDS, has oropharyngeal candidiasis? a. small vesicles b. fissured, white, thickened patches c. removable white plaques d. flat topped papules with thin, bluish-white spiderweb lines
a. a simple exercise program
which of the following interventions can significantly slow the decline in performing activities of daily living (ADLs) in clts with Alzheimer's disease living in a nursing home? a. a simple exercise program b. ginkgo biloba c. doing crossword puzzles d. improving nutritional state
d. obesity
which of the following is NOT part of the differential for a pt who complains of cough? a. heart failure b. reflux disease c. asthma d. obesity
a. leiomyoma
which of the following is a benign neoplasm? a. leiomyoma b. osteosarcoma c. glioma d. seminoma
b. chemotherapy drugs
which of the following is a genotoxic carcinogen? a. vinyl chloride polymers b. chemotherapy drugs c. asbestos d. wood and leather dust
c. smoking
which of the following is a modifiable risk factor for osteoporosis? a. low alcohol intake b. low caffeine intake c. smoking d. excessive exercise
a. diabetes mellitus
which of the following is a predisposing condition for furunculosis? a. diabetes mellitus b. HTN c. peripheral vascular disease d. chronic fatigue sundrome
c. seborrheic dermatitis
which of the following is a secondary skin lesion? a. acne nodule b. neoplasm c. seborrheic dermatitis d. herpes simplex
b. trichomonal vaginitis
which of the following is a sexually transmitted infection? a. candida vaginitis b. trichomonal vaginitis c. atrophic vaginitis d. lactobacilli vaginitis
b. glucose-6-phosphate dehydrogenase deficiency
which of the following is an X-linked recessive disorder commonly seen in African American men? a. sickle cell anemia b. glucose-6-phosphate dehydrogenase deficiency c. pyruvate kinase deficiency d. Bernard-Soulier syndrome
c. do you have urges to urinate that sometimes result in wetting accidents?
which of the following is an important question that you should ask to assess for urge urinary incontinence in men? a. do you frequently have strong urges to urinate? b. do you urinate more than you think you should? c. do you have urges to urinate that sometimes result in wetting accidents? d. are you bothered by waking up at night to go to the bathroom?
a. pap smear showing dysplasia
which of the following is an indication for a colposcopy? a. pap smear showing dysplasia b. recurrent STIs c. HIV infection d. hx of leiomyomas
b. itching
which of the following is an unusual side effect of tricyclic antidepressants? a. dry mouth b. itching c. constipation d. drowsiness
d. grandiose delusions
which of the following is characteristic of a manic episode? a. wt loss or gain b. insomnia or hypersomnia c. diminished ability to think or concentrate d. grandiose delusions
b. metabolic alkalosis
which of the following is defined as a pathophysiological process that occurs when there is a primary excess in the extracellular fluid of bicarbonate (HCO3) as a result of a loss of acid or the addition of excess bicarbonate? a. metabolic acidosis b. metabolic alkalosis c. respiratory acidosis d. respiratory alkalosis
d. Pica pg 913
which of the following is identified as an eating disorder of craving for food substitutes such as clay, ice chips, and cotton and is considered an OBJECTIVE finding associated with severe iron deficiency? a. ferritin b. Porter's syndrome c. hypochromasia d. Pica
d. hemoptysis
which of the following is least likely to be found in pts with pulmonary embolus (PE)? a. pleuritic chest pain b. tachypnea c. DVT s/s d. hemoptysis
d. isometric exercise
which of the following is not a contributing factor to development of venous thrombophlebitis? a. venous status b. injury to vascular intima c. malignancy-associated hypercoagulation states d. isometric exercise
a. decreasing the amt of liquids
which of the following is not an effective strategy to prevent the nausea and vomiting associated with the effects of radiation and chemotherapy a. decreasing the amt of liquids b. eating a soft, bland diet low in fat and sugar c. relaxation d. distraction
c. a period of intense itchiness after blanching
which of the following is the most common presentation in a patient with Raynaud phenomenon? a. digital ulceration b. worsening of sxs in warm weather c. a period of intense itchiness after blanching d. unilateral sxs
a. unilateral leg edema
which of the following is the most likely to be found in deep vein thrombophlebitis (DVT)? a. unilateral leg edema b. leg pain c. warmth over the affected area d. positive obturator sign
c. cigarette smoking
which of the following is the most potent risk factor for lower-extremity vascular occlusive disease? a. hypertension b. older age c. cigarette smoking d. leg injury
b. protamine sulfate
which of the following is the preferred medication to reverse the anticoagulant effects of unfractionated heparin? a. vit K b. protamine sulfate c. platelet transfusion d. plasma components
a. vit K
which of the following is the preferred medication to reverse the anticoagulation effects of warfarin? a. vit K b. protamine sulfate c. platelet transfusion d. plasma components
a. functional scoliosis is flexible; it is apparent with standing and disappears with forward bending
which of the following is true regarding scoliosis? a. functional scoliosis is flexible; it is apparent with standing and disappears with forward bending b. functional scoliosis is fixed; the curvature shows both on standing and bending forward c. structural scoliosis is fixed; the curvature shows both on standing and bending forward d. functional scoliosis is permanent, whereas structural scoliosis can result from outside influences such as leg length discrepancy or muscle spasms
a. CSF glucose of 35mg/dL
which of the following lab results would indicate a specific infection in the central nervous system? a. CSF glucose of 35mg/dL b. CSF pressure of 250mm of water c. CSF RBC count of 25/mm3 d. serum white blood cell count of 12,000/mm3
c. antibody testing
which of the following lab studies is used to determine if a clt has had hepatitis? a. serum protein b. protein electrophoresis c. antibody testing d. globulin levels
c. smoking
which of the following lifestyle factors is associated with an increased risk for breast cancer? a. being underweight b. having one to two drinks of alcohol per day c. smoking d. eating a low-fat diet
c. Pulmonary function tests
which of the following may be used to dx COPD? a. chest x-ray b. CT scan of the chest c. Pulmonary function tests d. arterial blood gases
d. hypertension
which of the following may immediately follow a stroke in the older adult and be the body's attempt to maintain perfusion? a. bradycardia b. tachycardia c. hypotension d. hypertension
a. timolol opthalmic drops
which of the following medications should be avoided in a pt who has asthma? a. timolol opthalmic drops b. naproxen c. lisinopril d. amlodipine
d. antihistamines
which of the following meds causes retention of urine by inhibiting bladder contractibility and may cause overflow incontinence in certain individuals? a. antispasmodics b. drugs that affect the sympathetic nervous system c. diuretics d. antihistamines
b. Jerry, age 30 who is a cross-country runner
which of the following men is more prone to prostatitis? a. Jim, age 52, who wears tight jeans and sits at a computer all day b. Jerry, age 30 who is a cross-country runner c. Marvin, age 46, who is a dog trainer d. Justin, age 39 who is a boat salesman
c. perineal sensation
which of the following objective data are associated with significantly better long-term outcomes in children born with open spina bifida? a. a higher APGAR score b. presence of Babinski's reflex c. perineal sensation d. a higher score on the Glascow Coma Scale
d. secondary ovarian tumors
which of the following ovarian tumors or cysts has the potential for malignancy? a. follicle cysts b. Brenner's tumor c. fibroma d. secondary ovarian tumors
b. prostaglandins
which of the following renal hormones regulates intrarenal blood flow by vasodilation or vasoconstriction? a. renin b. prostaglandins c. bradykinins d. erythropoietin
c. smoking
which of the following risk factors for a stroke can be eliminated? a. hypertension b. carotid artery stenosis c. smoking d. hyperlipidemia
a. the CAGE questionnaire
which of the following screening instruments is quick and easy to use and has a high level of dx accuracy to detect alcohol abuse? a. the CAGE questionnaire b. the HEAT instrument c. the DRINK tool d. MMSE
b. testicular torsion
which of the following scrotal d/o is most common in adolescents? a. acute epididymitis b. testicular torsion c. atrophic testes d. scrotal edema
c. lichenification
which of the following secondary skin lesions usually results from chronic scratching or rubbing? a. crusts b. scales c. lichenification d. atrophy
a. sunscreen
which of the following should be used with all acne medications? a. sunscreen b. oily makeup c. plain soap d. nothing should be used with acne medication
c. prodromal tingling or pruritus of the genital region
which of the following signs and/or sxs of a genital herpes infection usually occurs first? a. painful or pruritic vesicles b. dysuria c. prodromal tingling or pruritus of the genital region d. white, curdlike plaques on a red base in the vagina
c. taking the infant to an outdoor event
which of the following situations might precipitate a sickle cell crisis in an infant? a. taking the infant to visit a relative b. hepatitis B immunization c. taking the infant to an outdoor event d. having the infant sleep on its back
b. due to differences in race or gender, select grps may have 23 or 25 (cervical, thoracic, lumbar)
which of the following statement is true regarding vertebra? a. all people have only 24 vertebra, (cervical, thoracic, lumbar) b. due to differences in race or gender, select grps may have 23 or 25 (cervical, thoracic, lumbar) c. it is common to have few than 23 vertebrae (cervical, thoracic, lumbar) d. it is common to have few than 25 vertebrae (cervical, thoracic, lumbar)
a. it is often associated with being the first born female child
which of the following statements concerning developmental dysplasia of the hip (DDH) is correct? a. it is often associated with being the first born female child b. it results from an orthopedic malformation in utero c. it has no genetic predisposition d. it is more common in males
a. ovulation occurs on the 14th day, plus or minus 2 days, before the next menses
which of the following statements do you use when instructing women about their fertile period (when they are most likely to become pregnant)? a. ovulation occurs on the 14th day, plus or minus 2 days, before the next menses b. sperm are viable for 24 hrs c. the ovum is viable for 6 hrs d. the ovaries always release one ovum per month
c. symptoms are sometimes reported with minimally affected vessels
which of the following statements is most accurate in the assessment of a pt with varicose veins? a. the degree of venous tortuosity is well-correlated with the amt of leg pain reported b. as the number of affected veins increases, so does the degree of pt discomfort c. symptoms are sometimes reported with minimally affected vessels d. lower-extremity edema is usually seen only with severe disease
b. they have a more rapid disease progression when compared with their younger cohorts
which of the following statements is true about older men with HIV? a. they have a slower disease progression when compared with their younger cohorts b. they have a more rapid disease progression when compared with their younger cohorts c. they are rarely if ever injection drug users d. they cannot undergo txment with antiretrovirals
d. active and passive ROM of a joint should be equal, full, and pain free
which of the following statements is true regarding ROM of a joint? a. the normal active ROM of a joint is greater than the passive ROM to avoid further injury to the joint b. if there is a limitation of active ROM, you should not attempt passive ROM to avoid further injury to the joint c. active and passive ROM of a joint should be equal, full, and cause only mild discomfort d. active and passive ROM of a joint should be equal, full, and pain free
a. inability to concentrate, with psychomotor agitation or retardation
which of the following sxs related to memory indicates depression rather than delirium or dementia in the older adult? a. inability to concentrate, with psychomotor agitation or retardation b. impaired memory, especially of recent events c. inability to learn new material d. difficulty with long-term memory
d. Chadwick's sign
which of the following terms describes the bluish or purplish discoloration of the vulva, vagina, and a portion of the cervix that occurs in pregnancy? a. Goodell's sign b. Hegar's sign c. Piskacek's sign d. Chadwick's sign
a. Allen test
which of the following tests assesses the patency of the radial and ulnar arteries? a. Allen test b. Finkelstein's test c. Phalen's test d. Tinel's sign
c. serum acetylcholine receptor antibody level
which of the following tests is highly specific and fairly sensitive for myasthenia gravis? a. electromyography nerve conduction tests b. magnetic resonance imaging scan of the bran c. serum acetylcholine receptor antibody level d. lumbar puncture
c. ultraviolet light
which of the following therapeutic modalities is useful for severe uncontrollable atopic dermatitis? a. emollients b. compresses c. ultraviolet light d. tars
b. necrotizing fasciitis
which of the following types of cellulitis is referred to as "flesh eating bacteria"? a. erysipelas b. necrotizing fasciitis c. periorbital cellulitis d. peripheral vascular cellulitis
c. condyloma acuminata
which of the following warts (human papillomavirus (HPV) looks like a caulifower and is usually found in the anogenital region? a. plantar warts b. filiform and digitate warts c. condyloma acuminata d. verruca plana
a. neutrophils
which of the following white blood cell types is elevated in acute infections, the stress response, myelocytic leukemia and inflammatory or metabolic disorders? a. neutrophils b. eosinophils c. basophils d. monocytes
d. monocytes
which of the following white blood cell types is elevated in in chronic inflammatory disorder, TB, viral infections, leukemia, Hodgkin's disease and multiple myeloma? a. neutrophils b. eosinophils c. basophils d. monocytes
c. basophils
which of the following white blood cell types is elevated in in hypersensitivity responses, chronic myelogenous leukemia, chickenpox or smallpox, after a splenectomy, and in hypothyroidism? a. neutrophils b. eosinophils c. basophils d. monocytes
b. eosinophils
which of the following white blood cell types is elevated in parasitic infections, hypersensitivity reactions and autoimmune disorders? a. neutrophils b. eosinophils c. basophils d. monocytes
b. hypertrophic cardiomyopathy
which of the following would be considered a common cause of sudden death in athletes young than age 30? a. bronchospasm from exercise-induced asthma b. hypertrophic cardiomyopathy c. compartment syndrome d. meningitis
a. transillumination of the suspected mass
which of the following would enable you to r/o a dx of testicular cancer when examining a clt? a. transillumination of the suspected mass b. white race c. Scandinavian background d. hx of cryptorchidism
d. the prognosis is directly r/t to the thickness of the lesions
which of the statements about malignant melanomas is true? a. they usually occur in older adult males b. the clt has no family hx of melanoma c. they are common in blacks d. the prognosis is directly r/t to the thickness of the lesions
b. acute tubular necrosis
which of these are the most common cause of acute renal failure? a. renal calculi b. acute tubular necrosis c. cardiac failure d. acute glomerulonephritis
a. mycoplasma pneumoniae
which organism causes "walking pneumonia? a. mycoplasma pneumoniae b. streptococcus pneumoniae c. chlamydophila pneumoniae d. staphylococcus aureus
c. guillain-barre syndrome
which peripheral nervous system disorder usually follows a viral respiratory or GI infection? a. cytomegalovirus b. herpes zoster c. guillain-barre syndrome d. trigeminal neuralgia
b. a 65 y/o with emphysema
which pt might be expected to have the worst FEV1? a. an asthma pt in the green zone b. a 65 y/o with emphysema c. a 60 y/o with pneumonia d. a pt with bronchiolitis
ultrasonography (US)
which renal exam identifies the size of the kidneys or obstruction in the kidneys or the lower urinary tract and may detect tumors or cysts?
lindane pg 159
which scabicide is the most toxic?
b. absence seizures
which seizure type involves alteration of consciousness? a. myoclonic seizures b. absence seizures c. clonic seizures d. atonic seizures
d. acne rosacea
which skin lesion is morphologically classified as pustular? a. wart b. impetigo c. herpes simplex d. acne rosacea
c. actinic keratoses
which skin lesions are directly related to chronic sun exposure and photodamage? a. skin tags b. seborrheic kertatoses c. actinic keratoses d. angiomas
c. the clt has low self-esteem
which statement is accurate regarding a clt who is at highest risk for an eating disorder? a. the clt is male b. the clt is usually 25-35 years of age c. the clt has low self-esteem d. the clt has a bipolar personality
a. post-exposure prophylaxis for a health care worker should be started right away pg 973
which statement is most accurate about HIV post-exposure prophylaxis for a health care worker who was stuck with a needle? a. post-exposure prophylaxis for a health care worker should be started right away b. post-exposure prophylaxis for a health care worker should be 72 hrs after the stick with the needle c. renal and hepatitis function tests should be done 6 wks after beginning post-exposure prophylaxis tx d. post-exposure prophylaxis will prevent HVC
d. prostatodynia results in the same sxs as prostatitis, but there is no evidence of infection
which statement is true about prostatitis and prostatodynia? a. the terms are interchangeable b. prostatodynia may be acute, chronic, or nonbacterial c. the xs of both are extremely irritating d. prostatodynia results in the same sxs as prostatitis, but there is no evidence of infection
b. they do not lower blood pressure in normotensive clts
which statement is true about the use of alpha blockers in the txment of symptomatic benign prostatic hypertrophy? a. they are safe and effective and should be given in the am before breakfast b. they do not lower blood pressure in normotensive clts c. pedal edema is the most common adverse effect d. blood counts should be monitored periodically for reduction in the platelet count
struvite stones, occure when the urine alkaline is greater than pH 7.0 and a urea splitting organism such as Proteus or Klebsiella is present (pg 603)
which stone are found primarily in women and are associated with UTIs?
calcium oxalate and phosphate stones account for 65 to 85% of all cases of renal calculi (pg 603)
which stone is found predominately in men and in individuals whose die tis high in salt, animal fat, animal protein, and oxalate from green leafy vegetables?
a. epidermis
which structure of the skin is responsible for storing melanin? a. epidermis b. dermis c. sebaceous glands d. eccrine sweat glands
b. Kegel exercises
which technique uses a learned method to target muscle contraction and relaxation to assist with urinary continence? a. biofeedback b. Kegel exercises c. bladder training d. prompted voiding
d. Roos test
which test assesses for thoracic outlet syndrome by having the clt abduct his/her arms 90 degrees externally rotated with the elbows flexed 90 degrees, and then having the clt open and close his /her hands for 3 minutes? a. Neer test b. Speeds test c. Hawkins test d. Roos test
d. a Snellen test
which test is routinely recommended for a preparticpation sport physical? a. complete blood count b. a chest x-ray c. EKG d. a Snellen test
c.Thompson test
which test is used to dx an Archilles tendon rupture? a. boutonniere test b. Lachman test c.Thompson test d. Drawer test
The Allen test
which test should you perform prior to drawing ABGs?
a. alpha fetoprotein
which tumor marker may detect a tumor of the ovary or testis? a. alpha fetoprotein b. carcinoembronic antigen c. human chorionic gonadotropin d. cancer antigen 125
d. Mohs' micrographic surgery
which tx is considered the gold standard in tissue-conservative skin cancer removal? a. cryosurgery b. simple excision c. photodynamic tx d. Mohs' micrographic surgery
c. a high-fiber diet for constipation
which tx would you order for anogenital pruritus? a. suppositories for pain b. antifungal cream for itching c. a high-fiber diet for constipation d. zinc-oxide ointment
b. Paget's disease
which type of breast cancer involves infiltration of the nipple epithelium and has an initial sx of itching or burning of the nipple? a. ductal cancer b. Paget's disease c. mammaryd uct ectasia d. fibroadenoma
b. Bartholin's cyst
which type of cyst fo the female reproductive system usually results in pain, redness, a perineal mass, and dyspareunia? a. ovarian cyst b. Bartholin's cyst c. Gardner's cyst d. Nabothian cyst
c. potassium-sparing diuretic
which type of diuretic has the following side effects: hyperkalemia, headache, hyponatremia, nausea, diarrhea, urticaria, and menstrual disturbances? a. osmotic diuretic b. loop diuretic c. potassium-sparing diuretic d. thiazide diuretic
c. viral
which type of encephalitis is the most common? a. microbial b. herpes simplex virus c. viral d. pneumococcal
b. Stork's beak mark
which type of hemangioma in a newborn occurs on the nape of the neck and is usually not noticeable when it becomes covered by hair? a. nevus flammeus (port-wine stain) b. Stork's beak mark c. strawbetty hemangioma d. cavernous hemangioma
painless, gross hematuria
which type of hematuria in an older adult is an ominous sign?
a. stress incontinence
which type of incontinence has an associated sx of recurrent cystitis? a. stress incontinence b. urge incontinence c. overflow incontinence d. function incontinence
b. subdural
which type of intracranial hematoma is the most common and has the clinical manifestations of headache, drowsiness, agitation, slowed thinking, and confusion? a. epidural b. subdural c. intracerebral d. meningeal
calcium oxalate
which type of kidney stone is most common?
c. chronic lymphocytic leukemia
which type of leukemia produces sxs with an insidious onset including weakness, fatigue, massive lymphadenopathy, pruritic vesicular skin lesions, anemia, and thrombocytopenia? a. acute lymphocytic leukemia b. acute myelogenous leukemia c. chronic lymphocytic leukemia d. chronic myelogenous leukemia
a. aseptic
which type of meningitis is a more benign, self-limited syndrome caused primarily by viruses? a. aseptic b. bacterial c. chronic d. inflammatory
pneumocystis jirovecii (formerly carinii) pneumonia (pg 358)
which type of pneumonia is the leading cause of death in patients with AIDS?
c. urge incontinence
which type of urinary incontinence results from Parkinson's disease and multiple sclerosis? a. overflow incontinence b. stress incontinence c. urge incontinence d. functional incontinence
c. filiform warts
which type of wart has fingerlike, flesh colored projections emanating from a narrow or broad base? a. common warts b. flat warts (verruca plana) c. filiform warts d. plantar warts
a. Epstein-Barr pg 950
which virus is more commonly found in infectious mononucleosis? a. Epstein-Barr b. cytomegalovirus
b. lack of circumcision
while cystitis is more commonly seen in women, there are specific risk factors for UTI in males, including which of the following? a. hypospadias b. lack of circumcision c. high sperm count d. varicocele
children older than 5 years and young adults are at greatest risk, outbreaks can occur in populations living in close proximity (pg 360)
who are most likely affected by mycoplasma pneumonia?
b. Bill, who runs every day and takes excessive amts of vitamin C
who is at higher risk for developing nephrolithiasis? a. Jean, who exercises every day and drinks copious amts of water b. Bill, who runs every day and takes excessive amts of vitamin C c. MaryAnn, who watches her weight and eats a low-sodium diet d. Harvey, a couch potatoe who drinks a lot of no-sodium soda
a. Joey, age 2 who is dehydrated from gastroenteritis
who is at risk of developing a prerenal type of acute renal failure? a. Joey, age 2 who is dehydrated from gastroenteritis b. tommy, age 3 who accidentally took an overdose of tylenol c. Justine, age 5 who nearly drowned in a swimming pool d. Buddy, age 12 who was born with one kidney and just injured the other in a football game
d. 18 y/o woman
who is most likely to have new onset primary Raynaud phenomenon? a. 68 y/o man b. 65 y/o woman c. 25 y/p man d. 18 y/o woman
they are larger in size and result in more clinically signficant thromboembolic events
why are clots that orginate in the proximal veins potentially more dangerous?
because of the large amt of anaerobic bacteria in the mouth pg 193
why are human bites known to have a higher rate of infection?
men have the bacteriostatic effect of prostatic fluid and a longer urethra
why do middle age men not incur as many UTIs as middle aged women?
because it stimulates collateral vessel growth in the lower extremities (pg 489)
why is exercise recommended for peripheral artery disease?
the liver pushes the right kidney downward
why is the right kidney slightly lower than the left kidney?
c. to decrease the growth rate of epidermal cells
why is ultraviolet light therapy used to tx psoriasis? a. to dry the lesions b. to kill the bacteria c. to decrease the growth rate of epidermal cells d. to kill the fungi
because steroids can worsen them pg 156
why should topical steroids not be used in rashes that are suspected to be viral?
False
with the use of a direct thrombin inhibitor, ongoing INR is required True or False
a. inserting a key into the narrow slow of a lock
with which of the following movements might a clt with a cerebellar problem have difficulty? a. inserting a key into the narrow slow of a lock b. driving a car with a standard shift c. walking up stairs d. eating
c. iron -deficiency anemia
women who take oral contraceptives are less likely to experience: a. human papillomavirus infection b. migraine headace c. iron -deficiency anemia d. herpes simplex virus
dry skin pg 150
xerosis is
a patient between ages 50-85 presents with sudden onset of severe eye pain, vomiting, headache, the conjunctiva may be injected, steamy cornea, pupil may be fixed, partially dilated, narrow chamber angle. Does this require urgent treatment?
yes - symptoms of Acute glaucoma pg 248
d. a posterior rounding at the thoracic level
you are assessing M., a 69 y.o Asian woman, for the first time. you are trying to differential between scoliosis and kyphosis. Kyphosis involves: a. asymmetry of the shoulders, scapulae, and waist creases b. a lateral curvature and vertebral rotation on posteroanterior x-rays c. one leg appearing shorter than the other d. a posterior rounding at the thoracic level
b. cruciate ligament
you are assessing M., age 16 after a football injuryt to his rt knee. you elicit a positive anterior/posterior drawer sign. this test indicates an injury to the: a. lateral meniscus b. cruciate ligament c. medial meniscus d. collateral ligament
b. NSAIDs
you are caring for a pt that has a hx of psoriasis and now is showing signs of musculoskeletal s/s with joint involvement. seropositivity provides a definitive dx of psoriatic arthritis (PsA). your initial tx choice for management of the pt is: a. disease modifying antirheumtic drugs (DMARDs) b. NSAIDs c. tumor necrosis factor-alpha inhibitors (TNF-alphal inhibitor) d. uricosuric
a. synovial-fluid analysis and x-ray
you are considering a dx of calcium pyrophosphate dihydrate (CPPD) crystal deposition disease or pseudogout in a 72 y/o man who presents with c/o pain and stiffness in his wrists and knees. the most useful dx test to assist you in making this dx would be: a. synovial-fluid analysis and x-ray b. bacterial culture c. bone scan and MRI d. anticitrullinated protein antibody (ACPA) test and RA factor
c. establish an airway
you are driving home from work and stop at the scene of a motorcycle accident that must have just occurred because there are no rescue vehicles at the scene the drive is lying at the side of the road unconscious with an obvious open fx of his femur. which of the following actions should take priority? a. stop the bleeding b. determine if there has been a cervical fx c. establish an airway d. palpate the peripheral pulses
c. lymphoma of the mediastinum
you are examinging Joe, age 9 months, and note a palpable right supraclavicular node. You know that this finding is suspicious for: a. candidiasis b. cryptococcosis c. lymphoma of the mediastinum d. abdominal malignancy
d. black and tarry
you are performing a rectal exam on J. for f/u of his melena. what do you expect his stool to look like if his condition has not resolved? a. grayish tan b. bright red c. pale yellow, greasy, and fatty d. black and tarry
a. phimosis
you are performing a school PE on D. age 5. you are unable to retract his foreskin over the glans penis while inspecting his penis. this is referred to as: a. phimosis b. paraphimosis c. microphallus d. priapism
a. Brudzinski sign
you are performing some neurological assessment tests on Daniel. When you ask Daniel to lie supine and flex his head to his chest, what are you assessing? a. Brudzinski sign b. Kernig's sign c. decorticate posturing d. decerebrate posturing
a. blocking the release of follicle-stimulating hormone (FSH) and luteinizing hormone (LH)
you are referring a 73 y/o clt for management of his prostate cancer with hormonal txment. it is understood that goserelin acetate (zoladex) acts as a method of androgen ablation by: a. blocking the release of follicle-stimulating hormone (FSH) and luteinizing hormone (LH) b. blocking 5-alpha-reductase, which converts testosterone into dihydrotestosterone c. inhibiting the binding of testosterone to the cancer cells d. inhibiting the progression of cancer cells through the cell cycle
a. nothing, this is not a cause for concern
you are rolling your fingers along the inguinal ligament and you encounter small, freely mobile lymph nodes in this area. what do you suspect? a. nothing, this is not a cause for concern b. something is abnormal and warrants further eval and possible referral c. the lymph nodes should be biopsied d. the lymph nodes must be congential in origin
c. warts are caused by the human papillomavirus
you are teaching H. about the warts on his hands. What is included in your teaching? a. tx is usually effective, and most warts will not recur afterward. b. because warts have roots, it is difficult to remove them surgically c. warts are caused by the human papillomavirus d. shaving the wart may prevent its recurrence
b. Parkinson's disease
you assess for cogwheel rigidity in Sophia, age 76. What is cogwheel rigidity a manifestation of: a. Alzheimer's disease b. Parkinson's disease c. brain attack d. degenerative joint disease
d. appendicitis
you correctly perform the obturator test when you raise the cl't leg with knee flexed and internally rotate the leg. a positive obturator test is indicative of: a. avascular necrosis of the femoral head b. cholecystitis c. hip bursitis d. appendicitis
a. disseminated malignancy, particularly of the
you have a new clt, Rob, age 67, who presents with a generalized lymphadenopathy. You know that this is indicative of: a. disseminated malignancy, particularly of the hematological system b. cancer of the liver c. Sjogren's syndrome d. pancreatic cancer
b. recommend physical therapy for quadriceps strengthening exercises
you have just completed a work-on M., age 13, and confirmed Osgood-Schlatter disease. you should: a. refer to orthopedics for early surgical correction b. recommend physical therapy for quadriceps strengthening exercises c. advise him to temporarily d/c all sports activities until his growth plates have completely fused d. tell M. that he can resume his usual activities immediately without concern and should begin aggressive exercises to increase muscle bulk and strength
c. wash the medication off within 1-2 hrs
you have just txed J.'s condylomata acuminata with podophyllum in benzoin. what instructions do you give him? a. refrain from sexual relations for 48 hrs b. don't take a shower until tomorrow morning c. wash the medication off within 1-2 hrs d. go into the bathroom now and wash the medication off
b. pyelonephritis
you percuss for pain at the costovertebral angle when examining M. what condition are you assessing for? a. urethritis b. pyelonephritis c. kidney stone d. bladder tumor
greater than 7.0mg/dL
you should be concerned about the use of diuretics, CRF, high protein diet, gout, leukemia, or lymphoma when a serum uric acid is
d. straight-leg-raising test
you suspect a herniated disk on S., age 72. you elevate her affected leg when she is in the supine position and it elicits back pain and sciatic nerve pain. which indicates a positive test. this is known as which test or sign? a. femoral stretch test b. cross straight-leg-raising c. doorbell sign d. straight-leg-raising test
b. rt thoracic curvature
you suspect adolescent idiopathic scoliosis in V., age 15 who is in her growth spurt. you perform the Adams forward-bending test and note a right-sided rib hump. what is this indicative of? a. rt lumbar shifting b. rt thoracic curvature c. rt truncal shift d. spondylolysis
d. unsafe sexual behaviors
you suspect that your new clt Ralph has hepatitis C,although he is currently asymptomatic. Your suspicion is based on his medical hx, which includes which of the following factors that has been identified as a red flag for this disease? a. lactose intolerance b. frequent sore throats and upper respiratory infection c. a hx of mononucleosis at age 17 d. unsafe sexual behaviors
a. thrombocytopenia and elevated transaminase
your 18 y/o clt, Mandy, has infectious mononucleosis. What might you expect her blood work to reflect? a. thrombocytopenia and elevated transaminase b. elevated WBCs c. decreased WBCs d. decreased serum globulins
a. once all the vesicles are crusted over
your 24 y/o clt, whose varicella rash just erupted yesterday, ask you when she can go back to work. What do you tell her? a. once all the vesicles are crusted over b. when the rash is entirely gone c. once you have been on medication for at least 48 hrs d. now, as long as you stay away from children and pregnant women
c. 6-8 wks
your clt had a colostomy several wks ago and is having difficulty finding a permanent appliance that fits. How long do you tell him to wait for the stoma to shrink before buying a permanent appliance? a. 2-4 wks b. 4-6 wks c. 6-8 wks d. just over 2 months
c. osteosarcoma
your clt has just been told that he has a primary bone tumor. he was so upset when he heard this that he focused only on the word tumor and not on the prognosis or type of tumor. which of the following tumors is malignant? a. osteochondroma b. chondroma c. osteosarcoma d. giant-cell tumor
b. supplemental iron
your clt is on a low-protein diet for his ESRD, because of this, which of the following is essential that he include in his diet? a. a phosphate binder b. supplemental iron c. potassium supplements d. vitamin and mineral supplements
a. bladder infection
your clt's c/c is blood in the urine. you know that the most common cause of gross hematuria in the male population is: a. bladder infection b. BPH c. bladder tumor d. prostatitis
c. immunodeficiency
your clt, Jack, has decreased lymphocytes. You suspect: a. bacterial infection b. viral infection c. immunodeficiency d. parasitic infections
c. mononucleosis
your clt, Jeannie, age 64, comes to you complaining of tinnitus and lightheadedness without LOC. On PE, you note splenomegaly. to sort out your differential dx, you order an alkaline phosphatase and vitamin B12 level because you are ruling out a dx of: a. liver cancer b. pancreatic cancer c. mononucleosis d. polycythemia vera
c. she must be sent for a mammogram as soon as possible
your clt, Mrs. Old, age 64, is here to see you because she has pain in her left breast. She reports no pain in her right breast and no noted lesions or masses on breast self-exam, which she performs month. You know that: a. at her age, you do not need to worry about breast cancer b. initial presentation of breast pain is usually not suspicious for a malignancy c. she must be sent for a mammogram as soon as possible d. she has no personal or family hx of breast cancer; therefore, you are not concerned
d. splenectomy
your clt, Ms. Jones, has an elevated platelet count. You suspect: a. systemic lupus erythematosus b. infectious mononucleosis c. disseminated intravascular coagulation (DIC) d. splenectomy
d. liver disease
your clt, Shelly, has an elevated mean cell volume (MCV). What should you be considering in terms of dx? a. iron-deficiency anemia b. hemolytic anemias c. lead poisoning d. liver disease
a. artificial tears and chewing sugarless gum
your clt. Mr. Jones, has Sjogren's syndrome. Which tx do you suggest? a. artificial tears and chewing sugarless gum b. frequent rinsing out of the mouth with mouthwash c. drinking at least one glass of milk per day d. removing wax from the ears at regular intervals
b. physical sunscreens
zinc oxide, magnesium silicate, ferric chloride, and kaolin are examples of: a. chemical sunscreens b. physical sunscreens c. agents used in tanning booths d. emollients
Greater than 60 (pg 438, table 10.3)
What high-density lipoprotein (HDL) level is considered cardioprotective?
by the presence of WBC casts in the urine (pg 601)
how can one differentiate pyelonephritis from cystitis?
1. 5mm 2. 10mm 3. 15mm
guidelines for the classification of rxs to intradermal Mantoux tests have establish what three categories?
12-14 yrs, duration is six months and then regression is expected pg 831
gynecomastia associated with puberty begins at
asymptomatic pg 831
gynecomastia lasting longer than 1 year is usually:
lung volumes are reduced due to musculoskeletal d/o, tumors, lung resection, or ILD (pg 349)
restrictive lung disease are those in which the
15% or greater increase in the FEV1 after two puffs of a beta-adrenergic agonist has been inhaled (pg 341)
reversibility is defined as a
3-6 months Pg 343)
review tx therapy every
will clear with cough (pg 332)
rhonchi which are caused by mucous
reddish-orange (pg 582)
rifampin and phenazopyridine (pryridum) can turn the urine
evaluated by an endocrinologist pg 831
gynecomastia that presents before or after puberty and cannot be associated with physiologic againg, drug, or chronic disease, must be
c. early Alzheimer's disease
Sophie is 82 and scores 25 on the MMSE. what is your initial thought? a. normal for her age b. depression c. early Alzheimer's disease d. late Alzheimer's diseae
Stage D heart failure
have refractory syx of heart failure at rest despite medical therapy, are hospitalize, or require specialized interventions or hospice care pg 463
Stage C heart failure
have structural abnormalities and current or previous symptoms of heart failure pg 463
LDL = optimal according to NIH
<100 mg/dl
Triglycerides = normal
<15o mg/dl pg 438
total cholesterol = desirable
<200 mg/dl pg 438
HDL = low
<40 mg/dl pg 438
LDL = optimal
<70 mg/dl pg 438
persistent albuminuria with a normal GFR >90 mL/min per 1.73m2 of body surface area (pg 620)
Stage 1 of CRF is defined as:
persistent albuminuria with a normal GFR 60-89 mL/min per 1.73m2 of body surface area (pg 620)
Stage 2 of CRF is defined as:
A GFR of 30-59 mL/min per 1.73m2 of body surface area (pg 620)
Stage 3 of CRF is defined as:
A GFR of 15-29 mL/min per 1.73m2 of body surface area (pg 620)
Stage 4 of CRF is defined as:
ESRD with a GFR <15 mL/min per 1.73m2 of body surface area (pg 620)
Stage 5 of CRF is defined as:
40-75% of normal kidney function remains (pg 621)
Stage I - (decreased renal reserve) of renal function is
mild COPD FEV1/FVC <70% FEV1 >=80% PG 352)
Stage I COPD =
mild COPD FEV1/FVC <70%, FEV1 >= 80% pg 352
Stage I of COPD is
20-49% of normal kidney function remains (pg 621)
Stage II - (renal insufficiency) of renal function is
Moderate COPD FEV1/FVC <70% FEV1 50-79% PG 352)
Stage II COPD
moderate COPD FEV1/FVC<70%, FEV1 50-79% pg 352
Stage II of COPD is
<15% of normal kidney function remains (pg 621)
Stage III (End-stage renal disease) of renal function is
Severe COPD FEV1/FVC <70% FEV1 30-49% PG 352)
Stage III COPD
severe COPD FEV1/FVC <70%, FEV1 30-49% pg 352
Stage III of COPD is
VERY Severe COPD FEV1/FVC <70% FEV1 <30% PG 352)
Stage IV COPD
very severe COPD FEV1/FVC <70%, FEV1 <30% predicted or, <50% normal with chronic respiratory failure present pg 352
Stage IV of COPD is
post-influenza in the very young and very old
Staphylococcus pneumonia is more common
Pain starting in the flank and localizing in the costovertebral angle (quiz 621)
The clinical presentation of a client with urolithiasis would include:
total cholesterol = borderline
200-239 mg/dl pg 438
Triglycerides = high
200-499 mg/dl pg 438
a Pterygium commonly is seen at
3 and 9 oclock in the eye position pg 262
patients with proliferative retinopathy should be followed up with the opthalmologist every
3-4 months pg 273
Stat drug = Lipitor (atorvastatin) reduces LDL levels by
40-60% and has the greatest effect of lowering triglycerides pg 441
acute angle-closure glaucoma IOP can occur as high as
40-80 mm Hg pg 269
patients with active proliferative retinopathy should be followed up with the opthalmologist every
8 weeks pg 273
normal intraocular pressure is typically
8-21mm Hg pg 269 10-20 mm Hg pg 270
100mL per day
Anuria is defined as without urine and is an output of less than
Common history Burning, itching Feels like something is in the eye Crusting of eyelids/lashes Warm compresses 5-10 mins bid Scrub lash with diluted baby shampoo bid Treatment Bacitracin Erythromycin Azithromycin drops
Blepharitis (ppt week 2)
edema, pulmonary hypertension, cor pulmonale, cyanotic a CBC may reveal polycythemia secondary to the increased erythropoietin stimulation of increased red blood cells to compensate for the chronic hypoxemia - chronic bronchitis (pg 352)
Blue bloaters s/s are
c. PTSD
Bob, age 49, is complaining of recurrent, intrusive dreams since returning from his Marine combat training. You suspect: a. depersonalization b. schizophrenia c. PTSD d. anxiety
d. all of the above pp 882-883
A patient with type 2 diabetes comes to the clinic after reading about Metformin. Which of the following conditions that the patient also has would be a contraindication to taking Metformin? a. ketoacidosis b. cirrhosis or alcoholic patient c. hypoglycemic episodes d. all of the above
c. cluster headache
Ed, age 50, has a chronic, episodic headache. He states that it can wake him up at night, lasts 15 mins to 3 hrs, and has occurred daily over a period of 4-8 wks. You suspect a: a. tension headache b. migraine headache c. cluster headache d. potential brain tumor
Venous stasis (pg 485, 10.3)
The clinical risk factors for deep vein thrombosis (DVT), which include venous insufficiency, poststroke, and heart failure, fit into which of Virchow's Triad for being a causative factor for DVT?
b. gout
C, age 67, tells you that she has been dx with a condition that causes sudden flares of pain, swelling, and redness of the joints in her toes. she cannot remember the name of the dx but she knows that it is caused by urate crystal that get stuck in the joint and cause pain. J. is on hydrochlorothiazide (HCTZ) for management of her HTN. you suspect a dx of: a. septic arthritis b. gout c. rheumatoid arthritis d. Charcot neuro-osteoarthropathy
a. tricyclic antidepressants pg 875
A 64-year-old male with type 2 diabetes comes to the clinic with c/o "my feet feel like they are on fire." He has loss of vibratory sense, +1 Achilles reflex, and a tack embedded in his left heel. Which of the following would be an appropriate treatment? a. tricyclic antidepressants b. capsacin cream c. vitamin B12 injections d. insulin
a. MS
Diage, age 35, presents with weakness and numbness of the left arm, diplopia, and some bowel and bladder changes for the past week. She states that the same thing happened last year and lasted for several wks. What dx is a possibility? a. MS b. subduralhematoma c. pituitary tumor d. myasthenia gravis
Wolff-Parkinson-White syndrome (pg 472)
A Delta wave on the electrocardiogram (ECG) may be present in which condition?
One distinguishing characteristic between conjunctivitis and iritis is:
A ciliary flush
pneumococcal urinary antigen test that can detect a protein common to all pneumococcal serotypes within 15 minutes (pg 363)
A diagnostic testing for CAP with Streptococcus pneumoniae may include a
A decreased estrogen level (quiz 621)
A factor contributing to stress incontinence is:
arthroscopic extraction of a renal stone from the bladder (pg 606)
A lithotony specifically denotes an
a. Metformin (Glucophage). quiz 624
A newly diagnosed client with diabetes who has an HbA1c of 7.5 is started on therapeutic lifestyle changes (TLC) and medical nutritional therapy (MNT). Which oral antidiabetic agent is recommended as monotherapy? a. Metformin (Glucophage). b. Glipizide. c. Sitagliptin (Januvia). d. Exenatide (Byetta).
b. After hair is shampooed and towel-dried, apply permethrin 1% creme rinse to scalp and hair, and leave this on for 10 minutes before rinsing.
A nurse practitioner is teaching a client how to use permethrin 1% cream rinse (Nix) for treatment of pediculosis capitis. What is the most important information that should be given to the client? a. The shampoo should not be used again, because it is toxic and may be absorbed systemically and cause acute respiratory problems. b. After hair is shampooed and towel-dried, apply permethrin 1% creme rinse to scalp and hair, and leave this on for 10 minutes before rinsing. c. Shampoo hair daily for 1 week with permethrin 1%. d. It is not necessary to treat other members of the family or launder bedding or clothing.
a. Venous stasis
The clinical risk factors for deep vein thrombosis (DVT), which include venous insufficiency, poststroke, and heart failure, fit into which of Virchow's Triad for being a causative factor for DVT? a. Venous stasis b. Vessel injury c.Hypercoagulability
chronic renal failure (pg 620)
A renal ultrasound is performed at baseline for all cases of
dyspnea (pg 334)
A visual analog scale (0-100) or the Borg graph (0-10) should be used to rate
b. after childbirth, your carpal tunnel syndrome may resolve
A. age 42 is pregnant and was just given a dx of carpal tunnel syndrome. she is worried that this will affect her in caring for the baby. what do you tell her? a. don't worry, we'll find a brace that is very malleable b. after childbirth, your carpal tunnel syndrome may resolve c. if we do surgery now, you'll be recovered by the time the baby arrives d. you should prepare yourself or the probability of being unable to care for your baby
a. Lyme disease
Diana, 55, c/o ear pain, rt facial weakness and loss of taste? what dx would you consider? a. Lyme disease b. stoke c. ear infection d. brain tumor
a. Herpes zoster.
Elizabeth, 83, presents with a 3-day history of pain and burning in the left forehead. This morning she noticed a rash with erythematous papules in the at site. What do you suspect? a. Herpes zoster. b. Varicella. c. Rubella. d.Syphilis.
Beck depression scale (pg 356)
For those with COPD, what test can be used to test for depression?
b. Raynaud's phenomenon (quiz 621)
An adult 35-year-old female presents to your office with the chief complaint of intermittent episodes of color changes of her fingertips. The color changes from blue, white, and red and is accompanied by numbness and tingling. The attacks happen for 2-3 hours. Hands and feet become numb in the winter months. Based on this history what is a possible diagnosis? a. peripheral vascular disease b. Raynaud's phenomenon c. diabetic neuropathy d. poor circulation
Inflammation of the conjunctiva Highly contagious Direct contact with secretions S/S Redness Purulent or mucopurulent discharge Unilateral at onset Eyelids "glued" shut in morning Eyelid edema
bacterial conjunctivitis (ppt in week 2)
a. slightly below normal
Ann has sickle cell anemia. In regulating her and monitoring her hemoglobin and hematocrit levels, you want to maintain them at: a. slightly below normal b. strictly at normal c. slightly above normal d. around normal with only minor fluctuations
c. a second birth control method needs to be used during intercourse for the first 7 days while taking the pill. However, it doesn't protect you from STIs
Ann, age 19, is going to begin taking birth control pills. She asks you if she is safe immediately. How do you respond? a. yes, you should not get pregnant once you start the pill. However, it doesn't protect you from STIs b. for the first month, you need to be on a backup birth-control method. However, it doesn't protect you from STIs c. a second birth control method needs to be used during intercourse for the first 7 days while taking the pill. However, it doesn't protect you from STIs d. until you have your second period (cycle) with the pill, you are not considered safe
c. she will probably have growth hormone problems, in which case she can then begin growth hormone tx
Ann, age 5, is being txed with radiation for cancer. Her mothers asks about the effect radiation will have on Ann's future growth. although she knows that a specialist will be handling Ann's care, her mother asks for your opinion. How do you respond? a. let's worry about the cancer first, then see how her growth is affected b. chemotherapy may affect her future growth, but radiation will not c. she will probably have growth hormone problems, in which case she can then begin growth hormone tx d. that the least of your worries now; everything will turn out ok
M. pneumoniae L. pneumophila C. pneumoniae P. jirovecii, and virus (pg 361)
Atypical pneumonia is typically produced by
chlamydial conjunctivitis is a leading cause of blindness in developing nations and should by treated with systemic antibiotics in addition to topical agents. The drug of choice is
Azithromycin 1g as single dose or doxycycline 100mg bid x7 days. Treat the sexual partner simultaneously pg 259
d. engaging in any activity that would elicit prostatic massage
B. a 32 y/o gay man, has been dx with acute bacterial prostatitis. in addition to providing education, you would encourage him to avoid: a. rest b. NSAIDs c. hydration and stool softeners d. engaging in any activity that would elicit prostatic massage
b. chancroid
B. appears with a tender, ulcerated exudative, papular lesion on his pen. it has an erythematous halo, surrounding edema, and a friable base. what do you suspect? a. chancre b. chancroid c. condylomata acuminatum d. genital herpes
c. for the first 6 months, if you breastfeed and have very little supplementation, your chances are less than 2% that you'l get pregnant
Belle is breastfeeding her 3 month old infant with no supplementation./ She says she has heard that she cannot get pregnant during this time. What do you tell her? a. it is highly likely that you may become pregnant, so you should use another method of birth control b. yes, you are safe for as long as you breastfeed c. for the first 6 months, if you breastfeed and have very little supplementation, your chances are less than 2% that you'l get pregnant d. you are more at risk for getting pregnant now because of your fluctuating hormone levels
rare autosomal recessive intrinsic platelet disorder causing bleeding
Bernard-Soulier syndrome is a
c. tinea corporis
B., age 12, presents with annular lesions with a scaly border and central clearing on his trunk. What do you suspect? a. psoriasis b. erythema multiforme c. tinea corporis d. syphilis
a. you suspect a sexually transmitted infection, so you obtain a culture of the urethra, do a potassium hydroxide wet prep, and obtain another urine culture
B., age 24 c/o urgency, frequency, and dysuria. Your dipstick test shows no hematuria, and her urine culture shows no growth. What is your next action? a. you suspect a sexually transmitted infection, so you obtain a culture of the urethra, do a potassium hydroxide wet prep, and obtain another urine culture b. you suspect urethra irritation, so you tell her to take showers, not bubble baths, and wear white, dry underwear and loose-fitting clothing c. you suspect a UTI not visible yet on culture, so you start her on Bactrim DS d. you suspect that the vulva is irritated. You tell her to take a relaxing shower and dry the area well and come back in 1 wk if there is no improvement
d. herniated nucleus pulposus
B., age 49, comes in with low back pain. an x-ray fo the lumbar/sacral spine is within normal limits. which of the following dx do you explore further? a. scoliosis b. osteoarthritis c. spinal stenosis d. herniated nucleus pulposus
c. cancer of the prostate
BPH is a common finding as men age. classically, this conditon may begin with difficulty initiating the urinary stream, hesitancy, urgency, postvoid dribbling, urinary frequency, nocturia, urinary retention, sensation of a full bladder immediately after voiding, and incontinence. these preceding sxs would also cause you to consider what other condition of your diff list? a. epididymitis b. testicular cancer c. cancer of the prostate d. balanitis
Prevention and education regarding conjunctivitis
Bacterial & Viral conjunctivitis are contagious 24 hours of therapy prior to school, daycare etc. Good handwashing Washcloths, pillowcases in laundry Proper contact use No contacts X 1 week New contacts, eye makeup etc. (ppt, week 2)
a. Meniere's disease
Barbara, age, 36, presents with episodic attacks of severe vertigo, usually with associated ear fullness. Her attacks usually last several hrs and she feels well before and after the attacks. To what might you attribute these sxs? a. Meniere's disease b. vestibular neuronitis c. benign paroxysmal positonal vertigo d. otosclerosis
Strategy C
Begin with 2 drugs at the same time, either as 2 separate pills or as a single pill combination Initiate therapy with 2 drugs simultaneously, either as 2 separate drugs or as a single pill combination. Some committee members recommend starting therapy with ≥2 drugs when SBP is >160 mm Hg and/or DBP is >100 mm Hg, or if SBP is >20 mm Hg above goal and/or DBP is >10 mm Hg above goal. If goal BP is not achieved with 2 drugs, select a third drug from the list (thiazide-type diuretic, CCB, ACEI, or ARB), avoiding the combined use of ACEI and ARB. Titrate the third drug up to the maximum recommended dose.
c. CN VII
Bell's palsy affects which CN? a. CN V b. CN VI c. CN VII d. CN VIII
preauricular lymph nodes are not palpable in
bacterial conjunctivitis, allergic conjunctivitis, iritis, and acute glaucoma pg 247
c. Candida infection of the penis
balanitis is associated with: a. diabetes b. macular degeneration c. Candida infection of the penis d. measles
b. the collagen fibers that connect muscle to bone
C. a 22 y/o carpenter who is rt hand dominant, comes to you for f/u from the ED where he was seen for rt forearm pain. he states he was dx with rt forearm tendinitis and wants you to explain this dx to him. you explain that he has inflammation of one or more tendons, which are: a. the rope-like bundles of collagen fibrils that connect bone to bone b. the collagen fibers that connect muscle to bone c. the pouches of synovial fluid that cushion bone and other joint structures d. the fibrocartilaginous disks that separate bony surfaces
b. removal of a large core of tissue from the lesion for histological eval utilizing a large-gauge cutting needle
Dolly, is to undergo a cone-needle biopsy for a suspicious breast mass. This procedure includes: a. 21 or 22 gauge needle that is used to aspirate cells from the lesion for analysis b. removal of a large core of tissue from the lesion for histological eval utilizing a large-gauge cutting needle c.removal of a wedge of tissue for exam d. removal of the entire lesion
b. it is the worst headache I have ever had
Don, age 62, calls to complain of a severe headache. Which of the following statements most concerns you? a. it hurts whenever I turn my head specific ways b. it is the worst headache I have ever had c. nothing I do seems to help this constant ache d. I am so worried, can you do a CT scan?
rifampin plus pyraziname (PZA or Z) plus ethambutol for a min of 6 months (pg 374)
Drug resistance is common in HIV patients and immigrants, if INH resistance is confirmed, tx TB with
Recommendation 4
In the population aged 18 years with chronic kidney disease (CKD), initiate pharmacologic treatment to lower BP at SBP 140mmHg orD BP 90mmHgandtreat to goalSBP<140mmHgandgoalDBP<90mmHg. (Expert Opinion - Grade E)
Recommendation 5
In the population aged 18 years with diabetes, initiate pharmacologic treatment to lower BP at SBP 140 mmHg or DBP 90mmHg and treat to a goal SBP <140mmHg and goal DBP <90mmHg. (Expert Opinion -Grade E)
b. external vacuum device
D. has an ED and says that a friend told him about a method that uses a constricting ring around the base of the penis. what is he referring to? a. intracavernous injection txment b. external vacuum device c. urethral suppositories d. surgery
a. integumentary effects include swelling, erythema, and warmth at the involved site
D. age 49 developed osteomyelitis of the femur after a motorcycle accident. which of the following statements about the clinical manifestations of osteomyelitis is correct? a. integumentary effects include swelling, erythema, and warmth at the involved site b. there is a low-grade fever with intermittent chills c. musculoskeletal effects include thenderness of the entire leg d. cardiovascular effects include bradycardia
c. Dupuytren's contracture
D. who is 45 and of norther european ancestry, has a dysfunctional disfiguring condition affecting the palmar tissue between the skin and the distal palm and fourth and fifth fingers. what do you suspect? a. Hallux valgus b. DeQuervain's tenosynovitis c. Dupuytren's contracture d. Hallux rigidus
c. systemic antibiotics
D., age 29 has a high fever and red, warm, sharply marginated plaques on the right side of her face that are indurated and painful. You dx erysipelas. What tx do you begin? a. systemic steroids b. topical steroid c. systemic antibiotics d. NSAIDs
d. multiple pregnancies
D., is a 23 y/o pt dx with dyspareunia. Which of the following is NOT a cause for this condition? a. vulvovaginitis b. an incompletely stretch hymen c. vaginismus d. multiple pregnancies
directly observed therapy and is needed for all pts because of the difficulty in predicting which pt will adhere to a prescribed regimen (pg 374)
DOT is
d. have you thought of suicide?
Dan, age 82, recently lost his wife to breast cancer. He presents with weight loss, fatigue, and difficulty sleeping. What should your first response be? a. do you have a hx of thyroid problems in your family b. do you think a sleeping pill might help you sleep at night c. things might look up if you added nutritional supplements to your diet d. have you thought of suicide?
c. delirium
Dave, age 76, is brought in by his wife, who states that within the past 2 days Dave has become agitated and restless, has had few lucid moments, slept very poorly last night, and can remember only recent events. Of the following differential dx, which seems the most logical from this brief hx? a. depression b. dementia c. delirium d. schizophrenia
b. orchitis
E's C/C is heaviness in the scrotum. you assess swelling of the testicle, along with warm scrotal skin. what do you suspect? a. cryptorchidism b. orchitis c. testicular torsion d. epididymitis
b. Salter-Harris III
E. a 10 y/o boy, jumps off a 2 foot wall, twisting his foot and ankle upon landing. his ankle xpray demonstrates a fx of the distal tibia over the articular surface into the epiphysis and physis. based on the Salter-Harris classification for growth plate injuries you know this is a: a. Salter-Harris II b. Salter-Harris III c. Salter-Harris IV d. Salter-Harris V
b. vascular d/o
ED which affects 18-30 million men in the US increases with age. in men older than the age of 50, what are the most commonly found contributors to ED? a. endocrine diseases b. vascular d/o c. neurogenic disease d. psychiatric conditions
d. stretching and warm-up exercises are an important part of any exercise routitne
E., age 21 presents today with another muscle strain from one of her many sports activities. you think that she was probably never taught about health promotion and maintenance regarding physical activity. what information do you include in your teaching? a. after an activity, if any part hurts, apply ice for 20 minutes b. you must first get in shape with a rigorous schedule of wt training and then you can participate in any activity once you are physically fit c. after any strenuous activity, you must completely rest your muscles before beginning your next activity d. stretching and warm-up exercises are an important part of any exercise routitne
b. herpes zoster
E., age 83 presents with a 3 day hx of pain and burning in the left forehead. This am, she noticed a rash with erythematous papules in that site. What do you suspect? a. varicella b. herpes zoster c. syphilis d. rubella
peaked P waves in leads II, III, and AVF, and changes associated with right ventricular hypertrophy (pg 353)
ECG changes in pulmonary disease include
peaked P waves in leads II, III, and a Vf and changes associated with rt ventricular hypertrophy (pg 353)
ECG changes in pulmonary disease include
d. Benzoyl peroxide.
First-line treatment for acne vulgaris with closed comedones includes: a. Oral antibiotics. b. Hydrocortisone cream. c. Isotretinion. d. Benzoyl peroxide.
Acute otitis media is diagnosed when there is:
Fluid in the middle ear accompanied by otalgia and fever.
pregnant women because of effects on bone and cartilage formation in the developing fetus (pg 598)
Fluoroquinolones should be avoided in
No tetracaine needed if patient is not photophobic Will permanently stain clothing Ulcers fluoresce distinctive round lesions Abrasions may be singular or multiple with an "ice rink" appearance Foreign bodies fluoresce around FB and the center is dark Rinse fluorscein stain out of eye with NSS at end of exam
Fluorscein Stain (ppt week 2)
with the use of electromyographical studies pg 93
How is Amyotrophic Lateral Sclerosis (Lou Gehrig's disease) diagnosed?
via respiratory droplet
How is mycoplasma pneumoniae transmitted?
d. every visit pg 875
How often should the healthcare provider examine the feet of a person with diabetes? a. annually b. every 6 months c. every 3 months d. every visit
Recommendation 8
In the population aged18 years with CKD, initial (or add-on) antihypertensive treatment should include an ACEI or ARB to improve kidney outcomes. This applies to all CKD patients with hypertension regardless of race or diabetes status. (Moderate Recommendation - Grade B)
racial and ethnic minorities (pg 368)
In the U.S., TN is typically a disease of
c. rheumatoid arthritis pg 944
In the U.S., what is the 2nd most common connective tissue disease and the most destructive to the joints? a. osteoarthritis b. systemic lupus erythematosus c. rheumatoid arthritis d. Sjogren's syndrome
Recommendation 7
In the general black population, including those with diabetes, initial antihypertensive treatment should include a thiazide-type diuretic or CCB. (For general black population: Moderate Recommendation -Grade B; for black patients with diabetes: Weak Recommendation - Grade C)
Recommendation 6
In the general nonblack population, including those with diabetes, initial antihypertensive treatment should include a thiazide-type diuretic, calcium channel blocker (CCB), angiotensin-converting enzyme inhibitor (ACEI), or angiotensin receptor blocker (ARB). (Moderate Recommendation - Grade B)
Recommendation 2
In the general population <60 years, initiate pharmacologic treatment to lower BP at DBP 90mmHg and treat to a goal DBP<90mmHg. (For ages 30-59 years, Strong Recommendation - Grade A; For ages 18-29 years, Expert Opinion - Grade E)
Recommendation 3
In the general population <60 years, initiate pharmacologic treatment to lower BP at SBP140mmHg and treat to a goal SBP <140mmHg. (Expert Opinion - Grade E)
Recommendation 1
In the general population aged 60 years, initiate pharmacologic treatment to lower blood pressure (BP) at systolic blood pressure (SBP)150 mmHg or diastolic blood pressure (DBP)90mmHg and treat to a goal SBP <150 mm Hg and goal DBP <90 mm Hg. (Strong Recommendation - Grade A)
Recommendation 1 - Corollary Recommendation
In the general population aged 60years, if pharmacologic treatment for high BP results in lower achieved SBP (eg, <140mmHg) and treatment is well tolerated and without adverse effects on health or quality of life, treatment does not need to be adjusted. (Expert Opinion - Grade E)
plaque burden
Intravascular ultrasonography (IVUS) measures pg 445
not for use of acute asthma attacks do not give to pts with glaucoma or BHP (pg 345)
Ipratropium bromide (Atrovent) is
b. it decreases parasympathetic tone and produces bronchodilation
Ipratropium is very widely used in the tx of COPD. Which of the following statements about ipratropium is correct? a. it slows the progression of COPD b. it decreases parasympathetic tone and produces bronchodilation c. it has anti-inflammatory actions and reduces bronchoconstriction d. it is more effective than a beta agonist in producing bronchodilation
Yes (Advair Discus), it is extremely effective (pg 344)
Is Salmeterol effective when used in COMBINATION with inhaled corticosteroids?
Yes
Is pertussis a reportable disease? Yes No
yes, pg 953
Is splenic rupture a risk in those with infectious mononucleosis?
b. it is contraindicated with isosorbide monoNITRATE; let's discuss other options
J. asks for a rx for sildenafil (viagra). he says that the only med he takes is isosorbide mononitrate (Monoket) oral tables and that he has diabetes, but it is controlled by diet alone.w hat do you tell him? a. let's try a sample and see how you do b. it is contraindicated with isosorbide mononitrate; let's discuss other options c. because of your hx of diabetes, we can't use it d. I'd better refer you to a urologist
a. infertility is not an issue until you have had unprotected sex for at least 1 yr
J and Jane have been married for 6 months and are unable to conceive. they ask you to recommend an infertility specialist. what do you tell them? a. infertility is not an issue until you have had unprotected sex for at least 1 yr b. let's run some routine tests first; then I'll recommend someone c. tell me about your sexual experiences d. it's usually a problem with the woman, so let's have Jane examined first
c. finasteride
J. a 72 y/o is seen at the practice for f/u of several episodes of orthostatic hypotension. it also appears through a review of his systems and a DRE that he has BPH with lower urinary tract sxs. you review his recent ultrasonic eval that reports a prostate over 40ml and the results of the American Urological Association sxs index for BPH, and find his score to be 12. based on the preceding info, and the clt's desire for noninvasive med txment, what management would you offer him? a. prazosin b. doxazosin c. finasteride d. phenoxybenzamine
b. stage 2
J. age 13, notices a sparse growth of long, slightly pigmented, downy pubic hair at the base of his penis; slightly larger testes; and a larger red scrotum with a different texture. what Tanner stage is he in? a. stage 1 b. stage 2 c. stage 3 d. stage 4
c. begin lower back strengthening exercises depending on pain tolerance
J. age 22, a stock boy, has an acute episode of low back pain. you order an NSAID and tell him which of the following? a. maintain moderate bedrest for 3-4 days b. call the office for narcotic meds if there is no relief with the NSAID after 24-48 hrs c. begin lower back strengthening exercises depending on pain tolerance d. wear a Boston brace at night
a. breast cancer
J. age 42 presents for her well-woman exam and you notice "dimpling" on her left breast. Your initial reaction is that is may possible be: a. breast cancer b. fibrocystic breast disease c. Paget's disease d. striae from recent dieting
a. inability to shrug the shoulder
J. age 49 has recently begun a rigorous wt-lifting regimen. she presents in your office with a shoulder dislocation. which of the following clinical manifestations make you suspect an anterior shoulder dislocation over a posterior dislocation? a. inability to shrug the shoulder b. absence of pain c. inability to rotate the shoulder externally d. shortening of the arm
b. affected joints are swollen, cool, and bony hard on palpation
J. age 49 states that she thinks she has rheumatoid arthritis. before any dxnostic test are ordered, you complete a PE and make a tentative dx of osteroarthritis rather than rheumatoid arthritis. which clinical manifestation r/o rheumatoid arthritis? a. fatigue b. affected joints are swollen, cool, and bony hard on palpation c. decreased ROM d. stiffness
a. bony fragments are in many pieces
J. age 64 comes in for a visit. she has a cast on her rt arm and tells you that she has a comminuted fx of her radius. when she asks what that means, you tell her that in a comminuted fx the: a. bony fragments are in many pieces b. broken ends of the bone protrude through the soft tissues and skin c. bone breaks cleanly but does not penetrate the skin d. bone is crushed
b. acetaminophen
J. age 64, has rheumatoid arthritis. which of the following drugs would be of the least benefit? a. disease modifying antirheumatic drugs (DMARDs) b. acetaminophen c. NSAIDs d. glucocorticoids
d. raloxifene
J. age 76 has been given dxes of osteoporosis confirmed with a dual-energy x-ray absorptiometry (DEXA) scan. you have educated her about the importance of increasing calcium and vitamin D in her diet and starting a low impact wt bearing exercise program. you are also going to start her on medial management. J. ask you about a drug called a "SERM" that she has heard has been shown in studies to prevent vertebral fx. which of the following pharmacological tx for osteoporosis is classified as a selective estrogen receptor modulator (SERM)? a. aldendronate b. risedronate c. salmon calcitonin d. raloxifene
c. a loop of vas deferens is occluded through the scrotal skin
J. and Martha have 5 children and do not want any more. J. said he heard about a no-scalpel vasectomy and asks how it works. what do you tell him? a. for the vasectomy to be permanent, you must have the vas deferens excised b. it's safer for Martha to be sterilized c. a loop of vas deferens is occluded through the scrotal skin d. the testes are twisted,w hich occludes the vas deferens
b. ultrasound
J. appears with a rapid onset of unilateral scrotal pain radiating up to the groin and flank. you are trying to differentiate between epididymitis and testicular torsion. which test to determine whether swelling is in the testis or the epididymis should be your first choice? a. x-ray b. ultrasound c. technetium scan d. PE
c. iron-deficiency anemia
Laurie, age 29, appears with the following signs: pale conjunctiva and nailbeds, tachycardia, heart murmur, cheilosis, stomatitis, splenomegaly, koilonychia, and glossitis. What do you suspect? a. vitam B12 deficient b. folate deficiency c. iron-deficiency anemia d. chronic fatigue syndrome
Tasks of an APRN
Learn the patient's history/history of present illness Perform the physical exam Appropriate use/interpretation of tests Appropriate choice of therapies Consider need for consultation and/or collaboration Negotiate decisions with the patient Evaluate outcome(s) of care Derive decisions regarding care from evidence-based studies (ppt week 2)
gram negative bacteria but is also known as an Atypical form of pneumonia. The bacteria thrives in aquatic environments (pg 360)
Legionella pneumophila is caused by a
"it is hereditary and unfortunately, incurable, but there are many measures we can use in dealing with it."
Leslie, age 13, says her mother has polycystic kidney disease and asks about her chances of developing it. How do you respond?
NOT FOR ACUTE ASTHMA ATTACKS (pg 345)
Leukotriene Receptor Antagonists such as: Montelukas (singulair) Zafirlukas (accolate) Cromolyn sodium (intal) Nedocromil sodium (tilade) are
never used for ACUTE ATTACKS (pg 345)
Leukotriene receptor antagonists are
d. Limiting beer intake to 24 oz/day. (pg 430 table 10.2)
Lifestyle modifications to manage hypertension include: a.Maintaining a body mass index (BMI) of 17 (18.5-24.9). b.Restricting dietary sodium to 2 g per day. (2.4g/day) c.Encouraging 90 minutes of exercise per day.(30min per day/most of week) d.Limiting beer intake to 24 oz/day. (pg 430 table 10.2)
a. obtain a lead level
Lisa, age 2, had a complete blood count drawn at her last visit. It indicates that she has a microcytic hypochromic anemia. What should you do now at this visit? a. obtain a lead level b. instruct Lisa's parents to increase the amt of milk in her diet c. start Lisa on ferrous sulfate (feosol) and check the CBC in 6 wks d. recheck the CBC on this visit
borrelia burgdorferi pg 957
Lyme disease is a caused by
c. the person with dementia tries to hide problems concerning his/her memory, whereas the person with depression complains about memory
Lynn, age 72, presents for the first time with her daughter. Her daughter describes some recent disturbing facts about her mother. How can you differentiate between depression and dementia? a. you might be able to pinpoint the onset of dementia, but the onset of depression is difficult to identify b. a depressed person has wide mood swings, whereas a person with dementia demonstrate apathetic behavior c. the person with dementia tries to hide problems concerning his/her memory, whereas the person with depression complains about memory d. the person with dementia has a poor self-image, whereas the person with depression does not have a change in self-image
ranging from M0 or M1
M describes the extend of metastasis
b. direct inguinal hernia
M, age 70 is obese. he is c/o a bulge in his groin that has been there for months. he states that it is not painful, but it is annoying. you note that the origin of swelling is above the inguinal ligament directly behind and through the external ring. you dx this as: a. indirect inguinal hernia b. direct inguinal hernia c. femoral hernia d. strangulated hernia
c. yes, you have an increased risk for hormone dependent cancers because of your obesity
Maria asks if being overweight predisposes her to cancer. How do you respond? a. no you have the same risk as a normal weight individual b. you have less of a risk of cancer than normal weight individuals because you have protein stores to combat mutant cells c. yes, you have an increased risk for hormone dependent cancers because of your obesity d. yes, you have an increased risk because you have many more cells in all the organs of your body
b. there are many different clinical courses of MS, and the chronic-relapsing type is only one of them
Marian, age 39, has MS. She tells you that she heard that the majority of people with MS have the chronic-relapsing type of disease and that she has nothing to live for. How do you respond? a. the majority of people have this response to MS b. there are many different clinical courses of MS, and the chronic-relapsing type is only one of them c. the chronic-relapsing type of MS is in the minority d. there is an even chance that you have this type
d. PTSD
Marie, age 17, was raped when she was 13. She is now experiencing sleeping problems, flashbacks, and depression. What is your initial dx? a. depression b. panic disorder c. anxiety d. PTSD
c. do you have a plan?
Mark, age 29, tells you that he has thought about suicide. Which should you say next? a. how long have you felt this way? b. tell me more about it? c. do you have a plan? d. have you told anyone else?
b. staphylococcal scalded skin syndrome (SSSS)
Mark, aged 10, presents with impetigo that has been dx as infected with staphylococcus. The clinical presentation is the epithelium layer of the skin is peeling off in large pieces and appears his skin has been scalded. Which type of impetigo is this? a. bullous impetigo b. staphylococcal scalded skin syndrome (SSSS) c. nonbullous impetigo d. ecthyma
a. Pyridoxine (pg 374)
Marsha has been started on a tuberculosis (TB) regimen. Because isoniazid (INH) may cause peripheral neuropathy, you consider ordering which of the following drugs prophylactically? a. Pyridoxine b. Thiamine c. Probiotic d. Phytonadione
take 200mg of ofloxacin with a large glass of water after intercourse
Martha, age 42, states that she has itnercourse only about once a month, but immediately afterward she usually gets a UTI. She is frustrated with having to come to the office frequently to have an exam with a urine test. She says she is ready to kick her husband out of bed. How do you respond?
b. the sxs must be severe enough to upset the cl'ts daily routine or to impact his/ or her work or interfere with relationships
Mary, age 59, presents with depression. According to the diagnostic criteria for a major depressive d/o, which of the following criterion needs to be met? a. the depression must have a specific cause, like alcohol, drugs, medication s/e, or physical illness b. the sxs must be severe enough to upset the cl'ts daily routine or to impact his/ or her work or interfere with relationships c. the depression may be a normal reaction to the death of a loved one d. there must be some type of sleep disturbance
a. temporal arteritis
Mary, age 82, appears without an appt. She is c/o a new, unilateral headache, fever, and muscle aches. She denies any precipitating event. On further exam, you note that her erythrocyte sedimentation rate is over 100mm/min. What do you suspect? a. temporal arteritis b. meningitis c. subarachnoid hemorrhage d. intracerebral hemorrhage
Medications for Allergic conjunctivitis are
Mast cell stabilizers - lodaxamide, nedocromil 2%, pemirolast 0.1% Antihistamine - emedastine 0.05%, Levocabastine 0.05% Combination mast-cell & antihistamines Nonsteroidal antiinflammatory pg 258
b. L5-S1
Mattie, age 52, has a ruptured vertebral disk with the following sxs: pain int he midgluteal region, as well as the posterior thigh and calf to heel area; paresthesias in the posterior calf and lateral heel, foot, and toes; and difficulty walking on her toes. Which intervertebral disks are involved? a. L4-L5 b. L5-S1 c. C5-C6 d. C7-T1
c. hepatitis D
Maurice is an IV drug abuser with chronic hepatitis B (HBV). The development of which type of hepatitis poses the greatest risk to a clt with HBV? a. hepatitis A b. hepatitis C c. hepatitis D d. hepatitis E
metabolic acidosis
Maury has renal failure, his arterial blood gas readings show a decreased pH, normal PCO2, and decreased HCO3, what do you suspect?
In OE Ciprofloxin & Dexamethasone (Ciprodex) 4 gtts bid x7 days
May be used with perforated TM (ppt week 2)
b. yes, calcium intake should be increased to 1200mg/day along with 600mg of vitamin D to decrease bone turnover and increase intestinal absorption
Melody, age 50, desires hormone replacement therapy (HRT) for her hot flashes, which she can't stand. You have discussed the pros and cons and given her some alternative suggestions. Her mother had a hx of osteoporosis. You have decided to initiate therapy for one year. She asks you if she also needs to take calcium or vitamin D for prevention of osteoporosis. How do you respond? a. research has shown that HRT alone is sufficient to protect against osteoporosis b. yes, calcium intake should be increased to 1200mg/day along with 600mg of vitamin D to decrease bone turnover and increase intestinal absorption c. if you decide to take calcium and vitamin D, you can stop the HRT d. if you are getting sufficient exercise, you don't need to take calcium and vitamin D
d. ecthyma pg 181
Melody, aged 25, presents with impetigo that has been dx as infected with staphylococcus. The clinical presentation is pruritic tender, red vesicles surrounded by erythema with a rash that is ulcerating. She has not been adequately treated recently. Which type of impetigo is this? a. bullous impetigo b. staphylococcal scalded skin syndrome (SSSS) c. nonbullous impetigo d. ecthyma
seizure disorders, arrhythmias, active peptic ulcer disease (pg 346) AND are not for use in acute asthma attack
Methylxanthines (theophylline, bronkodyl, theobid, uniphyl, aminophylline, Phyllocontin) are contraindicated in:
c. adenocarcinomas of the colon, lung, breast, ovary, stomach, and pancrease
Mindy states that a relative is having a carcinoembryonic antigen (CEA) test done to detect some type of cancer. She wants to know what kind. You tell her a CEA is performed to detect: a. adenocarcinoma of the prostate b. medullary cancer of the thyroid c. adenocarcinomas of the colon, lung, breast, ovary, stomach, and pancreas d. multiple myeloma
In assessing a client with bacterial conjunctivitis, the nurse practitioner finds:
Minimal itching, moderate tearing, and profuse exudate.
10 (pg 361)
Most people are infected with the Pneumocystis jirovecii fungus by age
a. elevated uric acid level pg 899 elevated wbc count, elevated temperature, elevated serum uric acid or normourecemia (acute phase, pg 900)
Mr. KY presents in the clinic with pain, tenderness, erythema, and swelling of his left great toe. The healthcare provider suspects acute gout. Which of the following should the healthcare provider suspect in the initial test results for this patient? a. elevated uric acid level b. elevated blood urea nitrogen (BUN) c. decreased urine pH d. decreased C-reactive protein (CRP)
c. how do you feel this surgery will affect you?
Mr. M. is a 72 y/o African American with type I diabetes mellitus. he has been a chronic smoker for 50 yrs. he has been told recently that he must have an above-the-knee amputation because of a gangrenous foot. he has lost the will to live and states, "they shoot horses don't they? "how do you respond? a. you should be thankful they can save your life, if not your leg b. your wife needs you; you must think of her at this time c. how do you feel this surgery will affect you? d. Iwill stay with you before, during, and after the surgery because I know that this is a difficult time for you
a. discuss how a positive test would not change Mr. S' txment plan because his life expectancy is less than 10 yrs. review the risk-benefit ratio of txment should cancer be found and discuss this together with Mr. S
Mr. M. is a frail, 87 y/o in your nursing home. he has a hx of multiple strokes, diabetes type 2, heart disease managed medically, and hyperthyroidism. he has a life expectancy of less than 10 yrs. he has no overt urinary sxs. his anxious daughter, Karen, is requesting prostate cancer screening. what would be your most appropriate response? a. discuss how a positive test would not change Mr. M' txment plan because his life expectancy is less than 10 yrs. review the risk-benefit ratio of txment should cancer be found and discuss this together with Mr. M b. order a PSA test and perform a DRE c. send Mr. M for a CT scan d. send Mr. M for a MRI
c. Bulge test
Mr. MK was recently given a dx of degernative joint disease. which assessment test would you use to check for effusion on his knee? a. Thomas test b. Tinel's sign c. Bulge test d. Phalen's test
a. inguinal herniation and peritonitis
Mr. S. comes to you with scrotal pain. the exam of his scrotum, penis, and rectum are normal. which of the following conditions outside of the scrotum may present as scrotal pain? a. inguinal herniation and peritonitis b. renal colic and cardiac ischemia c. pancreatitis and Crohn's disease d. polyarteritis nodosa and ulcerative colitis
d. propranolol
Mr. Smith has smoked for 45 years. He has a hx of HTN, gout, and benign prostatic hyperplasia. Which fo the following medications may worsen one of his diseases? a. amlodipine b. multivitamin with iron c. vitamin B12 d. propranolol
b. Charcot joint pg 875 can be confused with cellulitis
Mr. YM, 55 years old, is seen in the clinic with concerns about his left foot. He has a 40-year history of type 1 diabetes with "fairly good" control on twice-daily insulin. He denies injury but states that he tripped a few months previous and that his foot is sore when he walks. Physical examination reveals an edematous, erythemic, and warm foot. There is a superficial ulcer on the plantar surface. Which of the following is the most likely diagnosis? a. fallen arch b. Charcot joint c. arthritis d. sprained ankle
b. dorsal kyphosis
Mr.s K. age 80 has a curvature of the spine. this is likely to indicate which age-related change? a. lordosis b. dorsal kyphosis c. scoliosis d. kyphoscoliosis
a. insert nothing in the vagina for 24 hrs before the exam
Mrs. G., called the office to make an appt for her annual Pap smear. She needs to be given the following instructions prior to her appt: a. insert nothing in the vagina for 24 hrs before the exam b. douching enhances visualization of the cervix and should be done before the appt c. an infection or menstrual period is no reason to cancel the appt d. the procedure is completely painless
prerenal type of ARF by causing a decreased intrarenal arteriolar resistance
NSAIDs and ACEi can cause the
d. you have to accept this eventually; just glance at it today
Nancy recently had a mastectomy and refuses to look at the site. Her husband does all the dressing changes. When she comes into the office for a postoperative checkup, what would you say to her? a. you will look at it when you are ready b. you must look at it today c. everything is going to be OK, it looks fine d. you have to accept this eventually; just glance at it today
A Chalazion is
Not an infection. Blocked meibomian gland. Develop over weeks to months. Typically self-limiting If persistent - refer to opthamology. (ppt week 2)
TX, N0, M0 pg 385
Occult cancer =
In treating a patient with allergic conjunctivitis, the most appropriate treatment option is:
Opthalmological antihistamines
There are THREE phases of cardiac rehab:
Phase I: begins early in the hospital postinfarct with ROM exercises progressing to an increase in ADLs Phase II: is a structured outpatient program of exercise training and risk factor counseling and education Phase III: establishes a lifelong pattern of aerobic exercise, further lifestyle modification and usually undertalen in a community wellness center pg 459
d. Not enough intrinsic factor 624 quiz
Pernicious anemia is a result of? a. Not enough vitamin D. b. Not enough iron. c. Not enough folic acid. d. Not enough intrinsic factor.
d. alcohol modifies the metabolism of carcinogens in the esophagus and increases their effectiveness
Shelly has esophageal cancer and ask you if alcohol played a part in its development. How do you respond? a. your cancer was caused by your cigarette smoking, nothing else b. alcohol is also a carcinogen c. alcohol directly alters the DNA and causes mutations d. alcohol modifies the metabolism of carcinogens in the esophagus and increases their effectiveness
b. Order a serum iron, TIBC, and serum ferritin level 624 quiz
Sherri's blood work returns with a decreased mean cell volume (MCV) and a decreased mean cellular hemoglobin concentration (MCHC). What should you do next? a. Order a serum ferritin. b. Order a serum iron, TIBC, and serum ferritin level. c. Order a serum folate level. d. Order a serum iron and total iron binding capacity (TIBC).
d. order a serum iron, TIBC, and serum ferritin level
Sherri's blood work returns with a decreased mean cell volume (MCV) and a decreased mean cellular hemoglobin concentration (MCHC). What should you do next? a. order a serum iron and total iron binding cap0acity (TIBC) b. order a serum ferritin c. order a serum folate level d. order a serum iron, TIBC, and serum ferritin level
a. a tricyclic antidepressant
Sigrid, age 83, has postherpetic neuralgia from a bout of herpes zoster last year. She has been having daily painful episodes and would like to start some kind of therapy. What do you recommend? a. a tricyclic antidepressant b. a beta blocker c. a systemic steroid d. an NSAID
b. nitrate
Sildenafil (viagra) 50mg taken 1 hr before sexual activity is ordered for M. for his ED. what med must you make sure he is not taking before writing the rx? a. antihistamine b. nitrate c. stool softener d. anticoagulant
nine times, is frequently accompanied by rheumatic disease and the average age of onset is 40-60 yrs pg 961
Sjogren's syndrome affects women how many more times than men?
dryness in the exocrine glands associated with mucous membranes, especially the salivary and lacrimal glands pg 961
Sjogren's syndrome presents with
a. asymmetry: one half does not match the other half
The ABCDEs of melanoma identification include which of the following? a. asymmetry: one half does not match the other half b. border: the borders are regular; they are not ragged, notched, or blurred c. color (pigmentation) is uniform d. diameter: the diameter is 5mm
excessive mucus secretion in the bronchial tree (pg 349)
The ATS has defined chronic bronchitis as a clinical d/o characterized by
trigeminal nerve pathology, also the drug carbamazepine results in pain relief and because of this effect, it may be used to dx trigeminal neuralgia pg 139
The MRI is the method of choice in differential dx of:
active and latent infection (pg 371)
The Mantoux testing does not distinguish between
I II II IV V grp I, II, and III can be treated as outpatient (pg 364)
The Pneumonia utcomes Research Team (PORT)-pneumonia severity index is a risk assessment tool to categorize pts into five grps, the grps are:
five stages of CRF (pg 620)
The Third National Health and Nutrition Examination Survey defined:
c.Cough (621 quiz)
The first sign of an asthma exacerbation is: a.Wheeze b.Dyspnea c.Cough d.Chest tightness
b. The typical lesion are bullae
The following statements are all true regarding herpes zoster except? a. Infection of the trigeminal nerve ophthalmic branch can cause corneal blindness. b. The typical lesion are bullae. c. It is due to reactivation of the latent varicella virus. d. It is usually more severe in immunocompromised individuals.
b. a blank stare pg 86
The hallmark of an absence seizure is: a. no activity at all. b. a blank stare. c. urine is usually voided involuntarily. d. the attack usually lasts several minutes.
the higher the degree of stenosis pg 113
The higher the pitch of a bruit:
Overflow incontinence (quiz 621)
The inability to empty the bladder, resulting in overdistention and frequent loss of small amounts of urine, describes which type of urinary incontinence?
Recommendation 9
The main objective of hypertension treatment is to attain and maintain goal BP. If goal BP is not reached within a month of treatment, increase the dose of the initial drug or add a second drug from one of the classes in recommendation 6 (thiazide-type diuretic, CCB, ACEI, or ARB).The clinician should continue to assess BP and adjust the treatment regimen until goal BP is reached. If goal BP cannot be reached with 2 drugs, add and titrate a third drug from the list provided. Do not use an ACEI and an ARB together in the same patient. If goal BP cannot be reached using only the drugs in recommendation 6 because of a contraindication or the need to use more than 3 drugs to reach goal BP, antihypertensive drugs from other classes can be used. Referral to a hypertension specialist may be indicated for patients in whom goal BP cannot be attained using the above strategy or for the management of complicated patients for whom additional clinical consultation is needed. (Expert Opinion - Grade E)
c. IgG pg 199
The majority of HSV-1 and HSV-2 infections are asymptomatic, so that only which elevated antibody titer shows evidence of previous infection? a. IgA b. IgE c. IgG d. IgM
UTI (quiz 621)
The most common cause of sepsis in older adults is:
b. actinic keratosis pg 234
The most common precancerous skin lesion found in Caucasians is: a. a skin tag b. actinic keratosis c. a melanoma d. a basal cell lesion
b. enzyme-linked immunosorbent assay (ELISA), if the ELISA is positive, then a confirmation test with the Western blot is done pg 978
The most cost effective screening test for HIV status is: a. Western blot b. enzyme-linked immunosorbent assay (ELISA)
hematuria (quiz 621)
The most frequent sign of bladder cancer is:
d. Schilling test
The test in which a small, radioactive tracer dose of cyanocobalamin is given by mouth and then a 24 hr urine sample is collected and assayed for radioactivity is the: a. Coombs' test b. oligonucleotide probe test c. sperocytic test d. Schilling test
Stage B heart failure
have structural abnormalities but no symptoms. This grp includes patients with Left ventricular hypertrophy, previous MI, left ventricular systolic dysfunction or valvular heart disease pg 463
Treat all contact lens wearers with a red eye as a corneal ulcer. Close follow up and eye drops of either tobramycin or quinilones. No contacts. Do not patch. No steroid eye drops unless Opthalmology consult obtained and recommends. Corneal abrasions treated with analgesics and eye drops. Do not patch.
Treatment (ppt week 2)
a. Acute blood loss 624 quiz
Under which of the following circumstances is the reticulocyte count elevated? a. Acute blood loss. b. Aplastic anemia. c. Iron-deficiency anemia. d. Poisonings.
an increase in uric acid production or ineffective elimination of uric acid, as found in gout. This may result from dietary intake of foods high in uric acid, acidic urinary pH, (e.g. type I renal tubular acidosis, gout, significant bicarbonate loss associated with severe diarrhea, ulcerative colitis) account for 15-20% of renal calculi (pg603)
Uric acid stones are formed from
a patient usually older than 40 but may occur in younger patients presents with decreased peripheral field of vision, decreased central vision is a later sign, increased intraocular pressure, increased cu-to-disc ratio. Does this require urgent treatment?
Usually no - symptoms of Chronic glaucoma pg 248
coagulation abnormalities abdominal/pelvic surgery - venous pooling/stasis estrogen/oral contraceptives pregnancy - venous congestion obesity - increased cardiac workload, venous pooling heart disease - venous stasis advanced neoplasm - coagulation abnormalities, interference with venous blood flow (pg 484)
Venous risk factor for Peripheral Arterial Disease are:
sensation of rotation or movement of the patient or the patient's surroundings pg 79
Vertigo is the
preauricular lymph nodes are palpable in
Viral conjunctivitis pg 247
pyelonephritis and interstitial nephritis
WBC casts are seen in
a. aspirin pg 117
What antiplatelet agent is most widely used for secondary prevention of stroke? a. aspirin b. ticlopidine c. clopidogrel d. aspirin and clopidogrel
the pt raises and lower their extremities with each of five repetitions for about 2 minutes, the legs should be raised to a 45-degree angle and then lowered again to the supine position (pg 489)
What are Buerger-Allen exercises?
Staphylococcus saprophyticus (+) and Escherichia coli (-) (quiz 621)
What are the two most common pathogens in community-acquired UTIs?
WIQ (walking impairment ques-tionnaire, pg 487)
What assessment tool is used for peripheral artery disease?
peripheral has an auditory-associated symptoms and central vertigo has visual-associated symptoms pgs 79 & 81
What characteristics differentiates peripheral vertigo from central vertigo?
Ascending venogram (pg 487)
What diagnostic test has long been considered the "gold standard" for a diagnosis of venous thromboembolism?
a. A rise in neutrophils. quiz 624
What does a shift to the left or left shift mean? a. A rise in neutrophils. b. A rise in lymphocytes. c. A rise in monocytes. d. A rise in basophils.
age "65" (1 point) pg 364
What does the "65" in CURB-65 stand for:
"B"P (Systolic <90 or diastolic <=60) = 1 point pg 364
What does the "B" in CURB-65 stand for:
Confusion (1 point) pg 364
What does the "C" in CURB-65 stand for:
"R"espiratory rate (>=30breaths/min) = 1 point pg 364
What does the "R" in CURB-65 stand for:
B"U"N> 19mg/dL (1 point) pg 364
What does the "U" in CURB-65 stand for:
a urinalysis (pg 582
What is the easiest, most noninvasive and most economical test to identify a UTI?
a.Penicillin
What is the first-line recommended treatment against Group A-hemolytic streptococci (GABHS), the most common cause of bacterial pharyngitis? a.Penicillin b.Quinolone c.Cephalosporin d.Macrolide
Diabetic nephropathy (quiz 621)
What is the most common cause of chronic renal failure (CRF)?
b. Sun exposure.
What is the most common rosacea trigger? a. Alcohol. b. Sun exposure. c. Cold weather. d. Skin care products
Salmeterol (Serevent) (pg 344)
Which long-term beta-blocker is no longer used by ITSELF because of an increased rate of morbidity and mortality?
spores pg 167
When looking under the microscope to diagnose an intravaginal infection, you see a cluster of small and oval to round shapes. What do you suspect they are?
bacteria pg 167
When looking under the microscope to diagnose an intravaginal infection, you see a small dark specks. What do you suspect they are?
pseudohypae or hyphae (yeast) pg 167
When looking under the microscope to diagnose an intravaginal infection, you see thin translucent walls that have septa dividing each segment, like a bamboo stem What do you suspect they are?
Narrow-angle glaucoma (quiz 621)
When prescribing oxybutinin (Ditropan) for the patient with overactive bladder symptoms, the nurse practitioner must consider which disorder in the medical history before prescribing?
a. magnetic resonance imaging (MRI) pg 135
Which is the most sensitive neuroimaging test to evaluate patients with encephalitis? a. magnetic resonance imaging (MRI) b. computed tomography (CT) c. electroencephalogram (EEG) d. an initial lumbar puncture (LP)
c. ascending venogram (pg 487)
Which diagnostic test has long been considered the "Gold standard" for a diagnosis of venous thromboembolism? a.Ultrasound b.Magnetic resonance imaging (MRI) c.Ascending venogram d.D-dimer
Bile acid sequestrants pg 442
Which drug categories commonly prescribed for hyperlipidemia should NOT be given if the triglycerides are over 400 mg/dL?
a. cholinesterase inhibitors pg 108
Which drug for Alzheimer's disease should be administered beginning at the time of diagnosis? a. cholinesterase inhibitors b. anxiolytics c. antidepressants d. atypical antipsychotics
a.The gum must be correctly chewed to a softened state and then placed in the buccal mucosa (pg 414)
Which information should be included when you are teaching your patient about the use of nicotine gum? a.The gum must be correctly chewed to a softened state and then placed in the buccal mucosa. b.Patients should not eat for 30 minutes prior to or during the use of the gum. c.Initially, one piece is chewed every 30 minutes while awake. d.Acidic foods and beverages should be encouraged during the nicotine therapy.
c. It can be exacerbated by stress.
Which is a true statement regarding psoriasis? a. It is usually worse in the summer. b. It is highly contagious. c. It can be exacerbated by stress. d. All clients have accompanying pruritis.
a. Ferritin. 624 quiz
Which is the best test to perform to spot an iron-deficiency anemia early before it progresses to full-blown anemia? a. Ferritin. b. Reticulocytes. c. Hematocrit. d. Hemoglobin.
b. ganglia transilluminate
how can you differentiate between a ganglion cyst and a neoplasm? a. a neoplasm is more painful b. ganglia transilluminate c. ganglia cause more swelling d. a neoplasm may fluctuate in size
c. oral antibiotics
balanitis may evolve into a chronic problem. if this occurs and the clt experiences severe purulence and phimosis, txment should involve: a. a stronger antifunal ointment b. change from an antifungal ointment to a powder c. oral antibiotics d. an antiviral ointment
tapping the facial nerve below the zygomatic arch anterior to the ear lobe pg 831
how do you test for Chvostek's sign?
with BP cuff 20mmHg above the normal systolic, inflate, hold for 3 minutes pg 830
how do you test for Trousseau's sign?
b. Brittle hair. quiz 624
Which of the following is a sign of hypothyroidism? a. Warm, smooth, moist skin. b. Brittle hair. c. Gynecomastia. d. A thyroid bruit.
a. high levels of lead exposure
Which of the following is associated with an elevated risk of renal cell carcinoma? a. high levels of lead exposure b. several episodes of kidney stones c. frequent urinary tract infections d. a permanent indwelling catheter
d. give folic acid 1mg PO daily
health maintenance in adults with sickle cell anemia includes which of the following: a. early sterilization should be performed to prevent transmission of the disease b. administer hepatitis A vaccine c. avoid use of oral contraceptives because of increased risk of clotting d. give folic acid 1mg PO daily
d. Doxycycline 100 mg po bid X 21 days.
Which of the following is recommended treatment for erythema migrans or early Lyme disease? a. Erythromycin 333mg po tid X 10 days. b. Dicloxacillin 500 mg po bid X 10 days. c. Ciprofloxacin 250 mg po bid X 14 days. d. Doxycycline 100 mg po bid X 21 days.
d. Doxycycline 100 mg po bid X 21 days. 624 quiz
Which of the following is recommended treatment for erythema migrans or early Lyme disease? a. Erythromycin 333mg po tid X 10 days. b. Dicloxacillin 500 mg po bid X 10 days. c. Ciprofloxacin 250 mg po bid X 14 days. d. Doxycycline 100 mg po bid X 21 days.
b. pulsatile pain pgs 120 & 123
Which of the following is the most commonly experienced symptom of a patient with a migraine? a. light sensitivity b. pulsatile pain c. sound sensitivity d. experiencing an aura
c. erysipelas pg 191
Which of the following types of cellulitis is a streptococcal infection of the superficial layers of the skin which does not involve the subcutaneous layers? a. necrotizing fasciitis b. periorbital cellulitis c. erysipelas d. flesh-eating cellulitis
b. periorbital cellulitis pg 191
Which of the following types of cellulitis will have an abnormal exam of the CNs III, IV and VI, high fever, tachycardia, lethargy or mental status changes? a. necrotizing fasciitis b. periorbital cellulitis c. erysipelas d. flesh-eating cellulitis
every 90 days pg 950
how frequently is routine clinical labs completed in those with rheumatoid arthritis?
a. purulent meningitis pg 130
Which type of meningitis has a rapid onset, hrs or days after exposure, usually caused by h. influenzae, n. meningitidis or s. pneumoniae? a. purulent meningitis b. chronic meningitis c. aseptic meningitis d. herpes meningitis
b. chronic meningitis
Which type of meningitis has a slower development of symptoms after exposure, usually caused bym. tuberculosis, atypical mycobacteria, fungi, or spirochetes a. purulent meningitis b. chronic meningitis c. aseptic meningitis d. herpes meningitis
c. aseptic meningitis pg 130
Which type of meningitis is more benign, self-limiting, and caused primarily by a virus? a. purulent meningitis b. chronic meningitis c. aseptic meningitis d. herpes meningitis
T-wave inversion, ST depression, no Q wave (pg 453)
Which typical electrocardiogram (ECG) change is usually seen with NSTEMI?
ST elevation, T wave inversion, significant Q wave (pg 453)
Which typical electrocardiogram (ECG) change is usually seen with STEMI?
ST elevation (pg 452)
Which typical electrocardiogram (ECG) change is usually seen with cardiac injury?
T-wave inversion (pg 452)
Which typical electrocardiogram (ECG) change is usually seen with cardiac ischemia?
the primary goal of therapy is to replace depleted stores of dopamine and a minimum of 3 months trial should be given before determining if the pt does not respond to the medication pg 102
Why is it necessary to start a patient on L-dopa for Parkinson disease?
What is the easiest way to differentiate between otitis externa and otitis media?
With otitis externa, movement or pressure on the pinna is extremely painful
extensively drug resistant TB - TB resistant to both INH and rifampin, as well as fluoroquinolones, and at least one of the injectable second line antiTB agents (kanamycin, capreomycin or amikacin) (pg 376)
XDR-TB is
tagamet, erythromycin, ciprofloxacin, and beta-blockers (pg 355)
Xanthines interact with other drugs commonly taken by COPD patients and are:
Should any patient newly diagnosed with diabetes be referred to an opthalmologist?
Yes pg 272
c. start vigorous exercise immediately (quiz 621)
You are caring for an adult, obese patient who is 5 days post-discharge from a hospitalization due to a deep vein thrombosis. Which instructions would likely NOT minimize recurrence of a thrombosis? a. use support stockings daily b. avoid prolonged inactivity c. start vigorous exercise immediately d. report bleeding of gums and bruising
d. metrorrhagia
menses at irregular intervals with excessive flow and duration is defined as: a. oligomenorrhea b. polymenorrhea c. menorrhagia d. metrorrhagia
b. assess the pedal and posterior tibial pulses (quiz 621)
You are evaluating a 65-year-old male with known peripheral vascular disease and who is a current smoker. What is your initial evaluation? a. order a venogram. b. assess the pedal and posterior tibial pulses. c. order compression stockings. d. order a venous reflex study.
d. I should wear rubber shoes in the shower to prevent transmission to others
You are teaching M., age 18, about his tinea pedis. You know he doesn't understand your directions when he tells you which of the following? a. I should dry between my toes every day b. I should wash my socks with bleach c. I should use an antifungal powder twice a day d. I should wear rubber shoes in the shower to prevent transmission to others
40-60mg/day as a single dose or in two divided doses over 3-10 days; tapering is not needed (pg 346)
a "short burst" of systemic corticosteroids is
a. smoking cessation, avoid cold weather & c. wear gloves and thick socks (quiz 621)
You are treating a 35-year-old female with the diagnosis of Raynaud's phenomenon. She is a current cigarette smoker who works in a warehouse without heating or cooling. What education may you provide for her regarding Raynaud's? Select all that apply. a. smoking cessation, avoid cold weather. b. take warm showers daily. c. wear gloves and thick socks. d. change jobs.
b. An office spirometry (621 quiz)
You are treating a patient in your office whom you suspect has chronic lung disease, but you are unsure if you should start treatment today. What do you consider ordering first? a. A pulmonary function test b. An office spirometry c. A pulmonary consult d. A chest x-ray
d. You educate the patient that, first, she should exhale fully, then shake the inhaler, and finally, as she presses the canister, inhale slowly and fully. (621 quiz)
You are treating an adult asthma patient and reviewing her metered dose inhaler (MDI) technique. She demonstrates her MDI technique by shaking the inhaler, placing the inhaler in her mouth, pressing the canister, and inhaling quickly. What is your response? a. You praise the patient for demonstrating the correct MDI technique. b. You educate the patient that she should always use a spacer or holding chamber with all MDI's. c. You instruct the patient to take a faster, even inspiratory effort. d. You educate the patient that, first, she should exhale fully, then shake the inhaler, and finally, as she presses the canister, inhale slowly and fully.
Asthma GERD Postnasal drip Sinusitis (621 quiz)
You are treating an elderly patient who tells you he has a history of allergies. His concern today is he is coughing more at night for the past 6 weeks and wheezing intermittently. He has no history of heart failure, smoking, or other lung disease. Your physical examination is unremarkable. What are your differential diagnosis(es) for cough? Select all that apply. Asthma GERD Postnasal drip Sinusitis
a. Consider home treatment (pg 364)
You are using the CURB-65 clinical prediction tool to decide whether Janet whom you have diagnosed with community-acquired pneumonia (CAP) should be hospitalized or could be treated at home. Her score is 1. What should you do? a. Consider home treatment. b. Plan for a short inpatient hospitalization. c. Closely supervise her outpatient treatment. d. Hospitalize and consider admitting her to the intensive care unit (ICU).
b. Plan for a short inpatient hospitalization. c. Closely supervise her outpatient treatment.
You are using the CURB-65 clinical prediction tool to decide whether Janet whom you have diagnosed with community-acquired pneumonia (CAP) should be hospitalized or could be treated at home. Her score is 2. What should you do? a. Consider home treatment. b. Plan for a short inpatient hospitalization. c. Closely supervise her outpatient treatment. d. Hospitalize and consider admitting her to the intensive care unit (ICU).
d. Hospitalize and consider admitting her to the intensive care unit (ICU) (pg 364)
You are using the CURB-65 clinical prediction tool to decide whether Janet whom you have diagnosed with community-acquired pneumonia (CAP) should be hospitalized or could be treated at home. Her score is 3. What should you do? a. Consider home treatment. b. Plan for a short inpatient hospitalization. c. Closely supervise her outpatient treatment. d. Hospitalize and consider admitting her to the intensive care unit (ICU).
background diabetic retinopathy,
microaneurysms, intraretinal hemorrhage, macular edema, and lipid deposits may be apparent pg 273
b. Halcinonide 1% ointment bid for 2 weeks.
You have diagnosed Tom with contact dermatitis on the left side of the face secondary to poison ivy. You would recommend? a. Hydrocortisone cream 1% bid until healed. b. Halcinonide 1% ointment bid for 2 weeks. c. Clotrimazole cream bid for 2 weeks. d. Washing with antibacterial soap bid to reduce risk of secondary infection until healed.
b. bacterial vaginosis
You have just finished a Pap smear on Sally, age 39. during the wet mount, you see cells with bacteria adherent to the cell wall giving it a stippled, granular appearance. What do you suspect? a. candidiasis b. bacterial vaginosis c. trichomoniasis d. cervicitis
b. Use your rescue inhaler, begin the prescription of oral glucocorticoids you have, and call back tomorrow
You have taught Amy, aged 15, about using a flow meter to assess how to manage her asthma exacerbations. She calls you today because her peak expiratory flow rate (PEFR) is 65%. What would you tell her? a."Take your short-acting beta-2 agonist, remain quiet, and call back tomorrow." b."Use your rescue inhaler, begin the prescription of oral glucocorticoids you have, and call back tomorrow." c."Drive to the emergency room (ER) now." d."Call 911."
a. warm compresses, elevate extremity, naproxen sodium 500mg BID with food (quiz 621)
You review a lower extremity duplex study to discover the diagnosis of a non-septic, superficial thrombophlebitis. What is the recommended treatment? a. warm compresses, elevate extremity, naproxen sodium 500mg BID with food b. hospital admission, IV antibiotics c. cool compresses, elevate extremity, ibuprofen 400mg TID, Keflex 500mg TID for 10 days d. hospital admission, IV heparin, then warafarin PO for 3 to 6 months
True pg 127
You should not use caffeine and ergotamine if you have coronary heart disease, high blood pressure, circulation problems, liver or kidney disease, or a serious infection called sepsis True False
c. "I should wear rubber shoes in the shower to prevent transmission to others."
You teach Mitch about his tinea pedis. Which of the following statements indicates he doesn't understand your directions? a.I should use an antifungal powder twice a day." b. "I should dry between my toes every day." c. "I should wear rubber shoes in the shower to prevent transmission to others." d. "I should wash my socks with bleach."
d. Hodgkin's lymphoma. quiz 624
Your client, age 60, presents with pruritus and complains of lymphadenopathy in his neck. He also complains of night sweats and has noticed a low-grade fever. He has not lost any weight and otherwise feels well. He is widowed and has been dating recently. On physical exam you notice enlarged supraclavicular nodes. You suspect? a. Lung cancer. b. Non-Hodgkin's lymphoma. c. A lingering viral infection from a bout of flu he had 6 weeks ago. d. Hodgkin's lymphoma.
b. hodgkin's lymphoma
Your clt, George, age 60 presents with pruritus and c/o of lymphadenopathy in his neck. He also c/o of night sweats and noticed a low-grade fever. He has not lost any wt and otherwise feels well. He is widowed and has been dating a new woman recently On PE, you find enlarged supraclavicular nodes. a. lung cancer b. hodgkin's lymphoma c. a lingering viral infection from a bout of flu he had 6 wks ago c. non-Hodgkin's lymphoma
c.His quit date should be in 8 days to allow a steady-state plasma level of bupropion to be reached before smoking cessation starts (pg 412)
Your patient has decided to give Bupropion a try to quit smoking. You are discussing his quit date, and he will begin taking the medicine tomorrow. When should he plan to quit smoking? a.He should stop smoking today. b.He should stop smoking tomorrow. c.His quit date should be in 8 days d.He will be ready to quit after the first 30 days.
c.His quit date should be in 1 week (pg 412)
Your patient has decided to give Varenicline (Chantix) a try to quit smoking. You are discussing his quit date, and he will begin taking the medicine tomorrow. When should he plan to quit smoking? a.He should stop smoking today. b.He should stop smoking tomorrow. c.His quit date should be in 1 week. d.He will be ready to quit after the first 30 days.
c. 5 to 15 µg/mL (pg 346)
Your patient is on Theobid for his asthma. You want to maintain his serum levels between: a. 0 to 5 µg/mL. b. 5 to 10 µg/mL. c. 5 to 15 µg/mL. d. 10 to 20 µg/mL.
an increase of alveolar ventilation, resulting in a decrease of carbonic acid
respiratory alkalosis occurs with
gemifloxacin levofloxacin moxifloxacin **all fluoroquinolones carry a risk of tendonitis and tendon rupture and are not used in those younger than 18 (pg 365)
respiratory fluoroquinolones are:
b. pernicious anemia
hemolytic anemia may be an inherited condition. Which of the following is not an inherited condition related to hemolytic anemia? a. hereditary spherocytosis b. pernicious anemia c. glucose-6-phosphate dehydrogenase deficiency d. sickle cell anemia
diabetic retinopathy is divided into three stages,
background diabetic preproliferative diabetic proliferative diabetic pg 273
diastolic dysfunction is also known as
backward failure pg 461
b.phenylephrine injection 0.3-0.5mL into the corpora cavernosa
b. presents to the ER with a dx of priapism. despite application of cold compresses and pain meds, relief is unsuccessful. what is the txment of choice? a. terbutaline 0.25mg SQ b.phenylephrine injection 0.3-0.5mL into the corpora cavernosa c. doxazosin 5mg SL d. lidocaine 1% via the glans
d. during the neonatal period
herpes implex virus can be potentially acquired through maternal transmission. This is least likely to occur: a. before labor b. during delivery c. postnatally d. during the neonatal period
Stage A heart failure
high risk but do not have structural abnormalities or symptoms, may have HTN, diabetes, CAD, family hx, or exposure to cardiotoxic drugs pg 463
Yes
can pts with pneumonia have some GI sxs like nausea, vomiting, and diarrhea? Yes No
yes, because vitamin B6 is needed for the biosynthesis of GABA pg 88
can vitamin B6 deficiency lead to seizures?
no, propecia is for men only pg 147
can women use propecia for alopecia?
b. sudden onset of weakness of the legs in a man with known prostate cancer
cancer of the prostate often begins with subtle sxs that develop very slowly over time and, if left untxed, will lead to metastasis. what prognostic finding is a significant indicator of probable metastatic disease? a. Gleason score of 2 b. sudden onset of weakness of the legs in a man with known prostate cancer c. TNM staging T1a, N0, M0 d. bladder outlet obstruction
c. women with diabetes
candidiasis is more common in: a. teenage girls b. women on low fat diets c. women with diabetes d. women with frequent UTIs
a. balanitis
candidiasis may occur in many parts of the body. J., age 29, has it in the the glans of his penis. What is your dx? a. balanitis b. thrush c. candidal paronychia d. subungual
Prerenal
cardiovascular failure is a major cause of which type of acute renal failure?
a. painful contraction of the hands or feet pg 830
carpopedal spasm is: a. painful contraction of the hands or feet b. painful flexion of the hands or feet c. spasm of the car pool
d. premenarchial onset, obesity, hyperinsulinemia
characteristics of polycystic ovary syndrome include: a. hirsutism, thinness, hypoinsulinemia b. menopausal onset, vitilgo, hyperinsulemia c. alopecia, thinness, abdominal cramping d. premenarchial onset, obesity, hyperinsulinemia
True (pg 362)
chest x-ray films may be normal when the pt is unable to mount an inflammatory response, is in the early stages of an infiltration process or in PCP associated with AIDS True or False?
atypical pneumonia, little isknown about the mode of transmission and pathogenesis (pg 360)
chlamydia pneumonia is caused by a gram negative bacterium but is also known as an
1/3 of all cases of physical urticarias pg 156
cholinergic urticaria accounts for
longer than 3 weeks (pg 331)
chronic cough lasts
of functioning nephrons, eventually leading to end-stage renal disease (ESRD) - the time frame is typically months to years (pg 617)
chronic renal failure is characterized by a progressive loss of
m. tuberculosis atypical mycobacteria fungi spirochetes SYMPTOMS develop over months; less acutely ill pg 130
chronic subacute meningitis are usually caused by:
Up to 1/2 of potentially fatal sebaceous cell carcinomas resemble
chronic, benign inflammatory disease such as a chalazion and blepharoconjunctivitis pg 251
25-30% correlation with the development of renal cell carcinoma (pg 607)
cigarette smoking has a
80-90% of COPD (pg 349)
cigarette smoking is responsible for
ankle/brachial index (ABI) reading of 0.6 to 0.9 indicates
claudication pg 487
anaerobic/cavitary pneumonia (pg 367)
clindamycin (cleocin) is an effective drug for pts with
a. tetany, hypotension, tachycardia, confusion, decreasing level of consciousness, hyperreflexia, dysrhythmias, sezisures and respiratory failure
clinical manifestation of metabolic alkalosis include: a. tetany b. nausea & vomiting c. weakness d. bradycardia
renal mass
clinical manifestations including microscopic or gross hematuria, a palpable abdominal mass, fever, and flank pain may indicate?
a. pain and paresthesia
clinical presentation of acute lower extremity atherosclerotic arterial disease most likely includes: a. pain and paresthesia b. pallor and pulselessness c. poikilothermy d. paralysis or loss of limb strength
d. spider varicosities
clinical presentation of advanced lower-extremity vascular disease includes all of the following except: a. resting pain b. absent posterior tibialis pulse c. blanching of the foot with elevation d. spider varicosities
Pain Discharge Vertigo Dizziness Tinnitus Hearing Loss
clinical presentation of ear disorders (ppt week 2
d. UTI
cloudy urine is usually indicative of which of the following conditions? a. diabetes b. proteinuria c. malignancy d. UTI
simple partial seizures complex partial seizures complex partial seizures that evolve into generalized tonic-clonic convulsions pg 86
how are partial seizures classified?
2 to 3mm (pg 386)
how big must the tumor nodule be before it is visible on a chest x-ray?
0 - absent 1+ diminished 2+ normal 3+ bounding (pg 487)
grading of pulses are:
in a wide variety of renal disorders
granular casts may be present
c. crepitus
grating of the bones or entrance of air into an open fx is manifested as: a. swelling b. ecchymosis c. crepitus d. pain and tenderness
bladder cancer (pg 584)
gross painless hematuria is a cardinal symptom of certain malignancies such as
URIs and some acute viral syndromes pg 908
fevers between 101.3F (38.5C) and 104F (40C) tend to be associated with
pancreatitis, UTIs, and intracranial pathology pg 908
fevers in excess of 104F (40C) tend to be associated with
hepatitis, some acute viral infections, and TB pg 908
fevers less than 101.3 (38.5C) are characteristic of
women between the ages of 20-55 pg 954
fibromyalgia syndrome is now consider to be the most common cause of generalized muscle pain in
5-alpha-reductase inhibitor that blocks conversion of testosterone to dihydrotestosterone, it reduces the size of the prostate, relieves pressure on the bladder and urethra, reduces the risk of urinary retention and results in a reduction of the sxs of BPH, usually 6 months or more of txment are required for maximal effects
finasteride (proscar) is a
liver and should be used cautiously in those with liver disease pg 147
finasteride is metabolized via the
INH rifampin pyrazinamide ethambutol streptomycin (pg 375)
first line TB drugs are:
c. a combination of diet modification, wt loss, and stress management
first line tx for polycystic ovary syndrome is: a. a bilateral oophorectomy b. oral testosterone therapy c. a combination of diet modification, wt loss, and stress management d. a laparoscopy with a bilateral wedge resection
a. NSAIDs
first-line drug tx for acute low back pain includes the use of: a. NSAIDs b. muscle relaxants c. opioids d. antidepressants
co-morbidities and/or those who have suspected macrolide resistant strains of Streptococcus
fluoroquinolones are commonly used first line for patients with
combination inhaled corticosteroids and LABAs
fluticasone and salmeterol (advair diskus) and budesonide and formoterol (symbicort) are a
of the hair follicles pg 184
folliculitis is a superficial to deep skin infection of what?
postnasal drip; postinfection; COPD; Asthma; GERD; Drugs; Occupational/environmental; Tumors, pulmonary embolism, aortic aneurysm, CHF, TB, sarcoidosis, foreign body, lung absess, Pneumocystis jirovecii and psychogenic factors (pg 332)
following conditions that may cause a cough are:
heparin IV. an initial IV bolus of 5000-10,000 units per/her followed by a continuous infusion at 1000 units per hr. The dosage of heparin should be adjusted according to the partial thromboplastin time (PTT). The goal is to achieve a PTT of two times the control value. Warfarin should be started at 5 to 10 mg daily PO (pg 490)
for deep vein thrombosis, pts must be hospitalized initially to begin agressive anticoagulant therapy with
c. the clt should stand, and the examiner should assume a seated position in front of the clt
for inspecting the male genitalia, what is the ideal position for the clt and examiner? a. the clt should be in a modified lithotomy position with the examiner at the foot of the table b. the clt should be in the dorsal recumbent position and the examiner should approach from the left side c. the clt should stand, and the examiner should assume a seated position in front of the clt d. the clt and examiner should both stand
1.5 to 2 times the control or about 15-17 second (pg 490)
for superficial thrombophlebitis, the dose of warfarin (Coumadin) should be adjusted to maintain a therapeutic PT which is:
d. if you have additional pain, an occasional acetaminophen is permitted in between the usual doses of the NSAID
for your clt with a knee injury, you order an NSAID to be taken on a routine basis for the next 2 wks. your teaching should include which of the following? a. you may take this medication on an empty stomach as long as you eat within 2-3 hrs of taking it b. if one pill does not seem to help, you can double the dose for subsequent doses c. if you notice nausea/vomiting or black or bloody stools, take the next dose with a glass of milk or a full meal d. if you have additional pain, an occasional acetaminophen is permitted in between the usual doses of the NSAID
systolic dysfunction is also known as
forward failure pg 461
reducing protein in the diet
general recommendations for the prevention of kidney stones, regardless of the type of stone the clt has, include:
a. children
generalized absence seizures usually occur in which age grp? a. children b. adolescents c. middle-aged adults d. older adults
systemic steroids may interfere with
glaucoma control pg 272
renal failure (pg 619)
glomerulonephritis is the 3rd most common cause of
c. podagra
gouty pain in the great toe is: a. toe gout b. hyperuricemia of the toe c. podagra d. tophus
before initiation of TB treatment and while being tx (pg 372)
liver function tests should be gotten
bacterial infection
lobar infiltrates strongly suggest
for maintenance therapy to prevent acute bronchospastic episodes ****they should not be used without inhaled steroids**** ( pg 354)
long acting beta-2 agonist are not first-line therapy for COPD but may be used
not be used as a rescue inhaler should not be used without inhaled corticosteroids is appropriate for pts with moderate to severe asthma and can be used before sleep to prevent nocturnal attacks may be helpful in preventing exercise induced asthma (pg 344)
long acting beta-2 agonist should
inhibt sodium chloride reabsorption inthe thick ascending limb of the loop of Henle
loop diuretics such as furosemide, bumetanide, and ethacrynic acid all act to
permethrin, lindane, crotamiton, or sulfur pg
lotions containing scabicides are
a. radical prostatectomy or radiation
low-grade, localized prostate cancer can be txed successfully with: a. radical prostatectomy or radiation b. chemotherapy c. cryosurgery (freezing) of a small part of the gland d. watchful waiting
d. urgency and nocturia
lower urinary tract sxs in males can present as a constellation of storage or voiding sxs. storage sxs include: a. hesitancy and poor flow b. intermittency and postvoid dribble c. straining and dysuria d. urgency and nocturia
C, so caution should be used when discussing these products for tx of asthma (pg 349)
lycopene, vit B12, vit C, and goldenseal are rated at level
equal or greater than 100 fL.
macrocytic anemia is defined as having an MCV
d. a deficiency of folic acid
macrocytic normochromic anemias are caused by: a. acute blood loss b. an infection or tumor c. a nutritional deficiency of iron d. a deficiency of folic acid
azithromycin clarithromycin erythromycin (pg 365)
macrolides include:
uncomplicated pneumonia in outpatients who are otherwise healthy and have not had recent antibiotic exposure
macrolides like azithromycin or clarithromycin or a tetracylinc like doxycyline is used for initial tx of
1cm or less, nonpalpable, caused by changes in skin pigmentation pg 153
macules are
d. stroke
major depression occurs most often in which of the following conditions? a. Parkinson's disease b. Alzheimer's disease c. myocardial infarction d. stroke
hypothyroidism, Cushing's syndrome, renal, hepatic disease, CHF, premenstrual syndrome, pregnancy, medication use, depression, and excessive calorie intake pg 835
major differential dx for weight gain are:
deadliest of skin cancers pg 149, 236
malignant melanoma is the
a. giving psychotropic drugs, such as amitriptyline in a low dose at bedtime
management of fibromyalgia would include: a. giving psychotropic drugs, such as amitriptyline in a low dose at bedtime b. instructing clts to keep as busy as possible to keep their minds off the sxs c. using high doses of NSAIDs d. avoiding exercise
osmotic diuretic
mannitol (osmitrol) is an
Dry eye occurs due to
mechanical abnormalities lacrimal gland malfunction mucin deficiency pg 245
melanocyte cells that determines skin color pg 147
melanin is skin pigment and is produced by
c. immobilization and malignancy
pregnant women may be prone to thrombophilias, which may be inherited or acquired. Which of the following is an example of a factor that predisposes pregnant women to acquired thrombophilic states in pregnancy? a. factor V Leiden b. homocystine c. immobilization and malignancy d. protein S and protein C
decrease in renal perfusion, e.g. hypovolemia, renovascular disease, decreased cardiac output, impaired renal autoregulations of blood flow, which is often associated with ACEi or NSAIDs (pg 612)
prerenal azotmeia is the name given to ANY condition that leads to an overall
age related hearing loss is termed
presbycusis and is a form of sensorineural hearing loss pg 276
True
prescribing a direct thrombin inhibitor is an acceptable therapeutic option to reduce the risk of recurrent DVT True or False
Otitis Media with Effusion (OME) is defined as
presence of middle ear fluid without acute signs of illness or inflammation of the middle ear mucosa (uptodate)
c. a dilated vessel
presentation of superficial venous thrombophlebitis usually includes: a. positive Homan's sign b. diminished dorsalis pedis pulse c. a dilated vessel d. dependent pallor
1. person with known HIV infection, suspected of having HIV, HIV status is unknown 2. close contacts of persons with newly dx infectious TB 3. person with fibrotic changes on chest x-ray exam consistent with old TB 4. recent TB skin test converters 5. persons with medical conditions that increase the risk of TB with a PPD 10mm or greater, e.g. diabetes, prolonged therapy with adrenocorticosteroids, immunosuppressive therapy, hematologic and reticuloendothelial diseases, IV drug user, end-stage-kidney-disease (pg 376)
preventive TB therapy should be given to what grp of people?
9 months of INH - 300mg daily
preventive tx for those with LTBI is
albumin and globulin (pg 584)
primary protein found in urine are
lungs, liver, and bone (pg 607)
primary renal malignancies usually spread to the
a. antiviral chemotherapy can control the signs and symptoms
principles of management of genital herpes include which of the following? a. antiviral chemotherapy can control the signs and symptoms b. antiviral chemotherapy, if prescribed early in a first clinical episode is curative c. antiviral chemotherapy does not control recurrent episodes d. antiviral topical tx offers minimal clinical benefit
iritis requires
prompt attention by an opthalmologist pg 261
d. trimethoprim-sulfamethoxazole (TMP-SMZ) DS 1 tab daily for 10 days
prophylaxis for the first episode of Pneumocystis jiroveci pneumonia in an adult or adolescent clt infected with HIV is: a. isoniazid (nydrazid) 300mg PO and pyridoxine (vitamin B6 50mg PO daily for 12 days b. clarithromycin 500mg PO bid for 2 wks c. rifampin 600mg PO daily for 12 months d. trimethoprim-sulfamethoxazole (TMP-SMZ) DS 1 tab daily for 10 days
c. human cytomegalovirus
prostate cancer is associated with which of the following viruses? a. herpes simplex virus types 1 and 2 b. human herpesvirus 6 c. human cytomegalovirus d. human T-lymphotropic viruses
symptom and should elicit an investigation of the possible causes pg 149
pruritus is a
c. NSAIDs
psoriasis may occur after months of using: a. vitamin b. hormone replacement therapy c. NSAIDs d. antihistamine nasal sprays
2-3 weeks (pg 379)
pts are typically considered infectious for how long after starting tx?
blood and sputum eosinophilia, positive allergy skin test results, and elevated levels of serum IgE and a hx of allergy and bronchodilator response (pg 355)
pts who are likely to respond to steroids are those who have
most infectious to others (pg 368)
pts with acid-fast staining organisms in their sputum are
lung cancer (pg 352)
pts with chronic bronchitis and have smoked 20+ years should be evaluated for
b. all exhibit cough
pts with cough variant asthma: a. all exhibit wheezing b. all exhibit cough c. may exhibit cough or wheeze d. have dyspnea
sexually transmitted disease and pts should be screens for syphilis and HIV pg 162
pubic lice is considered a
pregnant women or women who could become pregnant (pg 375)
pyrazinamide (PZA or P) should not be given to
The most important thing for the clinician to know in the case of eye pain, and in dealing with eye problems in general is
referral to an opthalmologist pg 246
a reduction of alveolar ventilation resulting in an accumulation of carbonic acid
respiratory acidosis occurs with